Download as pdf or txt
Download as pdf or txt
You are on page 1of 274

Civil Law Q&As (2007-2013) hectorchristopher@yahoo.com dbaratbateladot@gmail.

com

A Compilation of the

Questions and Suggested Answers

In the

PHILIPPINE BAR EXAMINATIONS 2007-


2013

In

CIVIL LAW
Compiled and Arranged By:

Baratbate-Ladot, Delight

Salise, Hector Christopher “Jay-Arh” Jr. M.

(University of San Jose-Recoletos School of Law)

ANSWERS TO BAR EXAMINATION QUESTIONS by the


UP LAW COMPLEX (2007-2013)

&

PHILIPPINE ASSOCIATION OF LAW SCHOOLS (2008)

“Never Let The Odds Keep You From Pursuing What You Know In Your Heart You Were Meant To Do.”-Leroy Satchel Paige
Page 1 of 180
Civil Law Q&As (2007-2013) hectorchristopher@yahoo.com dbaratbateladot@gmail.com

FOREWORD
This work is a compilation of the ANSWERS TO BAR
EXAMINATION QUESTIONS by the UP LAW COMPLEX ,
Philippine Association of Law Schools from 2007-2010 and
local law students and lawyers’ forum sites from 2011-2013
and not an original creation or formulation of the author.

The authors were inspired by the work of Silliman University’s


College of Law and its students of producing a very good
material to everyone involved in the legal field particularly
the students and the reviewees for free. Hence, this work is a
freeware.

Everyone is free to distribute and mass produce copies of this


work, however, the author accepts no liability for the content
of this reviewer, or for the consequences of the usage, abuse,
or any actions taken by the user on the basis of the
information given.

The answers (views or opinions) presented in this reviewer


are solely those of the authors in the given references and do
not necessarily represent those of the authors of this work.

The Authors.

“Never Let The Odds Keep You From Pursuing What You Know In Your Heart You Were Meant To Do.”-Leroy Satchel Paige
Page 2 of 180
Civil Law Q&As (2007-2013) hectorchristopher@yahoo.com dbaratbateladot@gmail.com

TABLE OF CONTENTS
(Titles are based on Silliman’s Compilation [Arranged by Topic])

Persons
Capacity: Juridical Capacity (2008)......................................................................................................12

Capacity; Juridical Capacity of Donee; Requisites for Acceptance (2012)...................12

Capacity: Legal Capacity; Lex Rei Sitae (2007).............................................................................13

Correction of Entries; Clerical Error Act (2008)............................................................................14

Nationality Principle (2009)......................................................................................................................14

Nationality Principle; Change of Name not Covered (2009)..................................................15

Conflict of Laws
Processual Presumption (2009)..............................................................................................................16

Jurisdiction; Courts may Assume Jurisdiction over Conflict of Laws Cases (2010). 17

Adoption
Adoption; Termination; Death of Adopter (2009).........................................................................17

Adoption; Illegitimate Child (2010)...................................................................................................... 18

Adoption; Illegitimate Child; Use of Mother’s Surname as Middle Name (2012)....19

Consent; Consent of the Adopter’s Heirs (2008).........................................................................19

Qualifications of Adopter (2010)...........................................................................................................20

“Never Let The Odds Keep You From Pursuing What You Know In Your Heart You Were Meant To Do.”-Leroy Satchel Paige
Page 3 of 180
Civil Law Q&As (2007-2013) hectorchristopher@yahoo.com dbaratbateladot@gmail.com

Family Code
Marriage; Annulment; Grounds (2009)..............................................................................................20

Marriage; Annulment; Grounds (2007)..............................................................................................21

Marriage; Annulment; Parties (2012)..................................................................................................22

Marriage; Annulment; Support Pendente Lite (2010)...............................................................22

Marriage; Divorce Decrees; Filipino Spouse Becoming Alien (2009)..............................23

Marriage; Divorce Decrees; Foreign Spouse Divorces Filipino Spouse (2012)..........24

Marriage; Divorce Decrees; Foreign Spouse Divorces Filipino Spouse (2010)..........25

Marriage; Legal Separation; Prescription (2012)........................................................................25

Marriage; Legal Separation; Prescription (2007)........................................................................26

Marriage; Psychological Incapacity (2013).....................................................................................26

Marriage; Psychological Incapacity (2012).....................................................................................28

Marriage; Requisites (2008).....................................................................................................................28

Marriage; Subsequent Marriage (2008)............................................................................................29

Marriage; Void Marriages; By Reason of Public Policy (2008).............................................30

Marriage; Void Marriages; By Reason of Public Policy (2007).............................................30

Marriage; Void Marriages; Property Relations (2009)..............................................................30

Marriage; Void Marriages; Status of Children (2009)................................................................31

Parental Authority; Illegitimate Minor Child (2009).................................................................. 32

Parental Authority; In Vitro Fertilization (2010)..........................................................................32

“Never Let The Odds Keep You From Pursuing What You Know In Your Heart You Were Meant To Do.”-Leroy Satchel Paige
Page 4 of 180
Civil Law Q&As (2007-2013) hectorchristopher@yahoo.com dbaratbateladot@gmail.com

Paternity & Filiation; Child Born Under a Void Marriage (2010)...................................... 33

Paternity & Filiation; Impugning Legitimacy (2010)..................................................................34

Paternity & Filiation; In Vitro Fertilization;

Surrogate Mother’s Remedy to Regain Custody (2010)..............34

Paternity & Filiation; Legitimacy; Presumption (2008)...........................................................35

Paternity & Filiation; Legitimation of a Child from a Previous Valid Marriage


(2008).....................................................................................................................................................................36

Paternity & Filiation; Legitimation of a Dead Child (2009)................................................... 37

Paternity & Filiation; Support: Ascendants & Descendants;

Collateral Blood Relatives (2008)..............................................................37

Paternity & Filiation; Use of Surname; Illegitimate Child (2009)......................................38

Paternity & Filiation; Who May Impugn Legitimacy (2009)..................................................39

Property Relations; Adulterous Relationship (2009).................................................................39

Property Relations; Accession (2012).................................................................................................40

Property Relations; Ante-Nuptial Debt (2007)..............................................................................40

Property Relations; Unions Without Marriage (2012)...............................................................41

Property Relations; Unions Without Marriage (2012)...............................................................42

Property Relations; Void Marriages (2010).....................................................................................43

Property Relations; Void Marriages (2010).....................................................................................45

“Never Let The Odds Keep You From Pursuing What You Know In Your Heart You Were Meant To Do.”-Leroy Satchel Paige
Page 5 of 180
Civil Law Q&As (2007-2013) hectorchristopher@yahoo.com dbaratbateladot@gmail.com

Succession
Disposition; Mortis Causa vs. Intervivos; Corpse (2009)........................................................46

Heirs; Fideicommissary Substitution (2008)..................................................................................46

Heirs; Intestate Succession; Legitime; Computation (2010)...............................................47

Heirs; Representation; Iron-Curtain Rule (2012).........................................................................49

Heirs; Reserva Troncal (2009).................................................................................................................49

Intestate Succession (2008)......................................................................................................................50

Intestate Succession (2008)......................................................................................................................51

Intestate Succession; Rights of Representation:

Illegitimate, Adopted Child; Iron Curtain Rule (2007)..................51

Legitimes; Compulsory Heirs (2012)...................................................................................................53

Legitime; Compulsory Heirs (2008)......................................................................................................53

Preterition; Disinheritance (2008)........................................................................................................54

Succession; Proof of Death between persons called to succeed each other (2008)
....................................................................................................................................................................................55

Succession; Rule on Survivorship (2009).........................................................................................56

Wills; Holographic Wills; Insertions & Cancellations (2012).................................................57

Wills; Holographic Wills; Probate (2009).......................................................................................... 57

Wills; Joint Wills (2008)................................................................................................................................59

Wills; Joint Wills; Probate (2012)...........................................................................................................59

“Never Let The Odds Keep You From Pursuing What You Know In Your Heart You Were Meant To Do.”-Leroy Satchel Paige
Page 6 of 180
Civil Law Q&As (2007-2013) hectorchristopher@yahoo.com dbaratbateladot@gmail.com

Wills; Prohibition to Partition of a Co-Owned Property (2010)...........................................60

Wills; Notarial Wills; Blind Testator; Requisites (2008)...........................................................61

Wills; Testamentary Disposition; Period to Prohibit Partition (2008).............................61

Wills; Witnesses to a Will, Presence required; Thumbmark as Signature (2007). .62

Donation
Donations; Formalities; In Writing (2007)........................................................................................63

Donations; Illegal & Impossible Conditions (2007)....................................................................64

Donation; Inter Vivos (2013).....................................................................................................................64

Property
Accretion; Alluvium (2008)........................................................................................................................65

Accretion; Rights of the Riparian Owner (2009)..........................................................................67

Builder; Good Faith; Requisites (2013)..............................................................................................68

Easement; Prescription; Acquisitive Prescription (2009).......................................................70

Easement; Right of Way (2013)..............................................................................................................70

Easement; Right of Way (2010)..............................................................................................................72

Hidden Treasure (2008)...............................................................................................................................73

Mortgage; Public or Private Instrument (2013)............................................................................74

Occupation vs. Possession (2007)........................................................................................................76

Ownership; Co-Ownership (2009)..........................................................................................................76

Ownership; Co-Ownership (2008)..........................................................................................................77

“Never Let The Odds Keep You From Pursuing What You Know In Your Heart You Were Meant To Do.”-Leroy Satchel Paige
Page 7 of 180
Civil Law Q&As (2007-2013) hectorchristopher@yahoo.com dbaratbateladot@gmail.com

Property; Movable or Immovable (2007)...........................................................................................78

Land Titles and Deeds


Acquisition of Lands; Sale of Real Property to an Alien (2009).........................................79

Non-Registrable Properties (2007).......................................................................................................80

Prescription; Acquisitive Prescription (2008)................................................................................81

Prescription; Judicially Foreclosed Real Property Mortgage (2012)................................82

Purchaser in Good Faith; Mortgaged Property (2008)............................................................. 83

Registration; Governing Law (2007)....................................................................................................84

Registration; Party Who First took Possession (2013)............................................................85

Registration; Requisites; Proof (2013)...............................................................................................86

Remedies; Fraud; Rights of Innocent Purchaser (2009)..........................................................89

Contracts
Contract to Sell vs. Conditional Contract of Sale (2012)........................................................90

Rescission of Contract; Fortuitous Event (2008).........................................................................90

Stipulation; Arbitration Clause (2009)................................................................................................91

Obligations
Extinguishment; Compensation (2009)..............................................................................................91

Extinguishment; Compensation (2008)..............................................................................................92

Extinguishment; Novation (2008).........................................................................................................93

Extinguishment; Payment of Check (2013).....................................................................................94

“Never Let The Odds Keep You From Pursuing What You Know In Your Heart You Were Meant To Do.”-Leroy Satchel Paige
Page 8 of 180
Civil Law Q&As (2007-2013) hectorchristopher@yahoo.com dbaratbateladot@gmail.com

Extinguishment; Payment of Check; Legal Tender (2008).....................................................95

Liability; Solidary Liability (2008).........................................................................................................96

Obligations; Without Agreement (2007).......................................................................................... 97

Trust
Trust De Son Tort (2007).............................................................................................................................98

Sales
Condominium Act; Partition of a Condominium (2009)............................................................99

Mortgage; Equitable Mortgage (2012)...............................................................................................99

Option Contract; Liquor & “Pulutan” as Consideration (2013)........................................100

Right of First Refusal; Lessee; Effect (2008)................................................................................101

Lease
Builder; Good Faith; Useful Improvements (2013)...................................................................103

Lease; Caveat Emptor (2009)................................................................................................................104

Agency
Agency; Sale of a Real Property through an Agent (2010).................................................104

Partnership
Liability; Liability of a Partner (2010)..............................................................................................105

Oral Partnership (2009)............................................................................................................................106

Share; Demand during the Existence of Partnership (2012)..............................................107

“Never Let The Odds Keep You From Pursuing What You Know In Your Heart You Were Meant To Do.”-Leroy Satchel Paige
Page 9 of 180
Civil Law Q&As (2007-2013) hectorchristopher@yahoo.com dbaratbateladot@gmail.com

Commodatum & Mutuum


Mutuum; Interest; Solutio Indebiti (2012).....................................................................................107

Guaranty
Guaranty (2009).............................................................................................................................................108

Surety
Surety (2010)...................................................................................................................................................108

Pledge
Pledge; Pactum Commissorium (2009)...........................................................................................109

Torts and Damages


Damages (2012)............................................................................................................................................109

Damages; Moral & Exemplary (2009)...............................................................................................110

Damages; Public Officers acting in the Performance of their Duties (2012)..........111

Death Indemnity (2009)............................................................................................................................ 112

Doctrine of Discovered Peril (Last Clear Chance) (2007).....................................................112

Liability; Owner of a Pet; Fortuitous Event (2010)..................................................................113

Liability; Special Parental Authority (2010)................................................................................115

Quasi-Delict; Claims; Requisites (2013)..........................................................................................115

Quasi Tort (2010)...........................................................................................................................................116

“Never Let The Odds Keep You From Pursuing What You Know In Your Heart You Were Meant To Do.”-Leroy Satchel Paige
Page 10 of 180
Civil Law Q&As (2007-2013) hectorchristopher@yahoo.com dbaratbateladot@gmail.com

MULTIPLE CHOICE QUESTIONS

2013 Civil Law Exam MCQ (October 13, 2013)...….


………………………………………..........118

2012 Civil Law Exam MCQ (October 14, 2012).….


……………………………………...............130

2011 Civil Law Exam MCQ (November 13, 2011).………………………………….


……............149

2010 Civil Law Exam MCQ (September 12, 2010).………………………………….


……...........176

2007 Civil Law Exam MCQ (September 09, 2007).………………………………….


……...........179
“Never Let The Odds Keep You From Pursuing What You Know In Your Heart You Were Meant To Do.”-Leroy Satchel Paige
Page 11 of 180
Civil Law Q&As (2007-2013) hectorchristopher@yahoo.com dbaratbateladot@gmail.com

Persons mother's womb. However, if the fetus


had an intra-uterine life of less than

Capacity: Juridical Capacity (2008) seven months, it is not deemed born


if it dies within twenty-four (24)
No. II. At age 18, Marian found out that hours after its complete delivery from
she was pregnant. She insured her own the maternal womb. The act of
life and named her unborn child as her naming the unborn child as sole
sole beneficiary. When she was already beneficiary in the insurance is
due to give birth, she and her boyfriend favorable to the conceived child and
Pietro, the father of her unboarn child, therefore the fetus acquires
were kidnapped in a resort in Bataan presumptive or provisional
where they were vacationing. The military personality. However, said
gave chase and after one week, they were presumptive personality only
found in an abandoned hut in Cavite. becomes conclusive if the child is
Marian and Pietro were hacked with bolos. born alive. The child need not survive
Marian and the baby delivered were both for twenty-four
found dead, with the baby's umbilical cord
already cut. Pietro survived.

(A). Can Marian's baby be the beneficiary


of the insurance taken on the life of the
mother? (2%)

SUGGESTED ANSWER:

Yes, the baby can be the beneficiary


of the life insurance of Marian. Art.
40 NCC
provides that "birth determines
personality; but the conceived child
shall be considered born for all
purposes that are favorable to it,
provided that it be born later with the
conditions specified in Art. 41. Article
41 states that "for civil purposes, the
fetus shall be considered born if it is
alive at the time it is completely
delivered from the
࿿࿿࿿࿿࿿࿿࿿࿿࿿'歇࿿࿿࿿࿿࿿࿿࿿࿿࿿(䨄岴࿿࿿࿿࿿࿿࿿࿿࿿࿿already due to give birth," indicating
that the child was more than seven
࿿࿿࿿࿿࿿࿿*巬࿿࿿࿿࿿࿿࿿࿿࿿࿿࿿+께㿏࿿࿿࿿࿿࿿࿿࿿࿿࿿ months old.

࿿࿿࿿࿿࿿࿿-찲ᆱ࿿࿿࿿࿿࿿࿿࿿࿿࿿.࿿࿿࿿࿿࿿࿿࿿࿿࿿࿿࿿

࿿࿿࿿࿿࿿0塐睈࿿࿿࿿࿿࿿࿿࿿࿿࿿1㦢瑉࿿࿿࿿࿿࿿࿿࿿278
Capacity; Juridical Capacity of Donee;

࿿࿿࿿࿿࿿࿿9�䷯࿿࿿࿿࿿࿿࿿࿿࿿࿿:헲℣࿿࿿࿿࿿࿿࿿࿿࿿࿿; Requisites for Acceptance (2012)

࿿࿿࿿࿿࿿࿿<➨ॉॉ࿿࿿࿿࿿࿿࿿࿿࿿࿿=࿿࿿࿿࿿࿿࿿࿿࿿࿿࿿ No.I. b) Ricky donated P 1 Million to the


unborn child of his pregnant girlfriend,
࿿࿿࿿࿿࿿࿿?掔⺊࿿࿿࿿࿿࿿࿿࿿࿿࿿@Ѐⓦ࿿࿿࿿࿿࿿࿿࿿࿿࿿
which she accepted. After six (6) months

࿿࿿࿿࿿࿿࿿B솴࿿࿿࿿࿿࿿࿿࿿࿿࿿࿿C겼㚲࿿࿿࿿࿿࿿࿿࿿࿿࿿ of pregnancy, the fetus was born and


baptized as Angela. However, Angela died
࿿࿿࿿࿿࿿࿿E湸䪲࿿࿿࿿࿿࿿࿿࿿࿿࿿F℣℣࿿࿿࿿࿿࿿࿿࿿࿿࿿ 20 hours after birth. Ricky sought to
recover the P 1 Million. Is Ricky entitled to
࿿࿿࿿࿿࿿࿿Hⓦ憁࿿࿿࿿࿿࿿࿿࿿࿿࿿Iæ㹤࿿࿿࿿࿿࿿࿿࿿࿿࿿
recover? Explain. (5%)
࿿࿿࿿࿿࿿K埐焇࿿࿿࿿࿿࿿࿿࿿࿿࿿L촚倕࿿࿿࿿࿿࿿࿿࿿࿿࿿
SUGGESTED ANSWER:
࿿࿿࿿࿿࿿࿿࿿N鹨℣࿿࿿࿿࿿࿿࿿࿿࿿࿿Oⓦ䝞࿿࿿࿿࿿࿿࿿࿿࿿࿿
Yes, Ricky is entitled to recover the
࿿࿿࿿࿿࿿࿿Q煌樻࿿࿿࿿࿿࿿࿿࿿࿿࿿R兾㉻࿿࿿࿿࿿࿿࿿࿿࿿࿿ P1,000,000.00. The NCC considers a

࿿࿿࿿࿿࿿࿿T켤℣࿿࿿࿿࿿࿿࿿࿿࿿࿿Uⓦ嵛࿿࿿࿿࿿࿿࿿࿿࿿࿿

࿿࿿࿿࿿࿿࿿W掄縏࿿࿿࿿࿿࿿࿿࿿࿿࿿X譒℣࿿࿿࿿࿿࿿࿿࿿࿿࿿

࿿࿿࿿࿿࿿࿿Zⓦᅪ࿿࿿࿿࿿࿿࿿࿿࿿࿿[닺㒔࿿࿿࿿࿿࿿࿿࿿࿿࿿

࿿࿿࿿࿿࿿࿿]閘摂࿿࿿࿿࿿࿿࿿࿿࿿࿿^ᚐ挟࿿࿿࿿࿿࿿࿿࿿࿿࿿

࿿࿿࿿࿿࿿`潖࿿࿿࿿࿿࿿࿿࿿࿿࿿࿿a�滉࿿࿿࿿࿿࿿࿿࿿࿿࿿

࿿࿿࿿࿿࿿࿿c࿿࿿࿿࿿࿿࿿࿿࿿࿿࿿d䚼㥼࿿࿿࿿࿿࿿࿿࿿࿿࿿

࿿࿿࿿࿿࿿࿿f쏤媜࿿࿿࿿࿿࿿࿿࿿࿿࿿g㘤℣࿿࿿࿿࿿࿿࿿࿿࿿࿿

࿿࿿࿿࿿࿿࿿i䈈溾࿿࿿࿿࿿࿿࿿࿿࿿࿿j跴࿿࿿࿿࿿࿿࿿࿿࿿࿿࿿

࿿࿿࿿࿿࿿࿿࿿l믢℣࿿࿿࿿࿿࿿࿿࿿࿿࿿mⓦ⚠࿿࿿࿿࿿࿿࿿࿿࿿࿿

࿿࿿࿿࿿࿿࿿o潠析࿿࿿࿿࿿࿿࿿࿿࿿࿿p鶪მ࿿࿿࿿࿿࿿࿿࿿࿿࿿

࿿࿿࿿࿿࿿࿿r課࿿ॉ࿿࿿࿿࿿࿿࿿࿿࿿࿿s泐㪖࿿࿿࿿࿿࿿࿿࿿࿿࿿

࿿࿿࿿࿿࿿࿿u墕࿿࿿࿿࿿࿿࿿࿿࿿࿿v큔㢥࿿࿿࿿࿿࿿࿿࿿࿿࿿

࿿࿿࿿࿿࿿࿿x㜴䭹࿿࿿࿿࿿࿿࿿࿿࿿࿿y듨䨐࿿࿿࿿࿿࿿࿿࿿࿿࿿

࿿࿿࿿࿿࿿࿿{煦䰪࿿࿿࿿࿿࿿࿿࿿࿿࿿|䪂℣࿿࿿࿿࿿࿿࿿࿿࿿࿿
“Never Let The Odds Keep You From Pursuing What You Know In Your Heart You Were Meant To Do.”-Leroy Satchel Paige
Page 12 of 180
Civil Law Q&As (2007-2013) hectorchristopher@yahoo.com dbaratbateladot@gmail.com

fetus a person for purposes favorable acquire a house in Australia because


to it provided it is born later in Australian Laws allow aliens to acquire
accordance with the provision of the property from the age of 16.
NCC. While the donation is favorable
to the fetus, the donation did not SUGGESTED ANSWER:

take effect because the fetus was not


TRUE. Since Australian Law allows
born in accordance with the NCC.
alien to acquire property from the

To be considered born, the fetus that age of 16, Roberta may validly own a

had an intrauterine life of less than house in Australia, following the

seven (7) months should live for 24 principle of lex rei sitae enshrined in

hours from its complete delivery from Art. 16, NCC, which states "Real

the mother’s womb. Since Angela had property as well as personal property

an intrauterine life of less than seven is subject to the law of the country

(7) months but did not live for 24 where it is situated." Moreover, even

hours, she was not considered born assuming that legal capacity of

and, therefore, did not become a Roberta in entering the contract in

person. Not being a person, she has Australia is governed by Philippine

no juridical capacity to be a donee, Law, she will acquire ownership over

hence, the donation to her did not the property bought until the

take effect. The donation not being contract is annulled.

effective, the amount donated may be


ALTERNATIVE ANSWER:
recovered. To retain it will be unjust
enrichment.
FALSE. Laws relating to family rights
and duties, or to the status, condition
or legal capacity of persons are

Capacity: Legal Capacity; Lex Rei binding upon the citizens of the

Sitae (2007) Philippines, even though living


abroad (Art. 15, NCC). The age of
No.VII. Write "TRUE" if the statement is majority under Philippine law is 18
true or "FALSE" if the statement is false. If years (R.A. No. 6809); hence,
the statement is FALSE, state the reason. Roberta, being only 17 years old, has
(2% each). no legal capacity to acquire and own
land.
(1). Roberta, a Filipino, 17 years of age,
without the knowledge of his parents, can

“Never Let The Odds Keep You From Pursuing What You Know In Your Heart You Were Meant To Do.”-Leroy Satchel Paige
Page 13 of 180
Civil Law Q&As (2007-2013) hectorchristopher@yahoo.com dbaratbateladot@gmail.com

Correction of Entries; Clerical Error the Rules of Court because said


Act (2008) changes are substantive corrections.

No. IV. Gianna was born to Andy and (B). Instead of a judicial action, can
Aimee, who at the time Gianna's birth administrative proceedings be brought for
were not married to each other. While the purpose of making the above
Andy was single at the time, Aimee was corrections? (2%)
still in the process of securing a judicial
declaration of nullity on her marriage to SUGGESTED ANSWER:

her ex-husband. Gianna's birth certificate,


No. An administrative proceeding
which was signed by both Andy and
cannot be brought for the purpose of
Aimee, registered the status of Gianna as
making the above corrections. R.A.
"legitimate", her surname carrying that of
9048, otherwise known as the Clerical
Andy's and that her parents were married
Error Act, which authorizes the city or
to each other.
municipal civil registrar or the consul

(A). Can a judicial action for correction of general to correct a clerical or

entries in Gianna's birth certificate be typographical error in an entry and/or

successfully maintained to: change the first name or nickname in


the civil register without need of a
a). Change her status from "legitimate" to judicial order. Errors that involve the
"illegitimate" (1%); change of nationality, age, status,
surname or sex of petitioner are not
and included from the coverage of the
said Act (Silverio v. Republic, G.R. No.
b). Change her surname from that of
174689, 22 Oct., 2007).
Andy's to Aimee's maiden surname? (1%)

SUGGESTED ANSWER:
Nationality Principle (2009)

Yes, a judicial action for correction of


No.XII. Emmanuel and Margarita,
entries in Gianna's birth certificate
American citizens and employees of the
can be successfully maintained to
U.S. State Department, got married in the
change (a) her status from
African state of Kenya where sterility is a
"legitimate" to "illegitimate," and (b)
ground for annulment of marriage.
her surname from that of Andy's to
Thereafter, the spouses were assigned to
Aimee's maiden surname in
the U.S. Embassy in Manila. On the first
accordance with Rule 108 of
year of the spouses’

“Never Let The Odds Keep You From Pursuing What You Know In Your Heart You Were Meant To Do.”-Leroy Satchel Paige
Page 14 of 180
Civil Law Q&As (2007-2013) hectorchristopher@yahoo.com dbaratbateladot@gmail.com

tour of duty in the Philippines, Margarita


filed an annulment case against ALTERNATIVE ANSWER:
Emmanuel before a Philippine court on the The forum has jurisdiction over an
ground of her husband’s sterility at the action for the annulment of marriage
time of the celebration of the marriage. solemnized elsewhere but only when
the party bringing the actions is
(A). Will the suit prosper? Explain your domiciled in the forum. In this case,
answer. (3%) none of the parties to the marriage is
domiciled in the Philippines. They are
SUGGESTED ANSWER:
here as officials of the US Embassy
No, the suits will not prosper. As
whose stay in the country is merely
applied to foreign nationals with the
temporary, lasting only during their
respect to family relations and status
fixed tour of duty. Hence, the
of persons, the nationality principle
Philippine courts have no jurisdiction
set forth in Article 15 of the Civil
over the action.
Code will govern the relations of
Emmanuel and Margarita. Since they
are American citizens, the governing
Nationality Principle; Change of Name
law as to the ground for annulment is
not Covered (2009)
not Kenyan Law which Magarita
invokes in support of sterility as such No.XX. (A). If Ligaya, a Filipino citizen
ground; but should be U.S. Law, residing in the United States, files a
which is the national Law of both petition for change of name before the
Emmanuel and Margarita as District Court of New York, what law shall
recognized under Philippine Law. apply? Explain. (2%)
Hence, the Philippine court will not
give due course to the case based on SUGGESTED ANSWER:
Kenyan Law. The nationality principle New York law shall apply. The petition
as expressed in the application of of change of name file din New York
national law of foreign nationals by does not concern the legal capacity
Philippine courts is established by or status of the petitioner. Moreover,
precedents (Pilapil v. Ibay-Somera, it does nto affect the registry of any
174 SCRA 653[1989], Garcia v. Recio, other country including the country
366 SCRA 437 [2001], Llorente v. of birth of the petitioner. Whatever
Court of Appeals 345 SCRA 92 [2000], judgment is rendered in that petition
and Bayot v. Court of Appeals 570 will have effect only in New York. The
SCRA 472 [2008]). New York court

“Never Let The Odds Keep You From Pursuing What You Know In Your Heart You Were Meant To Do.”-Leroy Satchel Paige
Page 15 of 180
Civil Law Q&As (2007-2013) hectorchristopher@yahoo.com dbaratbateladot@gmail.com

cannot, for instance, order the Civil petitioner and his transactions in the
Registrar in the Philippines to change Philippines. The Philippine court can
its records. The judgment of the New never acquire jurisdiction over the
York court allowing a change in the custodian in the US of the records of
name of the petitioner will be limited the petitioner. Moreover, change of
to the records of the petitioner in name has nothing to do with the legal
New York and the use of her new capacity or status of the alien. Since
name in all transactions in New York. Philippine records and transactions
Since the records and processes in are the only ones affected, the
New York are the only ones affected, Philippine court may effect the
the New York court will apply New change only in accordance with the
YorK law in resolving the petition. laws governing those records and
ALTERNATIVE ANSWER: transactions that law cannot be but
Philippine law shall apply (Art 15, Philippine law.
NCC). Status, conditions, family ALTERNATIVE ANSWER:
rights and duties are governed by U.S. law shall apply as it is his
Philippine laws as to Filipinos even national law. This is pursuant to the
though sojourning abroad. application of lex patriae or the
ALTENATIVE ANSWER: nationality principle, by which his
If Ligaya, a Filipino, files a petition for legal status is governed by national
change of name with the District law, the matter of change of name
Court of New YoRk, the laws of New being included in the legal status.
York will govern since change of The Supreme Court has reiterate in
name is not one of those covered by several cases, that the lex patriae as
the principles of nationality. provided in Article 15 of the Civil
Code is applicable to foreign
(B). If Henry, an American citizen residing nationals in determining their legal
in the Philippines, files a petition for status (supra).
change of name before a Philippine court,
what law shall apply? Explain. (2%)
Conflict of Laws
SUGGESTED ANSWER:
Processual Presumption (2009)
Philippine law will apply. The petition
for change of name in the Philippines
No.I. TRUE or FALSE. Answer TRUE if the
will affect only the records of the
statement is true, or FALSE if the
statement is false. Explain your answer in
not more than two (2) sentences.

“Never Let The Odds Keep You From Pursuing What You Know In Your Heart You Were Meant To Do.”-Leroy Satchel Paige
Page 16 of 180
Civil Law Q&As (2007-2013) hectorchristopher@yahoo.com dbaratbateladot@gmail.com

(A). The doctrine of "processual 23 Public Order. To maintain peace


presumption" allows the court of the and order, disputes that disturb the
forum to presume that the foreign law peace of the forum should be settled
applicable to the case is the same as the by the court of the forum even
local or domestic law. (1%) though the application of the foreign
law is necessary for the purpose.
SUGGESTED ANSWER:
TRUE. If the foreign law necessary to 24 Humanitarian Principle. An

the resolve an issue is not proven as aggrieved party should not be left

a fact, the court of the forum may without remedy in a forum even

presume that the foreign law is the though the application of the foreign

same as the law of the forum. law by the courts of the forum is
unavoidable in order to extend relief.

Adoption
Jurisdiction; Courts may Assume
Jurisdiction over Conflict of Laws
Adoption; Termination; Death of
Cases (2010)
Adopter (2009)

No.III. Define, Enumerate or Explain.


No.XIII. Rafael, a wealthy bachelor, filed a
(2% each)
petition for the adoption of Dolly, a one-year
old foundling who had a severe heart
0 Give at least two reasons why a court
ailment. During the pendency of the
may assume jurisdiction over a conflict of
adoption proceedings, Rafael died of natural
laws case.
causes. The Office of the Solicitor General

SUGGESTED ANSWER: files a motion to dismiss the petition on the


ground that the case can no longer proceed
0 Statute theory. There is a domestic because of the petitioner’s death.
law authorizing the local court to
assume jurisdiction. (A). Should the case be dismissed?
Explain. (2%)
1 Comity theory. The local court
assumes jurisdiction based on the SUGGESTED ANSWER:
principle of comity or courtesy.
It depends on the stage of the
proceedings when Rafael died. If he
ALTERNATIVE ANSWER:
died after all the requirements under
the law

“Never Let The Odds Keep You From Pursuing What You Know In Your Heart You Were Meant To Do.”-Leroy Satchel Paige
Page 17 of 180
Civil Law Q&As (2007-2013) hectorchristopher@yahoo.com dbaratbateladot@gmail.com

have been complied with and the Adoption; Illegitimate Child


(2010)
case is already submitted for
resolution, the court may grant the
No.VIII. Spouses Rex and Lea bore two
petition and issue a decree of
children now aged 14 and 8. During the
adoption despite the death of the
subsistence of their marriage, Rex begot a
adopter (Section 13, RA 8552).
child by another woman. He is now 10
Otherwise, the death of the petitioner
years of age.
shall have the effect terminating the
proceedings.
On Lea’s discovery of Rex’s fathering a
child by another woman, she filed a
(B). Will your answer be the same if it was
petition for legal separation which was
Dolly who died during the pendency of the
granted.
adoption proceedings? Explain. (2%)

Rex now wants to adopt his illegitimate


SUGGESTED ANSWER:
child.
No, if it was Dolly who died, the case
should be dismissed. Her death
0 Whose consent is needed for Rex’s
terminates the proceedings (Art. 13, adoption of his illegitimate child? (2.5%)
Domestic Adoption Law).
SUGGESTED ANSWER:
ALTERNATIVE ANSWER:
The consent of the 14-year-old
It depends. If all the requirements
legitimate child, of the 10- year -old
under the law have already been
illegitimate child and of the biological
complied with and the case is already
mother of the illegitimate child are
submitted for resolution, the death of
needed for the adoption (Section 7
the adoptee should not abate the
and 9, RA 8552). The consent of Lea
proceedings. The court should issue
is no longer required because there
the decree of adoption if will be for
was already a final decree of legal
the best interest of the adoptee.
separation.
While RA8552 provides only for the
case where it is the petitioner who
0 If there was no legal separation, can Rex
dies before the decree is issued, it is
still adopt his illegitimate child? Explain.
with more compelling reason that the
(2.5%)
decree should be allowed in case it is
the adoptee who dies because SUGGESTED ANSWER:
adoption is primarily for his benefit.
Yes, he can still adopt his illegitimate
child but with the consent of his
spouse,
“Never Let The Odds Keep You From Pursuing What You Know In Your Heart You Were Meant To Do.”-Leroy Satchel Paige
Page 18 of 180
Civil Law Q&As (2007-2013) hectorchristopher@yahoo.com dbaratbateladot@gmail.com

of his 14-year-old legitimate child, of Adoption of Stephanie Nathy Astorga


the illegitimate child, and of the Garcia, G.R. No. 148311, March 31,
biological mother of the illegitimate 2005, the Supreme Court ruled that
child (Section 7 and 9, RA 8552). the adopted child may use the
surname of the natural mother as his
middle name because there is no
prohibition in the law against it.
Adoption; Illegitimate Child; Use of
Moreover, it will also be for the
Mother’s Surname as Middle Name
benefit of the adopted child who shall
(2012)
preserve his lineage on his mother’s
side and reinforce his right to inherit
No.IV.b) Honorato filed a petition to adopt
from his mother and her family.
his minor illegitimate child Stephanie,
Lastly, it will make the adopted child
alleging that Stephanie’s mother is
conform with the time-honored
Gemma Astorga Garcia; that Stephanie
Filipino tradition of carrying the
has been using her mother’s middle name
mother’s surname as the person’s
and surname; and that he is now a
middle name.
widower and qualified to be her adopting
parent. He prayed that Stephanie’s middle
name be changed from "Astorga" to
"Garcia," which is her mother’s surname Consent; Consent of the Adopter’s
and that her surname "Garcia" be changed Heirs (2008)
to "Catindig," which is his surname. This
the trial court denied. Was the trial court No.V. Despite several relationships with
correct in denying Hororato’s request for different women, Andrew remained
Stephanie’s use of her mother’s surname unmarried. His first relationship with
as her middle name? Explain. (5%) Brenda produced a daughter, Amy, now 30
years old. His second, with Carla,
SUGGESTED ANSWER: produced two sons: Jon and Ryan. His
third, with Donna, bore him no children
No, the trial court was not correct.
although Elena has a daughter Jane, from
There is no law prohibiting an
a previous relationship. His last, with Fe,
illegitimate child adopted by his natural
produced no biological children but they
father to use as middle name his
informally adopted without court
mother’s surname. The law is silent as
proceedings, Sandy's now 13 years old,
to what middle name an adoptee may
whom they consider as their own. Sandy
use. In case of In re:
was orphaned as a baby and was
entrusted to them by the midwife who
attended to Sandy's birth. All the
“Never Let The Odds Keep You From Pursuing What You Know In Your Heart You Were Meant To Do.”-Leroy Satchel Paige
Page 19 of 180
Civil Law Q&As (2007-2013) hectorchristopher@yahoo.com dbaratbateladot@gmail.com

children, including Amy, now live with Qualifications of Adopter (2010)


andrew in his house.
No.IX. Eighteen-year old Filipina Patrice
(A). Is there any legal obstacle to the legal had a daughter out of wedlock whom she
adoption of Amy by Andrew? To the legal named Laurie. At 26, Patrice married
adoption of Sandy by Andrew and Elena? American citizen John who brought her to
(2%) live with him in the United States of
America. John at once signified his
SUGGESTED ANSWER: willingness to adopt Laurie.

Yes, there is a legal obstacle to the Can John file the petition for adoption? If
legal adoption of Amy by Andrew. yes, what are the requirements? If no,
Under Sec. 9(d) of RA 8552, the New why? (5%)
Domestic Adoption Act of 1998, the
written SUGGESTED ANSWER:
consent of the illegitimate
No, John cannot file the petition to
sons/daughters, ten (10) years of age
adopt alone. Philippine law requires
or over, of the adopter, if living with
husband and wife to adopt jointly
said adopter and the latter's spouse,
except on certain situations
if any, is necessary to the adoption.
enumerated in the law. The case of
All the children of Andrew are living
John does not fall in any of the
with him. Andrew needs to get the
exceptions (R.A. 8552).
written consent of Jon, Ryan, Vina
and Wilma, who are all ten (10) years Family Code
old or more. Sandy's consent to
Amy's adoption is not necessary Marriage; Annulment; Grounds (2009)
because she was not legally adopted
by Andrew. Jane's consent is likewise No.XII. Emmanuel and Margarita,
not necessary because she is not a American citizens and employees of the
child of Andrew. Sandy, an orphan U.S. State Department, got married in the
since birth, is eligible for adoption African state of Kenya where sterility is a
under Sec. 8(f) of RA 8552, provided ground for annulment of marriage.
that Andrew obtains the written Thereafter, the spouses were assigned to
consent of the other children the U.S. Embassy in Manila. On the first
mentioned above, including Amy and year of the spouses’ tour of duty in the
Elena obtains the written consent of Philippines, Margarita filed an annulment
Jane, if she is over ten years old (Sec. case against Emmanuel before a
9(d), RA 8552). Philippine court on the ground of

“Never Let The Odds Keep You From Pursuing What You Know In Your Heart You Were Meant To Do.”-Leroy Satchel Paige
Page 20 of 180
Civil Law Q&As (2007-2013) hectorchristopher@yahoo.com dbaratbateladot@gmail.com

her husband’s sterility at the time of the shall be determined by applying


celebration of the marriage. Kenyan law and not Philippine law.

(B). Assume Emmanuel and Margarita are However, while Kenyan law governs
both Filipinos. After their wedding in the formal validity of the marriage,
Kenya, they come back and take up the legal capacity of the Filipino
residence in the Philippines. Can their parties to the marriage is governed
marriage be annulled on the ground of not by Kenyan law but by Philippine
Emmanuel’s sterility? Explain. (3%) law (Article 15, NCC). Sterility of a
party as a ground for the annulment
SUGGESTED ANSWER:
of the marriage is not a matter of
No, the marriage cannot be annulled
form but a matter of legal capacity.
under the Philippine law. Sterility is
Hence, the Philippine court must
not a ground for annulment of
apply Phillippine law in determining
marriage under Article 45 of the
the status of the marriage on the
Family Code.
ground of absence or defect in the
legal capacity of the Filipino parties.
ALTERNATIVE ANSWER:
Since sterility does not constitute
No, the marriage cannot be annulled
absence or defect in the legal
in the Philippines.
capacity of the parties under
Philippine law, there is no ground to
The Philippine court shall have
avoid or annul the marriage. Hence,
jurisdiction over the action to annul
the Philippine court has to deny the
the marriage not only because the
petition.
parties are residents of the
Philippines but because they are
Filipino citizens. The Philippine court,
however, shall apply the law of the
Marriage; Annulment; Grounds (2007)
place where the marriage was
celebrated in determining its formal No. VII. Write "TRUE" if the statement is
validity (Article 26, FC; Article 17, true or "FALSE" if the statement is false. If
NCC). the statement is FALSE, state the reason.
(2% each).
Since the marriage was celebrated in
Kenya in accordance with Kenyan law, (4). The day after John and Marsha got
the formal validity of such marriage married, John told her that he was
is governed by Kenyan law and any impotent. Marsha continued to live with
issue as to the formal validity of that
marriage
“Never Let The Odds Keep You From Pursuing What You Know In Your Heart You Were Meant To Do.”-Leroy Satchel Paige
Page 21 of 180
Civil Law Q&As (2007-2013) hectorchristopher@yahoo.com dbaratbateladot@gmail.com

John for 2 years. Marsha is now estopped No, I do not agree. There are others
from filing an annulment case against who may file a petition for
John. declaration of nullity such as the
other spouse in bigamous marriages.
SUGGESTED ANSWER:

FALSE. Marsha is not estopped from


filing an annulment case against John Marriage; Annulment; Support
on the ground of his impotence, Pendente Lite (2010)
because she learned of his impotence
after the celebration of the marriage No.V. G filed on July 8, 2000 a petition for

and not declaration of nullity of her marriage to B.

before. Physical incapacity to During the pendency of the case, the

consummate is a valid ground for the couple entered into a compromise

annulment of marriage if such agreement to dissolve their absolute

incapacity was existing at the time of community of property. B ceded his right

the marriage, continues and appears to their house and lot and all his shares in

to be incurable. The marriage may be two business firms to G and their two

annulled on this ground within five children, aged 18 and 19.

years from its celebration.


B also opened a bank account in the
amount of P3 million in the name of the
two children to answer for their

Marriage; Annulment; Parties (2012) educational expenses until they finish


their college degrees.
No.IX.b) A petition for declaration of nullity
of a void marriage can only be filed by For her part, G undertook to shoulder the

either the husband or the wife? Do you day-to-day living expenses and upkeep of

agree? Explain your answer. (5%) the children. The Court approved the
spouses’ agreement on September 8,
SUGGESTED ANSWER: 2000.

Yes, I agree. Under the rules 23 Suppose the business firms suffered
promulgated by the Supreme Court, a reverses, rendering G unable to support
direct action for declaration of nullity herself and the children. Can G still ask for
may only be filed by any of the support pendente lite from B? Explain.
spouses. (3%)

ALTERNATIVE SUGGESTED ANSWER:

“Never Let The Odds Keep You From Pursuing What You Know In Your Heart You Were Meant To Do.”-Leroy Satchel Paige
Page 22 of 180
Civil Law Q&As (2007-2013) hectorchristopher@yahoo.com dbaratbateladot@gmail.com

SUGGESTED ANSWER: Marriage; Divorce Decrees; Filipino


Spouse Becoming Alien (2009)
If B acquiesces and does not file the
action to impugn the legitimacy of No.IV. Harry married Wilma, a very wealthy
the child within the prescriptive woman. Barely five (5) years into the
period for doing so in Article 170 of marriage, Wilma fell in love with Joseph.
the Family Code, G's daughter by Thus, Wilma went to a small country in
another man shall be conclusively Europe, became a naturalized citizen of
presumed as the legitimate daughter that country, divorced Harry, and married
of B by G. Joseph. A year thereafter, Wilma and
Joseph returned and established
0 Suppose in late 2004 the two children
permanent residence in the Philippines.
had squandered the P3 million fund for
their education before they could obtain (A). Is the divorce obtained by Wilma from
their college degrees, can they ask for Harry recognized in the Philippines?
more support from B? Explain. (3%) Explain your answer. (3%)

SUGGESTED ANSWER: SUGGESTED ANSRWER :

Yes, the two children can still ask for


As to Wilma, the divorced obtained
support for schooling or training for
by her is recognized as valid in the
some professions, trade or vocation,
Philippines because she is now a
even beyond the age of majority until
foreigner. Philippine personal laws do
they shall have finished or completed
not apply to a foreigner. However,
their education (Article 194,
recognition of the divorce as regards
Paragraph 2, Family Code; Javier v.
Harry will depend on the applicability
Lucero, 94
to his case of the second paragraph
Phil. 634 {1954}].Their having
of Article 26 of the Family Code. If it
squandered the money given to them
is applicable, divorce is recognized as
for their education will not deprive
to him and, therefore, he can
them of their right to complete an
remarry. However, if it is not
education, or to extinguish the
applicable, divorce is not recognized
obligation of the parents to ensure
as to him and, consequently, he
the future of their children.
cannot remarry.

ALTERNATIVE ANSWER:

“Never Let The Odds Keep You From Pursuing What You Know In Your Heart You Were Meant To Do.”-Leroy Satchel Paige
Page 23 of 180
Civil Law Q&As (2007-2013) hectorchristopher@yahoo.com dbaratbateladot@gmail.com

Yes , the divorce obtained by Wilma is SUGGESTED ANSWER :


recognized as valid in the Philippines.
At the time she got the divorce, she Yes, he can validly marry Elizabeth,
was already a foreign national having applying the doctrine laid down by
been naturalized as a citizen of that the Supreme Court in Republic v.
“small country in Europe.” Based on Obrecido (427 SCRA 114 [2005]).
precedents established by the Under the second paragraph of
Supreme Court ( Bayot v. CA, 570 Article 26 of the Family Code, for the
SCRA 472 [2008]), divorce obtained Filipino spouse to have capacity to
by a foreigner is recognized in the remarry, the law expressly requires
Philippines if validly obtained in the spouse who obtained the divorce
accordance with his or her national to be a foreigner at the time of the
law . marriage. Applying this requirement
to the case of Harry it would seem
(B). If Harry hires you as his lawyer, what that he is not given the capacity to
legal recourse would you advise him to remarry. This is because Wilma was a
take? Why? (2%) Filipino at the time of her marriage to
Harry.
SUGGESTED ANSWER:

In Republic v. Obrecido, however, the


I will advice Harry to:
Supreme Court ruled that a Filipino
spouse is given the capacity to
0 Dissolve and liquidate his
remarry even though the spouse who
property relations with Wilma ; and
obtained the divorce was a Filipino at
the time of the marriage, if the latter
1 If he will remarry, file a petition
was already a foreigner when the
for the recognition and enforcement
divorce was already obtained abroad.
of the foreign judgment of divorced
According to the court, to rule
(Rule 39,Rules of Court ).
otherwise will violate the equal
protection clause of the Constitution.
(C). Harry tells you that he has fallen in
love with another woman, Elizabeth, and
wants to marry her because, after all,
Marriage; Divorce Decrees; Foreign
Wilma is already married to Joseph. Can
Spouse Divorces Filipino Spouse
Harry legally marry Elizabeth? Explain.
(2012)
(2%)

23 Cipriano and Lady Miros married each


other. Lady Miros then left for the US and
there, she obtained American citizenship.
“Never Let The Odds Keep You From Pursuing What You Know In Your Heart You Were Meant To Do.”-Leroy Satchel Paige
Page 24 of 180
Civil Law Q&As (2007-2013) hectorchristopher@yahoo.com dbaratbateladot@gmail.com

Cipriano later learned all about this proving only that the foreign spouse has
including the fact that Lady Miros has obtained a divorce against her or him
divorced him in America and that she had abroad. (1%)
remarried there. He then filed a petition
for authority to remarry, invoking Par. 2, SUGGESTED ANSWER :

Art. 26 of the Family Code. Is Cipriano


FALSE, In Garcia v. Recio , 366 SCRA
capacitated to re-marry by virtue of the
437 (2001) , the SC held that for a
divorce decree obtained by his Filipino
Filipino spouse to have capacity to
spouse who was later naturalized as an
contract a subsequent marriage, it
American citizen? Explain. (5%)
must also be proven that the foreign
divorced obtained abroad by the
SUGGESTED ANSWER:
foreigner spouse give such foreigner
Yes, he is capacitated to remarry. spouse capacity to remarry.
While the second paragraph of Art 26
ALTERNATIVE ANSWER:
of the Family Code is applicable only
to a Filipino who married a foreigner TRUE, Art 26 (2) (FC), clearly provides
at the time of marriage, the Supreme that the decree of divorce obtained
Court ruled in the case of Republic v. abroad by the foreigner spouse is
Orbecido, G.R. No. 154380, 5 Oct, sufficient to capacitate the Filipino
2005, that the said provision equally spouse to remarry.
applies to a Filipino who married
another Filipino at the time of the
marriage, but who was already a
foreigner when the divorce was Marriage; Legal Separation;
obtained. Prescription (2012)

No.IV.a) After they got married, Nikki


discovered that Christian was having an
Marriage; Divorce Decrees; Foreign affair with another woman. But Nikki
Spouse Divorces Filipino Spouse decided to give it a try and lived with him
(2010) for two (2) years. After two (2) years, Nikki
filed an action for legal separation on the
No.I. True or False.
ground of Christian’s sexual infidelity. Will
the action prosper? Explain. (5%)
(A). Under Article 26 of the Family Code,
when a foreign spouse divorces his/her
Filipino spouse, the latter may re-marry by

“Never Let The Odds Keep You From Pursuing What You Know In Your Heart You Were Meant To Do.”-Leroy Satchel Paige
Page 25 of 180
Civil Law Q&As (2007-2013) hectorchristopher@yahoo.com dbaratbateladot@gmail.com

SUGGESTED ANSWER: the sexual infidelity committed in


2003, the prescriptive period runs
Although the action for legal
from 2003 and so on. The action for
separation has not yet prescribed,
legal separation for the last act of
the prescriptive period being 5 years,
sexual infidelity in 2005 will prescribe
if Obecido’s affair with another
in 2010.
woman was ended when Nikki
decided to live with him again, Nikki’s
action will not prosper on account of
condonation. However, if such affair Marriage; Psychological Incapacity
is still continuing, Nikki’s action (2013)
would prosper because the action will
surely be within five (5) years from No.I. You are a Family Court judge and

the commission of the latest act of before you is a Petition for the Declaration

sexual infidelity. Every act of sexual of Nullity of Marriage (under Article 36 of

liaison is a ground for legal the Family Code)filed by Maria against

separation. Neil. Maria claims that Neil is


psychologically incapacitated to comply
with the essential obligations of marriage
because Neil is a drunkard, a womanizer, a
Marriage; Legal Separation;
gambler, and a mama's boy- traits that
Prescription (2007)
she never knew or saw when Neil was
courting her. Although summoned, Neil did
No.VII. Write "TRUE" if the statement is
not answer Maria's petition and never
true or "FALSE" if the statement is false. If
appeared in court.
the statement is FALSE, state the reason.
(2% each).
To support her petition, Maria presented
three witnesses- herself, Dr. Elsie Chan,
(2). If a man commits several acts of
and Ambrosia. Dr. Chan testified on the
sexual infidelity, particularly in 2002,
psychological report on Neil that she
2003, 2004, 2005, the prescriptive period
prepared. Since Neil never acknowledged
to file for legal separation runs from 2002.
n9r responded to her invitation for

SUGGESTED ANSWER: interviews, her report is solely based on


her interviews with Maria and the spouses'
FALSE. The five-year prescriptive minor children. Dr. Chan concluded that
period for filing legal separation runs Neil is suffering from Narcissistic
from the Personality Disorder, an ailment that she
occurrence of sexual infidelity found to be already present since Neil's
committed in 2002 runs from 2002,
for
“Never Let The Odds Keep You From Pursuing What You Know In Your Heart You Were Meant To Do.”-Leroy Satchel Paige
Page 26 of 180
Civil Law Q&As (2007-2013) hectorchristopher@yahoo.com dbaratbateladot@gmail.com

early adulthood and one that is grave and mere conclusions. Being a drunkard,
incurable. Maria testified on the specific a womanizer, a gambler and a
instances when she found Neil drunk, with mama’s boy, merely shows Neil’s
another woman, or squandering the failure to perform his marital
family's resources in a casino. Ambrosia, obligations. In a number of cases, the
the spouses' current household help, Supreme Court did not find the
corroborated Maria's testimony. existence of psychological incapacity
in cases where the respondent
On the basis of the evidence presented, showed habitual drunkenness
will you grant the petition? (8%) (Republic v. Melgar, G.R. No. 139676,
2006), blatant display of infidelity
SUGGESTED ANSWER:
and irresponsibility (Dedel v. CA,
2004) or being hooked to gambling
No. The petition should be denied.
and drugs (Republic v. Tanyag-San

The psychological incapacity under Jose, G.R. No. 168328, 2007).

Art. 36 of the Family Code must be


ALTERNATIVE ANSWER:
characterized by (a) gravity, (b)
juridical antecedence, and (c)
Yes. The petition should be granted.
incurability. It is not enough to prove
that the parties failed to meet their The personal medical or psychological
responsibilities and duties as married examination of respondent is not a
persons; it is essential that they must requirement for declaration of
be shown to be incapable of doing so, psychological incapacity. It is the
due to some physiological (not totality of the evidence presented
physical) illness (Republic v. CA and which shall determine the existence
Molina, G.R. No. 108763, Feb 13, of psychological incapacity (Marcos v.
1997). Marcos, G.R. No. 136490, Oct 19,
2000). Dr. Chan’s report corroborated
In this case, the pieces of evidence
by Maria’s and Ambrosia’s
presented are not sufficient to
testimonies, therefore, sufficiently
conclude that indeed Neil is suffering
prove Neil’s psychological incapacity
from psychological incapacity
to assume his marital obligations.
[Narcissistic Personality Disorder]
existing already before the marriage,
incurable and serious enough to
prevent Neil from performing his
essential marital obligations. Dr.
Chan’s report contains

“Never Let The Odds Keep You From Pursuing What You Know In Your Heart You Were Meant To Do.”-Leroy Satchel Paige
Page 27 of 180
Civil Law Q&As (2007-2013) hectorchristopher@yahoo.com dbaratbateladot@gmail.com

Marriage; Psychological Incapacity the said report is the only evidence of


(2012) respondent’s psychological
incapacity.
No.II.b) The petitioner filed a petition for
declaration of nullity of marriage based
allegedly on the psychological incapacity Marriage; Requisites (2008)
of the respondent, but the psychologist
was not able to personally examine the No. III. Roderick and Faye were high school

respondent and the psychological report sweethearts. When Roderick was 18 and

was based only on the narration of Faye, 16 years old, they started to live

petitioner. Should the annulment be together as husband and wife without the

granted? Explain. (5%) benefit of marriage. When Faye reached 18


years of age, her parents forcibly took her
SUGGESTED ANSWER: back and arranged for her marriage to Brad.
Although Faye lived with Brad after the
The annulment cannot be guaranteed
marriage, Roderick continued to regularly
solely on the basis of the
visit Faye while Brad was away at work.
psychological report. For the report
During their marriage, Faye gave birth to a
to prove the
baby girl, Laica. When Faye was 25 years
psychological incapacity of the
old, Brad discovered her continued liason
respondent, it is required that the
with Roderick and in one of their heated
psychologist should personally
arguments, Faye shot Brad to death. She lost
examine the respondent and the
no time in marrying her true love Roderick,
psychological report should be based
without a marriage license, claiming that
on the psychologist’s independent
they have been continuously cohabiting for
assessment of the facts as to
more than 5 years.
whether or not the
respondent is psychologically (A). Was the marriage of Roderick and
incapacitated. Faye valid? (2%)

Since, the psychologist did not SUGGESTED ANSWER:


personally examine the respondent,
and his report is based solely on the No. The marriage of Roderick and
story of the petitioner who has an Faye is not valid. Art. 4, FC provides
interest in the outcome of the that the absence of any of the
petition, the marriage cannot be essential or formal requisites renders
annulled on the ground of the marriage void ab initio. However,
respondent’s psychological incapacity no license shall be
if

“Never Let The Odds Keep You From Pursuing What You Know In Your Heart You Were Meant To Do.”-Leroy Satchel Paige
Page 28 of 180
Civil Law Q&As (2007-2013) hectorchristopher@yahoo.com dbaratbateladot@gmail.com

necessary for the marriage of a man reqiured to submit the required certificate
and a woman who have lived together of capacity to marry from the German
as husband and wife for at least 5 Embassy in Manila, Adolf stated in the
years and without any legal application for marriage license that he
impediment to marry each other. In was a Filipino citizen. With the marriage
Republic v. Dayot, G.R. No. 175581, license stating that Adolf was a Filipino,
28 March 2008, reiterating the the couple got married in a ceremony
doctrine in Niñal v. Bayadog, G.R. No. officiated by the Parish Priest of Calamba,
133778, 14 March 2000, this five-year Laguna in a beach in Nasugbu, Batangas,
period is characterized by exclusivity as the local parish priest refused to
and continuity. In the present case, solemnize marriages except in his church.
the marriage of Roderick and Faye Is the marriage valid? Explain fully. (5%)
cannot be considered as a marriage
of exceptional character, because SUGGESTED ANSWER:

there were 2 legal impediments


No. The marriage is not valid. Art. 41
during their cohabitation: minority on
FC allows the present spouse to
the part of Faye, during the first two
contract a
years of cohabitation; and, lack of
subsequent marriage during the
legal capacity, since Faye married
subsistence of his previous marriage
Brad at the age of 18. The absence of
provided that: (a) his prior spouse in
a marriage license made the marriage
the first marriage had been absent
of Faye and Roderick void ab initio.
for four consecutive years; (b) that
the spouse present has a well-

Marriage; Subsequent Marriage founded belief that the absent


(2008) spouse was already dead, and
0 present spouse instituted a
No. I. Ana Rivera had a husband, a Filipino summary proceeding for the
citizen like her, who was among the
declaration of the presumptive death
passengers on board a commercial jet of absent spouse. Otherwise, the
plane which crashed in the Atlantic Ocean
second marriage shall be null and
ten void. In the instant case, the husband
0 years earlier and had never been of Ana was among the passengers on
heard of ever since. Believing that her board a commercial jet plane which
husband had died, Ana married Adolf Cruz crashed in the Atlantic Ocean. The
Staedtler, a divorced German national body of the deceased husband was
born of a German father and a Filipino not recovered to confirm his death.
mother residing in Stuttgart. To avoid Thus, following Art. 41, Ana
being
“Never Let The Odds Keep You From Pursuing What You Know In Your Heart You Were Meant To Do.”-Leroy Satchel Paige
Page 29 of 180
Civil Law Q&As (2007-2013) hectorchristopher@yahoo.com dbaratbateladot@gmail.com

should have first secured a judicial while Jane is a child of Elena from a
declaration of his presumptive death previous relationship. Thus, their
before she married Adolf. The marriage is not one of the prohibited
absence of marriages enumerated under Art. 38
the said judicial declaration of the FC.
incapacitated Ana from contracting
her second marriage, making it void
ab initio.
Marriage; Void Marriages; By Reason
of Public Policy (2007)

Marriage; Void Marriages; By Reason No. VII. Write "TRUE" if the statement is

of Public Policy (2008) true or "FALSE" if the statement is false. If


the statement is FALSE, state the reason.
No.V. Despite several relationships with (2% each).
different women, Andrew remained
unmarried. His first relationship with (5). Amor gave birth to Thelma when she

Brenda produced a daughter, Amy, now 30 was 15 years old. Thereafter, Amor met

years old. His second, with Carla, David and they got married when she was

produced two sons: Jon and Ryan. His 20 years old. David had a son, Julian, with

third, with Donna, bore him no children his ex-girlfriend Sandra. Julian and Thelma

although Elena has a daughter Jane, from can get married.

a previous relationship. His last, with Fe,


SUGGESTED ANSWER:
produced no biological children but they
informally adopted without court
TRUE. Julian and Thelma can get
proceedings, Sandy's now 13 years old,
married. Marriage between
whom they consider as their own. Sandy
stepbrothers and stepsisters are not
was orphaned as a baby and was
among the marriages prohibited
entrusted to them by the midwife who
under the Family Code.
attended to Sandy's birth. All the children,
including Amy, now live with andrew in his
house.
Marriage; Void Marriages; Property
Relations (2009)
(D). Can Jon and Jane legally marry? (1%)

No. III. In December 2000, Michael and


SUGGESTED ANSWER:
Anna, after obtaining a valid marriage

Yes. Jon and Jane can marry each


other; Jon is an illegitimate child of
Andrew

“Never Let The Odds Keep You From Pursuing What You Know In Your Heart You Were Meant To Do.”-Leroy Satchel Paige
Page 30 of 180
Civil Law Q&As (2007-2013) hectorchristopher@yahoo.com dbaratbateladot@gmail.com

license, went to the Office of the Mayor of impediment for them to validity
Urbano, Bulacan, to get married. The Mayor marry each other.
was not there, but the Mayor’s secretary
asked Michael and Anna and their witnesses
to fill up and sign the required marriage Marriage; Void Marriages; Status of
contract forms. The secretary then told them Children (2009)
to wait, and went out to look for the Mayor
who was attending a wedding in a No. III. In December 2000, Michael and Anna,

neighboring municipality. after obtaining a valid marriage license,


went to the Office of the Mayor of Urbano,
When the secretary caught up with the Bulacan, to get married. The Mayor was not
Mayor at the wedding reception, she there, but the Mayor’s secretary asked
showed him the marriage contract forms Michael and Anna and their witnesses to fill
and told him that the couple and their up and sign the required marriage contract
witnesses were waiting in his office. The forms. The secretary then told them to wait,
Mayor forthwith signed all the copies of and went out to look for the Mayor who was
the marriage contract, gave them to the attending a wedding in a neighboring
secretary who returned to the Mayor’s municipality.
office. She then gave copies of the
marriage contract to the parties, and told
Michael and Anna that they were already When the secretary caught up with the
married. Thereafter, the couple lived Mayor at the wedding reception, she
together as husband and wife, and had showed him the marriage contract forms
three sons. and told him that the couple and their
witnesses were waiting in his office. The
(C). What property regime governs the Mayor forthwith signed all the copies of
properties acquired by the couple? the marriage contract, gave them to the
Explain. (2%) secretary who returned to the Mayor’s
office. She then gave copies of the
SUGGESTED ANSWER: marriage contract to the parties, and told
Michael and Anna that they were already
The marriage being void, the married. Thereafter, the couple lived
property relationship that governed together as husband and wife, and had
their union is special co-ownership three sons.
under Article 147 of the Family Code.
This is on the assumption that there (A). Is the marriage of Michael and Anna
was no valid, voidable, or void? Explain your
answer. (3%)

“Never Let The Odds Keep You From Pursuing What You Know In Your Heart You Were Meant To Do.”-Leroy Satchel Paige
Page 31 of 180
Civil Law Q&As (2007-2013) hectorchristopher@yahoo.com dbaratbateladot@gmail.com

SUGGESTED ANSWER : (C). When Rona reaches seven (7) years


old, she tells Rodolfo that she prefers to
The marriage is void because the live with him, because he is better off
formal requisite of marriage financially than Nanette. If Rodolfo files an
ceremony was absent ( Art.3, F.C. action for the custody of Rona, alleging
209, Family Code). that he is Rona’s choice as custodial
parent, will the court grant Rodolfo’s
ALTERNATIVE ANSWER: petition? Why or why not? (2%)
The marriage is void because an
essential requisite was absent: SUGGESTED ANSWER:

consent of the parties freely given in No, because Rodolfo has no parental

the presence of the solemnizing authority over Rona. He who has the

officer (Art .2, FC). parental authority has the right to


custody. Under the Family Code, the
(B). What is the status of the three mother alone has parental authority
children of Michael and Anna? Explain your over the illegitimate child. This is
answer. (2%) true even if illegitimate father
recognized the child and even though
SUGGESTED ANSWER: he is giving support for the child. To
The children are illegitimate, having acquire custody over Rona, Rodolfo
been born outside a valid marriage. should first deprive Nanette of
parental authority if there is ground
under the law, and in a proper court
Parental Authority; Illegitimate Minor proceedings. In the same action, the
Child (2009) court may award custody of Rona to
Rodolfo if it is for her best interest.
No.XIV. Rodolfo, married to Sharon, had an
illicit affair with his secretary, Nanette, a
19-year old girl, and begot a baby girl,
Parental Authority; In Vitro
Rona. Nanette sued Rodolfo for damages:
Fertilization (2010)
actual, for hospital and other medical
expenses in delivering the child by No.VI. Gigolo entered into an agreement
caesarean section; moral, claiming that with Majorette for her to carry in her
Rodolfo promised to marry her, womb his baby via in vitro fertilization.
representing that he was single when, in Gigolo undertook to underwrite
fact, he was not; and exemplary, to teach Majorette’s pre-natal expenses as well as
a lesson to like-minded Lotharios. those attendant

“Never Let The Odds Keep You From Pursuing What You Know In Your Heart You Were Meant To Do.”-Leroy Satchel Paige
Page 32 of 180
Civil Law Q&As (2007-2013) hectorchristopher@yahoo.com dbaratbateladot@gmail.com

to her delivery. Gigolo would thereafter in Parañaque. After four (4) years or in
pay Majorette P2 million and, in return, 2001, G having completed her 4-year
she would give custody of the baby to college degree as a fulltime student, she
him. and B contracted marriage without a
license.
After Majorette gives birth and delivers the
baby to Gigolo following her receipt of P2 The marriage of B and G was, two years
million, she engages your services as her later, declared null and void due to the
lawyer to regain custody of the baby. absence of a marriage license.

23 Who of the two can exercise parental (B). Is Venus legitimate, illegitimate, or
authority over the child? Explain. (2.5%) legitimated? Explain briefly. (3%)

SUGGESTED ANSWER: SUGGESTED ANSWER:

Majorette, the mother, can exercise Venus is illegitimate. She was


parental authority. Since the child conceived and born outside a valid
was born out of wedlock, the child is marriage. Thus, she is considered
illegitimate and the mother has the illegitimate (Art 165, Family Code).
exclusive parental authority and While Venus was legitimated by the
custody over the child. subsequent marriage of her parents,
such legitimation was rendered
ALTERNATIVE ANSWER:
ineffective when the said marriage
Gigolo can exercise parental was later on declared null and void
authority over the child. Majorette due to absence of a marriage license.
has no blood relation to the child.
Under Article 178 of the Family Code,
She is just a “carrier” of the child.
“legitimation shall take place by a
subsequent valid marriage between
parents. The annulment of a voidable

Paternity & Filiation; Child Born marriage shall not affect the
Under a Void Marriage (2010) legitimation.” The inclusion of the
underscored portion in the Article
No.X. In 1997, B and G started living necessarily implies that the Article's
together without the benefit of marriage. application is limited to voidable
The relationship produced one offspring, marriages. It follows that when the
Venus. The couple acquired a residential
lot

“Never Let The Odds Keep You From Pursuing What You Know In Your Heart You Were Meant To Do.”-Leroy Satchel Paige
Page 33 of 180
Civil Law Q&As (2007-2013) hectorchristopher@yahoo.com dbaratbateladot@gmail.com

subsequent marriage is null or void, action to impugn, B can pray for the
the legitimation must also be null and correction of the status of the said
void. In the present problem, the daughter in her record of birth.
marriage between B and G was not
voidable but void. Hence, Venus has (B). If B acquiesces to the use of his

remained an illegitimate child. surname by G’s daughter by another man,


what is/are the consequence/s? Explain.
(5%)

Paternity & Filiation; Impugning SUGGESTED ANSWER:


Legitimacy (2010)
If B acquiesces and does not file the

No.IV. Spouses B and G begot two action to impugn the legitimacy of

offsprings. Albeit they had serious the child within the prescriptive

personality differences, the spouses period for doing so in Article 170 of

continued to live under one roof. B begot a the Family Code, G's daughter by

son by another woman. G also begot a another man shall be conclusively

daughter by another man. presumed as the legitimate daughter


of B by G.
(A). If G gives the surname of B to her
daughter by another man, what can B do
to protect their legitimate children's
Paternity & Filiation; In Vitro
interests? Explain. (5%)
Fertilization; Surrogate Mother’s

SUGGESTED ANSWER: Remedy to Regain Custody (2010)

B can impugn the status of G's No.VI. Gigolo entered into an agreement
daughter by another man as his with Majorette for her to carry in her
legitimate daughter on the ground womb his baby via in vitro fertilization.
that for biological reason he could Gigolo undertook to underwrite
not have been the father of the child, Majorette’s pre-natal expenses as well as
a fact that may be proven by the DNA those attendant to her delivery. Gigolo
test. Having been born during the would thereafter pay Majorette P2 million
marriage between B and G, G's and, in return, she would give custody of
daughter by another man is the baby to him.
presumed as the child of B under
Article 164 of the Family Code. In the After Majorette gives birth and delivers the
same baby to Gigolo following her receipt of P2

“Never Let The Odds Keep You From Pursuing What You Know In Your Heart You Were Meant To Do.”-Leroy Satchel Paige
Page 34 of 180
Civil Law Q&As (2007-2013) hectorchristopher@yahoo.com dbaratbateladot@gmail.com

million, she engages your services as her No, he cannot. Both he and Majorette
lawyer to regain custody of the baby. are guilty of violating the provision of
the Anti-Child Abuse Law (RA7610) on
768 What legal action can you file on child trafficking. Being in pari delicto,
behalf of Majorette? Explain. (2.5%) the partners shall be left where they
are and Gigolo cannot demand the
SUGGESTED ANSWER:
return of what he paid.

As her lawyer, I can file a petition for


ALTERNATIVE ANSWER:
habeas corpus on behalf Majorette to
recover custody of her child. Since Yes. The agreement between Gigolo
she is the mother of the child that and Majorette is a valid agreement.
was born out of wedlock, she has
exclusive parental authority and 0 Is the child entitled to support and

custody over the child. Gigolo, inheritance from Gigolo? Explain. (2.5%)

therefore, has no right to have


SUGGESTED ANSWER:
custody of the child and his refusal to
give up custody will constitute illegal If Gigolo voluntarily recognized the
detention for which habeas corpus is child as his illegitimate child in
the proper remedy. accordance with Article 175 in
relation to Article 172 of the Family
ALTERNATIVE ANSWER:
Code, the child is entitled to support
The action to regain custody will not and inheritance from Gigolo.
prosper. In the first place Majorette
ALTERNATIVE ANSWER:
cannot regain custody of the baby. As
surrogate mother she merely carries Yes, because Gigolo is the natural
the child in her womb for its and biological parent of the baby.
development. The child is the child of
the natural parents- Gigolo and his
partner. The agreement between
Gigolo and Majorette is a valid Paternity & Filiation; Legitimacy;

agreement. Presumption (2008)


No. III. Roderick and Faye were high school
0 Can Gigolo demand from Majorette the sweethearts. When Roderick was 18 and
return of the P2 million if he returns the Faye, 16 years old, they started to live
baby? Explain. (2.5%) together as husband and wife without the

SUGGESTED ANSWER:

“Never Let The Odds Keep You From Pursuing What You Know In Your Heart You Were Meant To Do.”-Leroy Satchel Paige
Page 35 of 180
Civil Law Q&As (2007-2013) hectorchristopher@yahoo.com dbaratbateladot@gmail.com

benefit of marriage. When Faye reached 18 March 2002, the Supreme Court ruled
years of age, her parents forcibly took her that impugning the legitimacy of the
back and arranged for her marriage to Brad. child is a strictly personal right of
Although Faye lived with Brad after the husband, except: (a) when the
marriage, Roderick continued to regularly husband died before the expiration of
visit Faye while Brad was away at work. the period fixed for bringing the
During their marriage, Faye gave birth to a action; (b) if he should die after the
baby girl, Laica. When Faye was 25 years filing of the complaint, without
old, Brad discovered her continued liason having desisted therefrom, or (c) if
with Roderick and in one of their heated the child was born after the death of
arguments, Faye shot Brad to death. She lost the husband. Laica's case does not
no time in marrying her true love Roderick, fall under any of the exceptions.
without a marriage license, claiming that
they have been continuosly cohabiting for (D). Can Laica be legitimated by the
more than 5 years. marriage of her biological parents? (1%)

(B). What is the filiation status of Laica? SUGGESTED ANSWER:


(2%)
No. Laica cannot be legitimated by
SUGGESTED ANSWER: the marriage of her biological parents
because only children conceived and
Laica is legitimate because children born outside of wedlock of parents
conceived or born during the who at the time of the conception of
marriage of the parents are the former were not disqualified by
presumed to be legitimate (Art. 164, any impediment to marry each other
FC). may be legitimated (Art. 177, FC).

(C).Can Laica bring an action to impugn


her own status on the ground that based
on DNA results, Roderick is her biological Paternity & Filiation; Legitimation of
father? (2%) a Child from a Previous Valid
Marriage (2008)
SUGGESTED ANSWER: No. IV. Gianna was born to Andy and
Aimee, who at the time Gianna's birth
No. Laica cannot bring an action to
were not married to each other. While
impugn her own status. In Liyao Jr. v.
Andy was single at the time, Aimee was
Tanhoti-Liyao, G.R. No. 138961, 07
still in the

“Never Let The Odds Keep You From Pursuing What You Know In Your Heart You Were Meant To Do.”-Leroy Satchel Paige
Page 36 of 180
Civil Law Q&As (2007-2013) hectorchristopher@yahoo.com dbaratbateladot@gmail.com

process of securing a judicial declaration statement is false. Explain your answer in


of nullity on her marriage to her ex- not more than two (2) sentences.
husband. Gianna's birth certificate, which
was signed by both Andy and Aimee, (E). A dead child can be legitimated. (1%)

registered the status of Gianna as


SUGGESTED ANSWER:
"legitimate", her surname carrying that of
Andy's and that her parents were married
TRUE To be legitimated, the law does
to each other.
not require a child to be alive at the
same time of the marriage of his / her
(C). Assuming that Aimee is successful in
parents ( Article 177, FC ).
declaring her former marriage void, and
Furthermore, Art. 181 of the Family
Andy and Aimee subsequently married
Code which states that “[Th]e
each other, would Gianna be legitimated?
llegitimation of children who died
(1%)
before the celebration of marriage

SUGGESTED ANSWER: will benefit their descendants,” does


not
Gianna cannot be legitimated by the preclude instances where such
subsequent marriage of Andy and legitimation will benefit no one but
Aimee. Art. 177 of the FC provides the child's ascendants ,or other
that "only children conceived and relatives .
born outside of wedlock of parents
who, at the time of the conception of
the former, were not disqualified by Paternity & Filiation;
Support:
any impediment to marry each other
Ascendants & Descendants; Collateral
may be legitimated." In the present
Blood Relatives (2008)
case, a legal impediment was existing
at the time of the conception of
No.V. Despite several relationships with
Gianna. Her mother, Aimee, was still
different women, Andrew remained
alive in the process of securing
unmarried. His first relationship with
judicial declaration of nullity on her
Brenda produced a daughter, Amy, now 30
marriage to her ex-husband.
years old. His second, with Carla,
produced two sons: Jon and Ryan. His
third, with Donna, bore him no children
Paternity & Filiation; Legitimation of
although Elena has a daughter Jane, from
a Dead Child (2009)
a previous relationship. His last, with Fe,
produced no biological children but they
No. I. TRUE or FALSE. Answer TRUE if the
informally adopted without court
statement is true, or FALSE if the
proceedings, Sandy's now 13 years old,
whom they consider as
“Never Let The Odds Keep You From Pursuing What You Know In Your Heart You Were Meant To Do.”-Leroy Satchel Paige
Page 37 of 180
Civil Law Q&As (2007-2013) hectorchristopher@yahoo.com dbaratbateladot@gmail.com

their own. Sandy was orphaned as a baby Paternity & Filiation; Use of Surname;
and was entrusted to them by the midwife Illegitimate Child (2009)
who attended to Sandy's birth. All the
children, including Amy, now live with No.XIV. Rodolfo, married to Sharon, had an

andrew in his house. illicit affair with his secretary, Nanette, a


19-year old girl, and begot a baby girl,
(B). In his old age, can Andrew be legally Rona. Nanette sued Rodolfo for damages:
entitled to claim support from Amy, Jon, actual, for hospital and other medical
Ryan, Vina, Wilma, and Sandy assuming expenses in delivering the child by
that all of them have the means to caesarean section; moral, claiming that
support him? (1%) Rodolfo promised to marry her,
representing that he was single when, in
SUGGESTED ANSWER: fact, he was not; and exemplary, to teach
a lesson to like-minded Lotharios.
Andrew, in his old age, cannot be
legally entitled to claim support (B). Suppose Rodolfo later on
because Art. 195, par 2 of the FC acknowledges Rona and gives her regular
limits the giving of support to support, can he compel her to use his
"legitimate ascendants and surname? Why or why not? (2%)
descendants."
SUGGESTED ANSWER:
(C). Can Amy, Jon, Ryan, Vina, Wilma, and No. he has no right to compel Rona to
Sandy legally claim support from each use his surname. The law does not
other? (2%) give him the right simply because he
gave her support (RA 9255).
SUGGESTED ANSWER:

Under the Family Code, an


Amy, Jon, Ryan, Vina, Wilma and
illegitimate child was required to use
Sandy cannot legally claim support
only the surname of the mother.
from each other because Art. 195, par
Under RA 9255, otherwise known as
5 limits the giving of support to
the Revilla law, however, the
"legitimate brothers and sisters,
illegitimate child is given the option
whether full or half blood."
to use the surname of the illegitimate
father when the latter has recognized
the former in accordance with law.
Since the choice belongs to the
illegitimate child, Rodolfo cannot

“Never Let The Odds Keep You From Pursuing What You Know In Your Heart You Were Meant To Do.”-Leroy Satchel Paige
Page 38 of 180
Civil Law Q&As (2007-2013) hectorchristopher@yahoo.com dbaratbateladot@gmail.com

compel Rona, if already of age, to use They are not related at all to
the surname against her will. If Rona Edilberto. They were born during the
is still a minor, to use the surname of marriage of Conrado and Clarita,
Rodolfo will require the consent of hence, are considered legitimate
Rona's mother who has sole parental children of the said spouses. This
authority over her. status is conferred on them at birth
by law.

Paternity & Filiation; Who May Under Philippine law, a person cannot
Impugn Legitimacy (2009) have more than one natural filiation.
The legitimate filiation of a person
No.V. Four children, namely: Alberto, can be changed only if the legitimate
Baldomero, Caridad, and Dioscoro, were father will successfully impugn such
born to the spouses Conrado and Clarita status.
de la Costa. The children’s birth
certificates were duly signed by Conrado, In the problem, therefore, the
showing them to be the couple’s filiation of Alberto and Baldomero as
legitimate children. legitimate children of Condrado
cannot be changed by their
Later, one Edilberto de la Cruz executed a
recognition by Edilberto as his
notarial document acknowledging Alberto
illegitimate children. Before they can
and Baldomero as his illegitimate children
be conferred the status of Edilberto’s
>with Clarita. Edilberto died leaving
illegitimate children, Condrado must
substantial properties. In the settlement of
first impugn their legitimacy. Since
his estate, Alberto and Baldomero
Condrado has not initiated any action
intervened claiming shares as the
to impugn their legitimacy, they
deceased’s illegitimate children. The
continue to be the legitimate of
legitimate family of Edilberto opposed the
Condrado. They cannot be the
claim.
illegitimate children of Edilberto at
the same time. Not being the
Are Alberto and Baldomero entitled to
illegitimate children of Edilberto, they
share in the estate of Edilberto? Explain.
have no right to inherit from him.
(4%)

SUGGESTED ANSWER:
Property Relations; Adulterous

No, Alberto and Baldomero are not Relationship (2009)

entitled to share in Edilberto’s


No. XI. TRUE or FALSE. Answer TRUE if
estate.
the statement is true, or FALSE if the
“Never Let The Odds Keep You From Pursuing What You Know In Your Heart You Were Meant To Do.”-Leroy Satchel Paige
Page 39 of 180
Civil Law Q&As (2007-2013) hectorchristopher@yahoo.com dbaratbateladot@gmail.com

statement is false. Explain your answer in SUGGESTED ANSWER:


not more than two (2) sentences.
It depends. If the value of the

(B). If there is no marriage settlement, the building is more than the value of the

salary of a "spouse" in an adulterous land, the building is conjugal and the

marriage belongs to the conjugal land becomes conjugal property

partnership of gains. (1%) under Art. 120 of the Family Code.


This is a case of reverse accession,
SUGGESTED ANSWER: where the building is considered as
False. In adulterous relationship, the the principal and the land, the
salary of a married partner belongs accessory. If, on the other hand, the
to the absolute community, or value of the land is more than the
conjugal partnership, of such married value of the building, then the
partner with his or her lawful spouse. ordinary rule of accession applies
Under Articles 148 of the Family where the land is the principal and
Code, the property relations between the building, the accessory. In such
married partner and his/her paramour case, the land remains paraphernal
is governed by ordinary co-ownership property and the building becomes
where the partners become co- paraphernal propery.
owners only when they contributed to
Note: The rule on reverse accession is
the acquisition of the property. The
applicable only to the regime of conjugal
paramour is deemed to have not
partnership of gains in both the Family
contributed in the earning of the
Code and the New Civil Code. The
salary of the married partner.
foregoing answer assumes that CPG is the
regime of the property relations of the
spouses.

Property Relations; Accession (2012)

No.III.(a) Maria, wife of Pedro, withdrew P 5


Property Relations; Ante-Nuptial Debt
Million from their conjugal funds. With this
(2007)
money, she constructed a building on a lot
which she inherited from her father. Is the
No. VII. Write "TRUE" if the statement is
building conjugal or paraphernal?
true or "FALSE" if the statement is false. If
Reasons. (5%)
the statement is FALSE, state the reason.
(2% each).

(3). An individual, While single, purchases


a house and lot in 1990 and borrows
money
“Never Let The Odds Keep You From Pursuing What You Know In Your Heart You Were Meant To Do.”-Leroy Satchel Paige
Page 40 of 180
Civil Law Q&As (2007-2013) hectorchristopher@yahoo.com dbaratbateladot@gmail.com

in 1992 to repair it. In 1995, such Property Relations; Unions Without


individual gets married while the debt is Marriage (2012)
still being paid. After the marriage, the
debt is still the responsibility of such No.V. a) Spouses Primo and Monina Lim,

individual. childless, were entrusted with the custody


of two (2) minor children, the parents of
SUGGESTED ANSWER: whom were unknown. Eager of having
children of their own, the spouses made it
FALSE. The absolute Community of appear that they were the children’s
property is liable for the ante-nuptial parents by naming them Michelle P. Lim
debts of either spouse in so far as the and Michael Jude Lim. Subsequently,
same redounded to the benefit of the Monina married Angel Olario after Primo’s
family (Art. 94 par.7, FC). death.

ALTERNATIVE ANSWER: She decided to adopt the children by


availing the amnesty given under R.A.
FALSE. The debt is already the
8552 to those individuals who simulated
responsibility of the community
the birth of a child. She filed separate
property, because the property
petitions for the adoption of Michelle, then
already
25 years old and Michael, 18. Both
constitutes absolute community
Michelle and Michael gave consent to the
property under Art. 91 of FC which
adoption.
took effect in 1988 while the house
and lot here involved was purchased The trial court dismissed the petition and
in 1990. There is no indication that ruled that Monina should have filed the
the spouse who bought the property petition jointly with her new husband.
had legitimate descendants by a Monina, in a Motion for Reconsideration
former marriage, which would argues that mere consent of her husband
exclude the house and lot from the would suffice and that joint adoption is not
community property, Art. 92 par 3, needed, for the adoptees are already
FC). If the spouses established a emancipated.
conjugal partnership, the property
belongs to the individual spouse if Is the trial court correct in dismissing the
full ownership was vested before petitions for adoption? Explain. (5%)
marriage (Art. 118, FC).

SUGGESTED ANSWER:

“Never Let The Odds Keep You From Pursuing What You Know In Your Heart You Were Meant To Do.”-Leroy Satchel Paige
Page 41 of 180
Civil Law Q&As (2007-2013) hectorchristopher@yahoo.com dbaratbateladot@gmail.com

Yes, the trial court was correct. At the Borromeo discovered that titles to the
three
time the petitions for adoptions were
0 lots have been transfereed in the name
filed, petitioner had already
of Descallar. Who is the rightful owner of
remarried. Under the law, husband
the properties? Explain. (5%)
and wife shall adopt jointly, except in
the cases enumerated in the law. The
SUGGESTED ANSWER:
adoption cases of Michelle and James
do not fall in any of the exceptions It depends. On the assumption that
provided in the law where a spouse is the Family Code is the applicable law,
permitted to adopt alone. Hence, the ownership of the properties
Monina should adopt jointly with her depends on whether or not, Jambrich
husband Angel (Adoption of Michelle and Descallar are capacitated to
P. Lim, G.R. Nos. 168992-93, May 21, marry each other during their
2009). cohabitation, and whether or not
both have contributed funds for the
acquisition of the properties.

Property Relations; Unions Without If both of them are capacitated to


Marriage (2012) marry each other, Art 147- co-
ownership will apply to their property
No.V. b) Jambrich, an Austrian, fell in-love
relations and the properties in
and lived together with Descallar and
question are owned by them in equal
bought their houses and lots at Agro-
shares even though all the funds
Macro Subdivision. In the Contracts to Sell,
used in acquiring the properties came
Jambrich and Descallar were referred to as
only from the salaries or wages, or
the buyers. When the Deed of Absolute
the income of Jambrich from his
Sale was presented for registration before
business or profession. In such case,
the Register of Deeds, it was refused
while Jambrich is disqualified to own
because Jambrich was an alien and could
any part of the properties, his
not acquire alienable lands of the public
subsequent transfer of all his interest
domain. After Jambrich and Descallar
therein to Borromeo, a Filipino, was
separated, Jambrich purchased an engine
valid as it removed the
and some accessories for his boat from
disqualification. In such case, the
Borromeo. To pay for his debt, he sold his
properties are owned by Borromeo
rights and interests in the Agro-Macro
and Descallar in equal shares.
properties to Borromeo.
If, on the other hand, Jambrich and
Descallar were not capacitated to
marry
“Never Let The Odds Keep You From Pursuing What You Know In Your Heart You Were Meant To Do.”-Leroy Satchel Paige
Page 42 of 180
Civil Law Q&As (2007-2013) hectorchristopher@yahoo.com dbaratbateladot@gmail.com

each other Art. 148-co-ownership marriage, the couple possessed the


governs their property relations. following properties:
Under this regime, Jambrich and
Descallar are co-owners of the 0.0 a house and lot acquired by B on

properties but only if both of them August 3, 1988, one third (1/3) of

contributed in their acquisition. If all the purchase price (representing

the funds used in acquiring the downpayment) of which he paid;

properties in question came from one third (1/3) was paid by G on

Jambrich, the entire property is his February 14, 1990 out of a cash gift

even though he is disqualified from given to her by her parents on her

owning it. His subsequent transfer to graduation on April 6, 1989; and

Borromeo, however, is valid as it the balance was paid out of the

removed the disqualification. In such spouses’ joint income; and

case, all the properties are owned by 0.1 an apartment unit donated to B by

Borromeo. If, on the other hand an uncle on June 19, 1987.

Descallar contributed to their


1Who owns the foregoing properties?
acquisition, the properties are co-
Explain. (5%)
owned by Descallar and Borromeo in
proportion to the respective
SUGGESTED ANSWER:
contributions of the Descallar and
Jambrich. Since the marriage was declared void
ab initio in 2001, no Absolute
Note: The facts of the problem are
Community or Conjugal Partnership
not exactly the same as in the case of
was ever established between B and
Borromeo v. Descallar, G.R. NO.
G. Their property relation is governed
159310, Feb 24, 2009, hence, the
by a “special co-ownership” under
difference in the resulting answer.
Article 147 of the Family Code
because they were capacitated to

Property Relations; Void Marriages marry each other.

(2010)
Under that Article 147, wages and
salaries of the “former spouses”
No.VII. G and B were married on July 3,
earned during their cohabitation shall
1989. On March 4, 2001, the marriage,
be owned by them in equal shares
which bore no offspring, was declared void
while properties acquired thru their
ab initio under Article 36 of the Family
work for industry shall be owned by
Code. At the time of the dissolution of the
them in proportion to

“Never Let The Odds Keep You From Pursuing What You Know In Your Heart You Were Meant To Do.”-Leroy Satchel Paige
Page 43 of 180
Civil Law Q&As (2007-2013) hectorchristopher@yahoo.com dbaratbateladot@gmail.com

their respective contributions. Care ࿿Ā࿿Ā࿿Ā࿿Ā࿿Ā࿿Ā࿿Ā࿿ĀĀ̀ȀȀ̀⠀ⓦĀ࿿Ā࿿Ā࿿


and maintenance of the family is
recognized as a valuable Ā࿿Ā࿿Ā࿿Ā࿿Ā࿿0 She is an

contribution. In the absence of proof undivided co-owner to the extent for


as to the value of their respective her contribution in its acquisition
contributions, they shall share when she paid 1/3 of the purchase
equally. price using the gift from her parents.
Although the gift was acquired by G
If ownership of the house and lot was
during her cohabitation with B, it is
acquired by B on August 3, 1988 at
her exclusive property. It did not
the time he bought it on installment
consist of wage or salary or fruit of
before he got married, he shall
her work or industry.
remain owner of the house and lot
but he must reimburse G for all the
0 1/3 of the house is co-owned by B
amounts she advanced to pay the
and G because the payment came
purchase price and for one-half share
from their co-owned funds, i.e., their
in the last payment from their joint
joint income during their cohabitation
income. In such case, the house and
which is shared by them equally in
lot were not acquired during their
the absence of any proof to the
cohabitation, hence, are not co-
contrary.
owned by B and G.

But if the ownership of the house and After summing up their prospective
lot was acquired during the shares, B and G are undivided co-
cohabitation, the house and lot will owners of the house and lot in equal
be owned as follows: shares.

0 1/3 of the house and lot is owned As to the apartment, it is owned


by B. He is an undivided co-owner to exclusive by B because he acquired it
that extent for his contributions in its before their cohabitation. Even if he
acquisition in the form of the down acquired it during their cohabitation,
payment he made before the it will still be his exclusive property
celebration of the marriage. The because it did not come from his
money he used to pay the down wage or salary, or from his work or
payment was not earned during the industry. It was acquired gratuitously
cohabitation, hence, it is his from his uncle.
exclusive property.
0 If G and B had married on July 3, 1987
and their marriage was dissolved in 2007,
1 1/3 of the house and lot is owned
by who owns the properties? Explain. (5%)
SUGGESTED ANSWER:

“Never Let The Odds Keep You From Pursuing What You Know In Your Heart You Were Meant To Do.”-Leroy Satchel Paige
Page 44 of 180
Civil Law Q&As (2007-2013) hectorchristopher@yahoo.com dbaratbateladot@gmail.com

The answer is the same as in letter A. in Parañaque. After four (4) years or in
Since the parties to the marriage 2001, G having completed her 4-year
which was later declared void ab college degree as a fulltime student, she
initio were capacitated to marry each and B contracted marriage without a
other, the applicable law under the license.
New Civil Code was Article 144.This
Article is substantially the same as The marriage of B and G was, two years

Article 147 of the Family Code. later, declared null and void due to the
absence of a marriage license.
Hence, the determination of
ownership will remain the same as in (A). If you were the judge who declared
question A. And even assuming that the nullity of the marriage, to whom would
the two provisions are not the same, you award the lot? Explain briefly. (3%)
Article 147 of the Family Code is still
the law that will govern the property SUGGESTED ANSWER:
relations of B and G because under
Since the marriage was null and void,
Article 256, the Family Code has
no Absolute Community or Conjugal
retroactive effect insofar as it does
Partnership was established between
not prejudice or impair vested or
B and G. Their properties are
acquired rights under the new Civil
governed by the “special co-
Code or other laws. Applying Article
ownership” provision of Article 147 of
147 retroactively to the case of G and
the Family Code because both B and
B will not impair any vested right.
G were capacitated to marry each
Until the declaration of nullity of the
other. The said Article provides that
marriage under the Family Code, B
when a man and a woman who are
and G have not as yet acquired any
capacitated to marry each other, live
vested right over the properties
exclusively with each other as
acquired during their cohabitation.
husband and wife without the benefit
of marriage, or under a void
marriage: (1) their wages and
Property Relations; Void Marriages
salaries shall be owned by them in
(2010)
equal shares; and (2) property
acquired by both of them through
No.X. In 1997, B and G started living
their work or industry shall be
together without the benefit of marriage.
governed by the rules on co-
The relationship produced one offspring,
ownership. In co-ownership, the
Venus. The couple acquired a residential
parties are co-owners if they
lot
contributed something of value in the
acquisition of
“Never Let The Odds Keep You From Pursuing What You Know In Your Heart You Were Meant To Do.”-Leroy Satchel Paige
Page 45 of 180
Civil Law Q&As (2007-2013) hectorchristopher@yahoo.com dbaratbateladot@gmail.com

the property. Their share is in Succession


proportion to their
respective
Disposition; Mortis Causa vs.
contributions. In an ordinary co-
Intervivos; Corpse (2009)
ownership the care and maintenance
of the family is not recognized as a
No. XI. TRUE or FALSE. Answer TRUE if
valuable contribution for the
the statement is true, or FALSE if the
acquisition of a property. In the
statement is false. Explain your answer in
Article 147 “special co-ownership”
not more than two (2) sentences.
however, care and maintenance is
recognized as a valuable contribution (E). A person can dispose of his corpse
which will entitle the contributor to through an act intervivos. (1%)
half of the property acquired.
SUGGESTED ANSWER:
Having been acquired during their
False. A persons cannot dispose of his
cohabitation, the residential lot is
corpse through an act inter vivos,
presumed acquired through their
i.e., an act to take effect during his
joint work and industr under Article
lifetime. Before his death there is no
147, hence, B and G are co-owners of
corpse to dispose. But he is allowed
the said property in equal shares.
to do so through an act mortis causa,

Article 147 also provides that when a i.e., an act to take effect upon his

party to the void marriage was in bad death.

faith, he forfeits his share in the co-


ownership in favor of the common
children or descendants, the default
Heirs; Fideicommissary Substitution
of children or descendants, the
(2008)
forfeited share shall belong to the
innocent party. In the foregoing No. XIII. Raymond, single, named his sister
problem, there is no showing that Ruffa in his will as a devisee of a parcel of
one party was in bad faith. Hence, land which he owned. The will imposed
both shall be presumed in good faith upon Ruffa the obligation of preseving the
and no forfeiture shall take place. land and transferring it, upon her death, to
her illegitimate daughter Scarlet who was
then only one year old. Raymond later
died, leaving behind his widowed mother,
Ruffa and Scarlet.

“Never Let The Odds Keep You From Pursuing What You Know In Your Heart You Were Meant To Do.”-Leroy Satchel Paige
Page 46 of 180
Civil Law Q&As (2007-2013) hectorchristopher@yahoo.com dbaratbateladot@gmail.com

(A). Is the condition imposed upon Ruffa, Ruffa (Art. 992, Civil Code). Moreover,
to preserve the property and to transmit it Scarlet is not a compulsory heir of
upon her death to Scarlet, valid? (1%) Raymond, hence she can inherit only
by
SUGGESTED ANSWER: testamentary succession. Since
Raymond executed a will in the case
Yes, the condition imposed upon
at bar, Scarlet may inherit from
Ruffa to preserve the property and to
Raymond.
transmit it upon her death to Scarlet
is valid
because it is tantamount to
fideicommissary substitution under Heirs; Intestate Succession; Legitime;
Art. 863 of the Civil Code. Computation (2010)

(B). If Scarlet predeceases Ruffa, who No.XI. The spouses Peter and Paula had
inherits the property? (2%) three (3) children. Paula later obtained a
judgment of nullity of marriage. Their
SUGGESTED ANSWER: absolute community of property having
been dissolved, they delivered P1 million
Ruffa will inherit the property as
to each of their 3 children as their
Scarlet's heir. Scarlet acquires a right
presumptive legitimes.
to the succession from the time of
Raymond's death, even though she Peter later re-married and had two (2)
should predecease Ruffa (Art. 866, children by his second wife Marie. Peter
Civil Code). and Marie, having successfully engaged in
business, acquired real properties. Peter
(C). If Ruffa predeceases Raymond, can
later died intestate.
Scarlet inherit the property directly from
Raymond? (2%) (A). Who are Peter’s legal heirs and how
will his estate be divided among them?
SUGGESTED ANSWER:
(5%)

If Ruffa predeceases Raymond,


SUGGESTED ANSWER:
Raymond's widowed mother will be
entitled to the inheritance. Scarlet, The legal heirs of Peter are his
an illegitimate child, cannot inherit children by the first and second
the property by intestate succession marriages and his surviving second
from Raymond who is a legitimate wife.
relative of
Their shares in the estate of Peter
will depend, however, on the cause of
the

“Never Let The Odds Keep You From Pursuing What You Know In Your Heart You Were Meant To Do.”-Leroy Satchel Paige
Page 47 of 180
Civil Law Q&As (2007-2013) hectorchristopher@yahoo.com dbaratbateladot@gmail.com

nullity of the first marriage. If the 0 If the ground of nullity is not


nullity of the first marriage was psychological capacity:
psychological incapacity of one or 2 legitimate ¼ of the estate for
both spouses, the three children of children each o second
f
that void marriage are legitimate and marriage
all of the legal heirs shall share the
Surviving ¼ of the estate
estate of Peter in equal shares. If the
second spouse
judgment of nullity was for other
causes, the three children are 3 illegitimate 1/12 of estate for
children each of first marriage
illegitimate and the estate shall be
distributed such that an illegitimate
child of the first marriage shall
receive half of the share of a Note: The legitime of an illegitimate
legitimate child of the second child is supposed to be ½ the legitime
marriage, and the second wife will of a legitimate child or 1/8 of the
inherit a share equal to that of a estate. But the estate will not be
legitimate child. In no case may the sufficient to pay the said legitime of
two legitimate children of the second the 3 illegitimate children, because
marriage receive a share less than only ¼ of the estate is left after
one-half of the estate which is their paying the legitime of the surviving
legitime. When the estate is not spouse which is preferred.
sufficient to pay all the legitimes of
Hence, the remaining ¼ of the estate
the compulsory heirs, the legitime of
shall be divided among the 3
the spouse is preferred and the
illegitimate children.
illegitimate children suffer the
reduction.
(B). What is the effect of the receipt by
Peter’s 3 children by his first marriage of
Computation:
their presumptive legitimes on their right
23 If the ground of nullity is to inherit following Peter’s death? (5%)
psychological incapacity:
3 children by first 1/6 of the estate SUGGESTED ANSWER:
marriage for each
In the distribution of Peter’s estate,
2 children by second 1/6 of the estate ½ of the presumptive received by the
marriage for each
3 children of the first marriage shall

Surviving second 1/6 of the estate be collated to Peter’s estate and shall
spouse be

“Never Let The Odds Keep You From Pursuing What You Know In Your Heart You Were Meant To Do.”-Leroy Satchel Paige
Page 48 of 180
Civil Law Q&As (2007-2013) hectorchristopher@yahoo.com dbaratbateladot@gmail.com

imputed as an advance of their Art 992 of the NCC, an illegitimate


respective inheritance from Peter. child has no right to inherit ab
Only half of the presumptive legitime intestato from the legitimate children
is collated to the estate of Peter and relatives of his father or mother.
because the other half shall be Arnel is disqualified to inherit from
collated to the estate of his first wife. Ricky because Arnel is an illegitimate
child of Franco and Ricky is a
legitimate relative of Franco.

Heirs; Representation; Iron-Curtain


Rule (2012)
Heirs; Reserva Troncal (2009)
No.VIII.a) Ricky and Arlene are married.
They begot Franco during their marriage. No. I. TRUE or FALSE. Answer TRUE if the
Franco had an illicit relationship with statement is true, or FALSE if the
Audrey and out of which, they begot Arnel. statement is false. Explain your answer in
Frnaco predeceased Ricky, Arlene and not more than two (2) sentences.
Arnel. Before Ricky died, he executed a
will which when submitted to probate was (B).In reservatroncal, all reservatarios

opposed by Arnel on the ground that he (reser vees) inherit as a class and in equal

should be given the share of his father, shares regardless of their proximity in

Franco. Is the opposition of Arnel correct? degree to the prepositus. (1%)

Why? (5%)
SUGGESTED ANSWER:

SUGGESTED ANSWER: FALSE. Not all the relatives within the


third degree will inherit as
No, his opposition is not correct. reservatario , and not all those who
Arnel cannot inherit from Ricky in the are entitled to inherit will inherit in
representation of his father Franco. the equal shares . The applicable
In representation, the representative laws of intestate succession will
must not only be a legal heir of the determine who among the relatives
person he is representing, he must will inherit as reservatarios and what
also be a legal heir of the decedent shares they will tak, i.e., the direct
he seeks to inherit from. line excludes the collateral, the
descending direct line excludes the
While Arnel is a legal heir of Franco,
ascending ,the nearer excludes the
he is not a legal heir of Ricky because
more remote, the nephews and
under
nieces exclude

“Never Let The Odds Keep You From Pursuing What You Know In Your Heart You Were Meant To Do.”-Leroy Satchel Paige
Page 49 of 180
Civil Law Q&As (2007-2013) hectorchristopher@yahoo.com dbaratbateladot@gmail.com

the uncles and the aunts, and half (1). The wife of Ramon will, therefore,
blood relatives inherit half the share receive one half (½) of the estate or
of full-blooded relatives. the amount of P5,000,000.00.
(2). The three (3) full-blood brothers,
will, therefore, receive P1,000,000.00
each.
Intestate Succession (2008)
(3). The nephew will
receive
No. VII. Ramon Mayaman died intestate, P1,000,000.00 by right of
leaving a net estate of P10,000,000.00. representation.
Determine how much each heir will (4). The two (2) half-brothers will
receive from the estate: receive
P500,000.00 each.
(A). If Ramon is survived by his wife, three
full-blood brothers, two half-brothers, and (B). If Ramon is survived by his wife, a
one nephew (the son of a deceased full- half-sister, and three nephews (sons of a
blood brother)? Explain. (3%) deceased full-blood brother)? Explain.
(3%)
SUGGESTED ANSWER:
Having died intestate, the estate of SUGGESTED ANSWER:

Ramon shall be inherited by his wife The wife will receive one half (1/2) of

and his full and half blood siblings or the estate or P5,000,000.00. The

their respective representatives. In other half shall be inherited by (1)

intestacy, if the wife concurs with no the full-blood brother, represented by

one but the siblings of the husband, his three children, and (2) the half-

all of them are the intestate heirs of sister. They will divide the other half

the deceased husband. The wife will between them such that the share of

receive half of the intestate estate, the half-sister is just half the share of

while the siblings or their respective the full-blood brother. The share of

representatives, will inherit the other the full-blood brother shall in turn be

half to be divided among them inherited by the three nephews in

equally. If some siblings are of the equal shares by right of presentation.

full-blood and the other of the half


blood, a half blood sibling will receive Therefore, the three (3) nephews will
half the share of a full-blood sibling. receive P1,111,111.10 each the half-
sister will receive the sum of
P1,666,666.60.
“Never Let The Odds Keep You From Pursuing What You Know In Your Heart You Were Meant To Do.”-Leroy Satchel Paige
Page 50 of 180
Civil Law Q&As (2007-2013) hectorchristopher@yahoo.com dbaratbateladot@gmail.com

Intestate Succession (2008) (D). How should the house and lot, and the
cash be distributed? (1%)
No.X. Arthur executed a will which contained
only: (i) a provision disinheriting his daughter SUGGESTED ANSWER:
Bernica for running off with a married man,
and (ii) a provision disposing of his share in Since the probate of the will cannot

the family house and lot in favor of his other be allowed, the rules on intestate
children Connie and Dora. He did not make succession apply. Under Art. 996 of
any provisions in favor of his wife Erica, the Civil Code, if a widow or widower
because as the will stated, she would and legitimate children or
anyway get ½ of the house and lot as her descendants are left, the surviving
conjugal share. The will was very brief and spouse has the same share as of the
straightforward and both the above children. Thus, ownership over the
provisions were contained in page 1, which house and lot will be created among
Arthur and his instrumental witness, signed wife Erica and her children Bernice,
at the bottom. Page 2 contained the Connie and Dora. Similarly, the
attestation clause and the signatures, at the amount of P 1 million will be equally
bottom thereof, of the 3 instrumental divided among them.
witnesses which included Lambert, the
driver of Arthur; Yoly, the family cook, and
Attorney Zorba, the lawyer who prepared the
Intestate Succession; Rights
will. There was a 3rd page, but this only of
contained the notarial acknowledgement. Representation: Illegitimate, Adopted
The attestation clause stated the will was Child; Iron Curtain Rule (2007)
signed on the same occasion by Arthur and
his instrumental witnesses who all signed in No. X. For purpose of this question,
the presence of each other, and the notary assume all formalities and procedural
public who notarized the will. There are no requirements have been complied with.
marginal signatures or pagination appearing
on any of the 3 pages. Upon his death, it was In 1970, Ramon and Dessa got married.
discovered that apart from the house and Prior to their marriage, Ramon had a child,
lot, he had a P 1 million account deposited Anna. In 1971 and 1972, Ramon and
with ABC bank. Dessa legally adopted Cherry and Michelle
respectively. In 1973, Dessa died while
giving birth to Larry Anna had a child, Lia.
Anna never married. Cherry, on the other
hand, legally adopted Shelly. Larry had

“Never Let The Odds Keep You From Pursuing What You Know In Your Heart You Were Meant To Do.”-Leroy Satchel Paige
Page 51 of 180
Civil Law Q&As (2007-2013) hectorchristopher@yahoo.com dbaratbateladot@gmail.com

twins, Hans and Gretel, with his girlfriend, also of the person from whom the
Fiona. In 2005, Anna, Larry and Cherry person being represented was
died in a car accident. In 2007, Ramon supposed to inherit. While Shelly is a
died. Who may inherit from Ramon and legal heir of Cherry, Shelly is not a
who may not? Give your reason briefly. legal heir of Ramon. Adoption created
(10%) a purely personal legal relation only
between Cherry and Shelly.
SUGGESTED ANSWER:
(2). Hans and Gretel are barred from
The following may inherit from inheriting from Ramon under Art.
Ramon:
992, NCC. Being illegitimate children,
they cannot inherit ab intestao from
(1). Michelle, as an adopted child of
Ramon.
Ramon, will inherit as a legitimate
child of Ramon. As an adopted child,
ALTERNATIVE ANSWER:
Michelle has all the rights of a
legitimate child (Sec 18, Domestic
The problem expressly mentioned the
Adoption Law).
dates of the adoption of Cherry and
Michelle as 1971 and 1972. During
(2). Lia will inherit in representation
that time, adoption was governed by
of Anna. Although Lia is an
the New Civil Code. Under the New
illegitimate child, she is not barred by
Civil Code, husband and wife were
Articles 992, because her mother
allowed to adopt separately or not
Anna is an illegitimate herself. She
jointly with the other spouse. And
will represent Anna as regards Anna's
since the problem does not
legitime under Art. 902, NCC and as
specifically and categorically state, it
regards Anna's intestate share under
is possible to construe the use of the
Art. 990, NCC.
word "respectively" in the problem as
indicative of the situation that Cherry
The following may not inherit
from was adopted by Ramon alone and
Ramon: Michelle was adopted by Dessa alone.
In such case of separate adoption the
(1). Shelly, being an adopted child, alternative answer to the problem
she cannot represent Cherry. This is will be as follows: Only Lia will inherit
because adoption creates a personal from Ramon in representation of
legal relation only between the Ramon's illegitimate daughter Anna.
adopter and the adopted. The law on Although Lia is an illegitimate child,
representation requires the she is not barred from inheriting from
representative to be a legal heir of Ramon because her
the person he is representing and
“Never Let The Odds Keep You From Pursuing What You Know In Your Heart You Were Meant To Do.”-Leroy Satchel Paige
Page 52 of 180
Civil Law Q&As (2007-2013) hectorchristopher@yahoo.com dbaratbateladot@gmail.com

mother is herself illegitimate. Shelly SUGGESTED ANSWER:


cannot inherit in representation of
A testator may dispose of by will the
Cherry because Shelly is just an
free portion of his estate. Since the
adopted child of Cherry. In
legitime of JCP is 1/8 of the estate,
representation, the representative
SGO is ¼ of the estate and that of
must not only be a legal heir of the
HBR and RVC is ½ of the hereditary
person he is representing but also of
estate under Art 889 of the NCC, the
the decedent from whom the
remaining 1/8 of the estate is the free
represented person is supposed to
portion which the testator may
inherit. In the case of Shelly, while
dispose of by will.
she is a legal heir of Cherry by virtue
of adoption, she is not a legal heir of
Ramon. Adoption creates a personal
legal relation only between the Legitime; Compulsory Heirs
adopting parent and the adopted (2008)

child (Teotico v. Del Val, 13 SCRA 406,


No. XII. Ernesto, an overseas Filipino
1965. Michelle cannot inherit from
worker, was coming home to the
Ramon, because she was adopted not
Philippines after working for so many
by Ramon but by Dessa. In the eyes
years in the Middle East. He had saved
of the law, she is not related to
P100.000 in his saving account in Manila
Ramon at all. Hence, she is not a
which intended to use to start a business
legal heir of Ramon. Hans and Gretel
in his home country. On his flight home,
are not entitled to inherit from
Ernesto had a fatal heart attack. He left
Ramon, because they are barred by
behind his widowed mother, his common-
Art. 992 NCC. Being illegitimate
law wife and their twins sons. He left no
children of Larry, they cannot inherit
will, no debts, no other relatives and no
from the legitimate relatives of their
other properties except the money in his
father Larry. Ramon is a legitimate
saving account. Who are the heirs entitled
relative of Larry who is the legitimate
to inherint from him and how much should
father.
each receive?(3%)

SUGGESTED ANSWER:

Legitimes; Compulsory Heirs (2012)


The mother and twin sons are

No.VIII.b) How can RJP distribute his estate entitled to inherit from Ernesto. Art.

by will, if his heirs are JCP, his wife; HBR 991 of the Civil Code, provides that if

and RVC, his parents; and an illegitimate legitimate

child, SGO?
“Never Let The Odds Keep You From Pursuing What You Know In Your Heart You Were Meant To Do.”-Leroy Satchel Paige
Page 53 of 180
Civil Law Q&As (2007-2013) hectorchristopher@yahoo.com dbaratbateladot@gmail.com

ascendants are left, the twin sons The attestation clause stated the will was
shall divide the inheritance with them signed on the same occasion by Arthur
taking one-half of the estate. Thus, and his instrumental witnesses who all
the widowed mother gets P50,000.00 signed in the presence of each other, and
while the twin sons shall receive the notary public who notarized the will.
P25,000.00 each. The common-law There are no marginal signatures or
wife cannot inherit from him because pagination appearing on any of the 3
when the law speaks "widow or pages. Upon his death, it was discovered
widower" as a compulsory heir, the that apart from the house and lot, he had
law refers to a legitimate spouse (Art. a P 1 million account deposited with ABC
887, par 3, Civil Code). bank.

(A). Was Erica preterited? (1%)

Preterition; Disinheritance (2008) SUGGESTED ANSWER:

No.X. Arthur executed a will which contained Erica cannot be preterited. Art. 854 of
only: (i) a provision disinheriting his daughter the Civil Code provides that only
Bernica for running off with a married man, compulsory heirs in the direct line
and (ii) a provision disposing of his share in can be preterited.
the family house and lot in favor of his other
children Connie and Dora. He did not make (B). What other defects of the will, if any,
any provisions in favor of his wife Erica, can cause denial of probate? (2%)
because as the will stated, she would
SUGGESTED ANSWER:
anyway get ½ of the house and lot as her
conjugal share. The will was very brief and
The other defects of the will that can
straightforward and both the above
cause its denial are as follows: (a)
provisions were contained in page 1, which
Atty. Zorba, the one who prepared
Arthur and his instrumental witness, signed
the will was one of the three
at the bottom. Page 2 contained the
witnesses, violating the three-
attestation clause and the signatures, at the
witnesses rule; (b) no marginal
bottom thereof, of the 3 instrumental
signature at the last page; (c ) the
witnesses which included Lambert, the
attestation did not state the number
driver of Arthur; Yoly, the family cook, and
of pages upon which the will is
Attorney Zorba, the lawyer who prepared the
written; and, (d) no pagination
will. There was a 3rd page, but this only
appearing correlatively in letters on
contained the notarial acknowledgement.
the upper part of the three pages
(Azuela v. C.A., G.R.

“Never Let The Odds Keep You From Pursuing What You Know In Your Heart You Were Meant To Do.”-Leroy Satchel Paige
Page 54 of 180
Civil Law Q&As (2007-2013) hectorchristopher@yahoo.com dbaratbateladot@gmail.com

No. 122880, 12 Apr 2006 and cited (B). Between Marian and the baby, who is
cases therein, Art 805 and 806, Civil presumed to have died ahead? (1%)
Code).
SUGGESTED ANSWER:
(C). Was the disinheritance valid? (1%)
Marian is presumed to have died
SUGGESTED ANSWER: ahead of the baby. Art. 43 applies to
persons who are called to succeed
Yes, the disinheritance was valid. Art. each other. The proof of death must
919, par 7, Civil Code provides that be established by positive or
"when a child or descendant leads a circumstantial evidence derived from
dishonorable or disgraceful life, like facts. It can never be established
running off with a married man, there from mere inference. In the present
is sufficient cause for disinheritance." case, it is very clear that only Marian
and Pietro were hacked with bolos.
There was no showing that the baby
was also hacked to death. The baby's
Succession; Proof of Death between
death could have been due to lack of
persons called to succeed each other
nutrition.
(2008)

ALTERNATIVE ANSWER:
No. II. At age 18, Marian found out that
she was pregnant. She insured her own
The baby is presumed to have died
life and named her unborn child as her
ahead of Marian. Under Par. 5, rule
sole beneficiary. When she was already
131, Sec. 5
due to give birth, she and her boyfriend
0 of the Rules of Court, if one is under
Pietro, the father of her unboarn child,
15 or above 60 and the age of the
were kidnapped in a resort in Bataan
other is in between 15 and 60, the
where they were vacationing. The military
latter is presumed to have survived.
gave chase and after one week, they were
In the instant case, Marian was
found in an abandoned hut in Cavite.
already 18 when she found out that
Marian and Pietro were hacked with bolos.
she was pregnant. She could be of
Marian and the baby delivered were both
the same age or maybe 19 years of
found dead, with the baby's umbilical cord
age when she gave birth.
already cut. Pietro survived.

(C). Will Pietro, as surviving biological


father of the baby, be entitled to claim the

“Never Let The Odds Keep You From Pursuing What You Know In Your Heart You Were Meant To Do.”-Leroy Satchel Paige
Page 55 of 180
Civil Law Q&As (2007-2013) hectorchristopher@yahoo.com dbaratbateladot@gmail.com

proceeds of the life insurance on the life of


Marian? (2%) Marilyn is not entitled to a share in
the estate of Dr. Lopez. For purpose
SUGGESTED ANSWER: of succession, Dr. Lopez and his son
Roberto are presumed to have died at
Pietro, as the biological father of the
the same time, there being no
baby, shall be entitled to claim the
evidence to prove otherwise, and
proceeds of life insurance of the
there shall be no transmission of
Marian because he is a compulsory
rights from one to the other (Article
heir of his child.
43, NCC). Hence, Roberto, inherited
nothing from his father that Marilyn
would in turn inherit from Roberto

Succession; Rule on Survivorship .The children of Roberto, however,


(2009) will succeed their grandfather, Dr.
Lopez ,in representation of their
No. II. Dr. Lopez, a 70-year old widower, father Roberto and together Roberto
and his son Roberto both died in a fire that will receive 1/3 of the estate of Dr.
gutted their home while they were Lopez since their father Roberto was
sleeping in their air-conditioned rooms. one of the three children of Dr.
Roberto’s wife, Marilyn, and their two Lopez . Marilyn cannot represent her
children were spared because they were in husband Roberto because the right is
the province at the time. Dr. Lopez left an not given by the law to a surviving
estate worth P20M and a life insurance spouse.
policy in the amount of P1M with his three
children --- one of whom is Roberto --- as
As to the proceeds of the insurance
beneficiaries.
on the life of Dr. Lopez:

Marilyn is now claiming for herself and her


Since succession is not involved as
children her husband’s share in the estate
regards the insurance contract, the
left by Dr. Lopez, and her husband’s share
provisions of the Rules of Court (Rule
in the proceeds of Dr. Lopez’s life
131, Sec. 3 , [jj] [5] ) on survivorship
insurance policy. Rule on the validity of
shall apply. Under the Rules, Dr.
Marilyn’s claims with reasons. (4%)
Lopez, who was 70 years old, is
presumed to have died ahead of
SUGGESTED ANSWER :
Roberto who is presumably between
the ages 15 and 60. Having survived
As to the Estate of Dr. Lopez:
the insured, Roberto's right as a
beneficiary became vested
“Never Let The Odds Keep You From Pursuing What You Know In Your Heart You Were Meant To Do.”-Leroy Satchel Paige
Page 56 of 180
Civil Law Q&As (2007-2013) hectorchristopher@yahoo.com dbaratbateladot@gmail.com

upon the death of Dr. Lopez. When should be given effect must be
Roberto died after Dr. Lopez, his right denied. The said cancellation has
to receive the insurance became part revoked the entire will as nothing
of his hereditary estate, which in turn remains of the will after the name of
was inherited in equal shares by his Rosa was cancelled. Such cancellation
legal heirs, namely, his spouse and is valid revocation of the will and
children. Therefore, Roberto's does not require authentication by
children and his spouse are entitled the full signature of the testator to
to Roberto's one-third share in the be effective.
insurance proceeds.
However, if the cancellation of Rosa’s
name was not done by the testator
himself, such cancellation shall not be
effective and the will in its original
Wills; Holographic Wills; Insertions & tenor shall remain valid. The
Cancellations (2012) effectively of the holographic will
cannot be left to the mercy of
No.VII.a) Natividad’s holographic will, unscrupulous third parties.
which had only one (1) substantial
provision, as first written, named Rosa as The writing of Gregorio’s name as

her sole heir. However, when Gregorio sole heir was ineffective, even though

presented it for probate, it already written by the testator himself,

contained an alteration, naming Gregorio, because such is

instead of Rosa, as sole heir, but without an alteration that requires

authentication by Natividad’s signature. authentication by the full signature

Rosa opposes the probate alleging such of the testator to be valid and

lack of proper authentication. She claims effective. Not having an

that the unaltered form of the will should authenticated, the designation of

be given effect. Whose claim should be Gregorio as an heir was ineffective,

granted? Explain. (5%) (Kalaw v. Relova, G.R. No. L-40207,


Sept 28, 1984).
SUGGESTED ANSWER:

It depends. If the cancellation of


Rosa’s name in the will was done by Wills; Holographic Wills; Probate
(2009)
the testator himself, Rosa’s claimed
that the holographic will in its
No.VI. On December 1, 2000, Dr. Juanito
original tenor
Fuentes executed a holographic will,
wherein he gave nothing to his recognized
illegitimate son, Jay. Dr. Fuentes left for the
“Never Let The Odds Keep You From Pursuing What You Know In Your Heart You Were Meant To Do.”-Leroy Satchel Paige
Page 57 of 180
Civil Law Q&As (2007-2013) hectorchristopher@yahoo.com dbaratbateladot@gmail.com

United States, passed the New York court shall apply the New Civil Code
medical licensure examinations, resided in determining the formal validity of
therein, and became a naturalized the holographic will. The subsequent
American citizen. He died in New York in change in the citizenship of Dr.
2007. The laws of New York do not Fuentes did not affect the law
recognize holographic wills or compulsory governing the validity of his will.
heirs. Under the new Civil Code, which was
the law used by Dr. Fuentes, the law
(A). Can the holographic will of Dr. Fuentes enforced at the time of execution of
be admitted to probate in the Philippines? the will shall govern the formal
Why or why not? (3%) validity of the will (Art. 795, NCC).

SUGGESTED ANSWER: (B). Assuming that the will is probated in


Yes, the holographic will of Dr. the Philippines, can Jay validly insist that
Fuentes may be admitted to probate he be given his legitime? Why or why not?
in the Philippines because there is no (3%)
public policy violated by such
probate. The only issue at probate is SUGGESTED ANSWER:
the due execution of the will which No, Jay cannot insist because under
includes the formal validity of the New York law he is not a compulsory
will. As regards formal validity, the heir entitled to a legitime.
only issue the court will resolve at
probate is whether or not the will was The national law of the testator

executed in accordance with the form determines who his heirs are, the

prescribed by the law observed by order that they succeed, how much

the testator in the execution of his their successional rights are, and

will. For purposes of probate in the whether or not a testamentary

Philippines, an alien testator may disposition in his will is valid (Art 16,

observe the law of the place where NCC). Since, Dr. Fuentes was a US

the will was executed (Art 17, NCC), citizen, the laws of the New York

or the formalities of the law of the determines who his heirs are. And

place where he resides, or according since the New York law does not

to the formalities of the law of his recognize the concept of compulsory

own country, or in accordance with heirs, Jay is not a compulsory heir of

the Philippine Civil Code (Art. 816, Dr. Fuentes entitled to a legitime.

NCC). Since Dr. Fuentes executed his


will in accordance with the Philippine
law, the Philippine

“Never Let The Odds Keep You From Pursuing What You Know In Your Heart You Were Meant To Do.”-Leroy Satchel Paige
Page 58 of 180
Civil Law Q&As (2007-2013) hectorchristopher@yahoo.com dbaratbateladot@gmail.com

Wills; Joint Wills (2008) SUGGESTED ANSWER:

No. XI. John and Paula, British citizens at No. The testamentary dispositions
birth, acquired Philippine citizenship by are not valid because (a) omission of
naturalization after their marriage. During Mary, a legitimate child, is
their marriage the couple acquired tantamount to preterition which shall
substanial landholdings in London and in annul the institution of Peter and
Makati. Paula bore John three children, Paul as heirs (Art. 854, Civil Code);
Peter, Paul and Mary. In one of their trips and, (b) the disposition that Peter
to London, the couple executed a joint will and Paul could not dispose of nor
appointing each other as their heirs and divide the London estate for more
providing that upon the death of the than 20 years is void (Art. 870, Civil
survivor between them the entire estate Code).
would go to Peter and Paul only but the
two could not dispose of nor divide the
London estate as long as they live. John
Wills; Joint Wills; Probate (2012)
and Paul died tragically in the London
Subway terrorist attack in 2005. Peter and
No.VII.b) John Sagun and Maria Carla
Paul filed a petition for probate of their
Camua, British citizens at birth, acquired
parent's will before a Makati Regional Trial
Philippine citizenship by naturalization
Court.
after their marriage. During their
marriage,
(A). Should the will be admitted to
the couple acquired substantial
probate? (2%)
landholdings in London and in Makati.

SUGGESTED ANSWER: Maria begot three (3) children, Jorge,


Luisito, and Joshur. In one of their trips to
No. The will cannot be admitted to London, the couple executed a joint will
probate because a joint will is appointing each other as their heirs and
expressly prohibited under Art. 818 providing that upon the death of the
of the Civil Code. This provision survivor between them, the entire estate
applies John and Paula became would go to Jorge and Luisito only but the
Filipino citizens after their marriage. two (2) could not dispose of nor divide the
London estate as long as they live. John
(B). Are the testamentary dispositions and Maria died tragically in the London
valid? (2%) subway terrorist attack in 2005. Jorge and
Luisito filed a petition for probate of their
parents’ will before a Makati Regional Trial

“Never Let The Odds Keep You From Pursuing What You Know In Your Heart You Were Meant To Do.”-Leroy Satchel Paige
Page 59 of 180
Civil Law Q&As (2007-2013) hectorchristopher@yahoo.com dbaratbateladot@gmail.com

Court. Joshur vehemently objected SUGGESTED ANSWER:


because he was preterited.
Assuming the will of John and Maria was

23 Should the will be admitted to probate? valid, the testamentary prohibition on the

Explain. (2%) division of the London estate shall be valid


but only for 20 years. Under Arts 1083 and
SUGGESTED ANSWER: 494 of the NCC, a testamentary
disposition of the testator cannot forbid
No, the will should not be admitted to
the partition of all or part of the estate for
probate. Since the couples are both
a period longer than twenty (20) years.
Filipino citizens, Art 818 and 819 of
the NCC shall apply. Said articles
prohibits the execution of joint wills
and make them void, even though Wills; Prohibition to Partition of a Co-
authorized of the country where they Owned Property (2010)
were executed.
No.I. True or False.
Are the testamentary dispositions valid?
Explain. (2%) X, a widower, died leaving a will stating
that the house and lot where he lived
SUGGESTED ANSWER: cannot be partitioned for as long as the
youngest of his four children desires to
Since the joint will is void, all the
stay there. As coheirs and co-owners, the
testamentary disposition written
other three may demand partition
therein are also void. However, if the
anytime. (1%)
will is valid, the institutions of the
heirs shall be annulled because SUGGESTED ANSWER:
Joshur was preterited. He was
preterited because he will receive FALSE, The other three co – heirs may

nothing from the will, will receive not anytime demand the partition of

nothing in testacy, and the facts do the house and lot since it was

not show that he received anything expressly provided by the decedent

as an advance on his inheritance. He in his will that the same cannot be

was totally excluded from the partitioned while his youngest child

inheritance of his parents. desires to stay there. Article 1083 of


the New Civil Code allows a decedent
Is the testamentary prohibition against the to prohibit, by will, the partition of a
division of the London estate valid? property and his estate for a period
Explain. (1%) not longer than 20 years no

“Never Let The Odds Keep You From Pursuing What You Know In Your Heart You Were Meant To Do.”-Leroy Satchel Paige
Page 60 of 180
Civil Law Q&As (2007-2013) hectorchristopher@yahoo.com dbaratbateladot@gmail.com

matter what his reason maybe. (B). Act as a witness to a will? (1%)
Hence, the three co-heir cannot
demand its partition at anytime but SUGGESTED ANSWER:

only after 20 years from the death of


Stevie cannot be a witness to a will.
their father. Even if the deceased
Art. 820 of the Civil Code provides
parent did not leave a will, if the
that "any person of sound mind and
house and lot constituted their family
of the age of eighteen years or more,
home, Article 159 of the Family Code
and not blind, deaf or dumb, and able
prohibits its partition for a period of
to read and write, may be a witness
ten (10) years, or for as long as there
to the execution of a will.
is a minor beneficiary living in the
family home.
(C). In either of the above instances, must
the will be read to him? (1%)

SUGGESTED ANSWER:
Wills; Notarial Wills; Blind Testator;
Requisites (2008)
If Stevie makes a will, the will must
be read to him twice, once by one of
No. XIV. Stevie was born blind. He went to
the subscribing witnesses, and again,
school for the blind, and learned to read in
by the notary public before whom the
Baille Language. He Speaks English
will is acknowledged (Art. 808, Civil
fluently. Can he:
Code).

(A). Make a will? (1%)

SUGGESTED ANSWER:
Wills; Testamentary Disposition;

Assuming that he is of legal age (Art. Period to Prohibit Partition (2008)

797, Civil Code) and of sound mind at


No. XI. John and Paula, British citizens at
the time of execution of the will (Art.
birth, acquired Philippine citizenship by
798, Civil Code), Stevie, a blind
naturalization after their marriage. During
person, can make a notarial will,
their marriage the couple acquired
subject to compliance with the "two-
substanial landholdings in London and in
reading rule" (Art. 808, Civil Code)
Makati. Paula bore John three children,
and the provisions of Arts. 804, 805
Peter, Paul and Mary. In one of their trips
and 806 of the Civil Code.
to London, the couple executed a joint will
appointing each other as their heirs and

“Never Let The Odds Keep You From Pursuing What You Know In Your Heart You Were Meant To Do.”-Leroy Satchel Paige
Page 61 of 180
Civil Law Q&As (2007-2013) hectorchristopher@yahoo.com dbaratbateladot@gmail.com

providing that upon the death of the that she can sign her full name later. While
survivor between them the entire estate the will was being signed, Roberta
would go to Peter and Paul only but the experienced a stomach ache and kept
two could not dispose of nor divide the going to the restroom for long periods of
London estate as long as they live. John time. Hannah, while waiting for her turn to
and Paul died tragically in the London sign the will, was reading the 7 th Harry
Subway terrorist attack in 2005. Peter and Potter book on the couch, beside the table
Paul filed a petition for probate of their on which everyone was signing. Benjamin,
parent's will before a Makati Regional Trial aside from witnessing the will, also offered
Court. to notarize it. A week after, Clara was run
over by a drunk driver while crossing the
(C). Is the testamentary prohibition street in Greenbelt.
against the division of the London estate
valid? (2%) May the will of Clara be admitted to
probate? Give your reasons briefly. (10%)
SUGGESTED ANSWER:
SUGGESTED ANSWER:
No. the testamentary prohibition
against the division of the London Probate should be denied. The
estate is void (Art. 870, Civil Code). A requirement that the testator and at
testator, however, may prohibit least three (3) witnesses must sign
partition for a period which shall not all in the "presence" of one another
exceed twenty was not complied with. Benjamin who
years (Art. 870 in relation to Art. 494, notarized the will is disqualified as a
par 3, Civil Code). witness, hence he cannot be counted
as one of the three witnesses (Cruz v.
Villasor, 54 SCRA 31, 1973). The
testatrix and the other witnesses
Wills; Witnesses to a Will, Presence
signed the will not in the presence of
required; Thumbmark as Signature
Roberta because she was in the
(2007)
restroom for extended periods of
time. Inside the restroom, Roberta
No.VI. Clara, thinking of her mortality,
could not have possibly seen the
drafted a will and asked Roberta, Hannah,
testatrix and the other witnesses sign
Luisa and Benjamin to be witnesses.
the will by merely casting her eyes in
During the day of signing of her will, Clara
the proper direction (Jaboneta v.
fell down the stairs and broke her arms.
Gustilo, 5 Phil 541, 1906; Nera v.
Coming from the hospital, Clara insisted
Rimando, 18 Phil
on signing her will by thumb mark and
said
“Never Let The Odds Keep You From Pursuing What You Know In Your Heart You Were Meant To Do.”-Leroy Satchel Paige
Page 62 of 180
Civil Law Q&As (2007-2013) hectorchristopher@yahoo.com dbaratbateladot@gmail.com

451, 1914). Therefore, the testatrix Because the Picasso painting reminded
signed the will in the presence of only Angie of him, Brad in his will bequeathed
two witnesses, and only two the painting to Angie. Brad died in 1995.
witnesses signed the will in the Saddened by Brad's death, Jennifer asked
presence of the testatrix and of one for the Picasso painting as a remembrance
another. of him. Angie refused and claimed that
Brad, in his will, bequeathed the painting
It is to be noted, however, that the to her. Is Angie correct? Why or why not?
thumb mark intended by the testator (10%)
to be his signature in executing his
last will and testament is valid (Payad SUGGESTED ANSWER:
v. Tolentino, 62 Phil 848, 1936; Matias
v. Salud, L-104 Phil 1046, 23 June, NO. Angie is not correct. The Picasso

1958). The problem, however, states painting is not given or donated by

that Clara "said that she can sign her Jennifer to Brad. She merely "placed

full name later;" Hence, she did not it in his bedroom." Hence, she is still

consider her thumb mark as her the owner of the painting. Not being

"complete" signature, and intended the owner of the Picasso painting,

further action on her part. The Brad cannot validly bequeath the

testatrix and the other witness same to Angie (Art. 930, NCC). Even

signed the will in the presence of assuming that the painting was

Hannah, because she was aware of impliedly given or donated by

her function and role as witness and Jennifer to Brad, the donation is

was in a position to see the testatrix nevertheless void for not being in

and the other witnesses sign by writing. The Picasso painting must be

merely casting her eyes in the proper worth more than 5,000 pesos. Under

direction. Art. 748, NCC, the donation and


acceptance of a movable worth more
Donation
than 5,000 pesos must be in writing,
otherwise the donation is void. The
Donations; Formalities; In Writing
(2007) donation being void, Jennifer
remained the owner of the Picasso
No. VIII. In 1986, Jennifer and Brad were painting and Brad could not have
madly in love. In 1989, because a certain validly disposed of said painting in
Picasso painting reminded Brad of her, favor of Angie in his will.
Jennifer acquired it and placed it in his
bedroom. In 1990, Brad and Jennifer broke ALTERNATIVE ANSWER:
up. While Brad was mending his broken
heart, he met Angie and fell in love.
“Never Let The Odds Keep You From Pursuing What You Know In Your Heart You Were Meant To Do.”-Leroy Satchel Paige
Page 63 of 180
Civil Law Q&As (2007-2013) hectorchristopher@yahoo.com dbaratbateladot@gmail.com

YES. Angie is correct. Even assuming illegal and impossible donations


that there was void donation because imposed in an onerous donation shall
the same was not in writing, Brad annul the donation (Art. 1183, NCC).
was in uninterrupted possession of This is so, because onerous donations
the Picasso painting from 1989 to are governed by the law on contracts
1995, lasting for six (6) years prior to (Art. 733, NCC).
his death. Brad has already acquired
ownership of the
painting through acquisitive
Donation; Inter Vivos (2013)
prescription. Under Art. 1132, NCC,
ownership of movables prescribes
No.V. Josefa executed a deed of donation
through continuous possession for
covering a one-hectare rice land in favor
four
of her daughter, Jennifer. The deed
years in good faith and for eight (8)
specifically provides that:
years without need of other
conditions. A void donation may be "For and in consideration of her
the basis of possession in the concept love and service Jennifer has
of owner and of just title for purposes shown and given to me, I hereby
of acquisitive prescription. freely, voluntarily and irrevocably
donate to her my one-hectare rice
land covered by TCT No. 11550,
located in San Fernando,
Donations; Illegal & Impossible
Pampanga. This donation shall take
Conditions (2007)
effect upon my death."

No.I. Distinguish the following concepts:


The deed also contained Jennifer's signed

(B). Illegal and impossible conditions in a acceptance, and an attached notarized

simple donation v. illegal and impossible declaration by Josefa and Jennifer that the

conditions in an onerous donation. (5%) land will remain in Josefa's possession and
cannot be alienated, encumbered, sold or
SUGGESTED ANSWER: disposed of while Josefa is still alive.

Illegal and impossible conditions in a Advise Jennifer on whether the deed is a


simple donation are considered as donation inter vivos or mortis causa and
not written. Such conditions, shall explain the reasons supporting your
therefore, be disregarded but the advice. (8%)
donation remains valid (Art. 727,
NCC). On the other hand,

“Never Let The Odds Keep You From Pursuing What You Know In Your Heart You Were Meant To Do.”-Leroy Satchel Paige
Page 64 of 180
Civil Law Q&As (2007-2013) hectorchristopher@yahoo.com dbaratbateladot@gmail.com

SUGGESTED ANSWER: same should be harmonized with its


express irrevocability (Austria-Magat
The donation is a donation inter v. CA, G.R. No. 106755, Feb 1, 2002).
vivos.

ALTERNATIVE ANSWER:
When the donor intends that the
donation shall take effect during the
The donation is donation mortis
lifetime of the donor, though the causa.
property shall not be delivered till
after the donor’s death, this shall be The deed clearly states that the

a donation inter vivos (Art. 729, Civil donation shall take effect upon the

Code). death of the donor, Josefa. The donor,


moreover, retained ownership of the
The Civil Code prefers inter vivos subject property as it was declared
transmissions. Moreover, mortis causa that the property cannot be
donations should follow the formalities alienated, encumbered, sold or
of a will (Art. 728, Civil Code). Here disposed of while the donor is still
there is no showing that such alive.
formalities were followed. Thus, it is
favorable to Jennifer that the deed is a As the donation is in the nature of a

donation inter vivos. mortis causa disposition, the


formalities of a will should have been
Furthermore, what is most significant complied with under Art. 728 of the
in determining the type of donation is Civil Code, otherwise, the donation is
the absence of stipulation that the void and would produce no effect
donor could revoke the donation; on (The National Treasure of the
the contrary, the deeds expressly Philippines v. Vda. de Meimban, G.R.
declare them to be “irrevocable,” a No. L-61023, Aug 22, 1984).
quality absolutely incompatible with
the idea of conveyances mortis causa Property
where revocability is the essence of
the act, to the extent that a testator Accretion; Alluvium (2008)
cannot lawfully waive or restrict his
right of revocation. The provisions of No. IX. The properties of Jessica and Jenny,
the deed of donation which state that who are neighbors, lie along the banks of
the same will only take effect upon the Marikina River. At certain times of the
the death of the donor and that there year, the river would swell and as the
is a prohibition to alienate, encumber, water recedes, soil, rocks and other
dispose, or sell the materials are

“Never Let The Odds Keep You From Pursuing What You Know In Your Heart You Were Meant To Do.”-Leroy Satchel Paige
Page 65 of 180
Civil Law Q&As (2007-2013) hectorchristopher@yahoo.com dbaratbateladot@gmail.com

deposited on Jessica's and Jenny's land but is also the consequences of


properties. This pattern of the river the direct and deliberate intervention
swelling, receding and depositing soil and of man, it is man-made accretion and
other materials being deposited on the a part of the public domain (Tiongco
neighbors' properties have gone on for v. Director of Lands, 16 C.A. Rep 211,
many years. Knowing his pattern, Jessica cited in Nazareno v. C.A., G.R. No.
constructed a concrete barrier about 2 98045, 26 June 1996). Thus, Jessica
meters from her property line and cannot legally claim ownership of the
extending towards the river, so that when additional 2 meters of land along her
the water recedes, soil and other materials property because she constructed a
are trapped within this barrier. After concrete barrier about 2 meters from
several years, the area between Jessica's her property causing deposits of soil
property line to the concrete barrier was and other materials when the water
completely filled with soil, effectively recedes. In other words, the increase
increasing Jessica's property by 2 meters. in her property was not caused by
Jenny's property, where no barrier was nature but was man-made.
constructed, also increased by one meter
along the side of the river. (B). If Jessica's and Jenny's properties are
registered, will the benefit of such
(A). Can Jessica and Jenny legally claim registration extend to the increased area
ownership over the additional 2 meters of their properties? (2%)
and one meter, respectively, of land
deposited along their properties?(2%) SUGGESTED ANSWER:

SUGGESTED ANSWER: If the properties of Jessica and Jenny


are registered, the benefit of such
Only Jenny can claim ownership over registration does not extend to the
the additional one meter of land increased area of their properties.
deposited along her property. Art. Accretion does not automatically
457 of the Civil Code provides that become registered land because
"to the owners of lands adjoining the there is a specific technical
banks of river belong the accretion description of the lot in its Torrens
which they gradually receive from the title. There must be a separate
effects of the current of the water." application for registration of the
Where the land is not formed solely alluvial deposits under the Torrens
by the natural effect of the water System (Grande v. CA, G.R. No. L-
current of the river bordering 17652, 30 June, 1962).

“Never Let The Odds Keep You From Pursuing What You Know In Your Heart You Were Meant To Do.”-Leroy Satchel Paige
Page 66 of 180
Civil Law Q&As (2007-2013) hectorchristopher@yahoo.com dbaratbateladot@gmail.com

(C). Assume the two properties are on a Ulpiano built three huts on this additional
cliff adjoining the shore of Laguna Lake. area, where he and his two married
Jessica and Jenny had a hotel built on the children live. On this same area, Ulpiano
properties. They had the erath and rocks and his family planted peanuts, monggo
excavated from the properties dumped on beans and vegetables. Ulpiano also
the adjoining shore, giving rise to a new regularly paid taxes on the land, as shown
patch of dry land. Can they validly lay by tax declarations, for over thirty years.
claim to the patch of land? (2%)
When Marciano learned of the increase in
SUGGESTED ANSWER: the size of the land, he ordered Ulpiano to
demolish the huts, and demanded that he
No. Jessica and Jenny cannot validly be paid his share in the proceeds of the
lay claim to the patch of land because harvest. Marciano claims that under the
in order to acquire land by accretion, Civil Code, the alluvium belongs to him as
there should be a natural and actual a registered riparian owner to whose land
continuity of the accretion to the land the accretion attaches, and that his right
of the riparian owner caused by is enforceable against the whole world.
natural ebb and flow of the current of
the river (Delgado v. Samonte, CA- (A). Is Marciano correct? Explain. (3%)
G.R. No. 34979-R, 10 Aug 1966).
SUGGESTED ANSWER:
Marciano’s contention is correct.
Since that accretion was deposited on
Accretion; Rights of the Riparian his land by the action of the waters of
Owner (2009) the river and he did not construct any
structure to increase the deposition
No.XVI. Marciano is the owner of a parcel
of soil and silt, Marciano
of land through which a river runs out into
automatically owns the accretion. His
the sea. The land had been brought under
real right of ownership is enforceable
the Torrens System, and is cultivated by
against the whole world including
Ulpiano and his family as farmworkers
Ulpiano and his two married children.
therein. Over the years, the river has
Although Marciano’s land is
brought silt and sediment from its sources
registered, the three (3) hectares
up in the mountains and forests so that
land deposited through accretion was
gradually the land owned by Marciano
not
increased in area by three hectares.
automatically registered. As an
unregistered land, it is subject to
acquisitive prescription by third
persons.
“Never Let The Odds Keep You From Pursuing What You Know In Your Heart You Were Meant To Do.”-Leroy Satchel Paige
Page 67 of 180
Civil Law Q&As (2007-2013) hectorchristopher@yahoo.com dbaratbateladot@gmail.com

production, gathering and


Although Ulpiano and his children live preservation of the fruits (Art 443,
in the three (3) hectare unregistered NCC).
land owned by Marciano, they are
farm workers; therefore, they are He may also ask for reimbursement of
possessors not in the concept of the taxes he has paid, as these are
owners but in the concept of mere charges on the land owned by
holders. Even if they possess the land Marciano. This obligation is based on
for more than 30 years, they cannot a quasi-contract (Art 2175, NCC).
become the owners thereof
through extraordinary acquisitive
prescription, because the law
Builder; Good Faith; Requisites (2013)
requires possession in the concept of
the owner. Payment of taxes and tax
No.VIII. Ciriaco Realty Corporation (CRC)
declaration are not enough to make
sold to the spouses Del a Cruz a500-
their possession one in the concept of
square meter land (Lot A) in Paranaque.
owner. They must repudiate the
The land now has a fair market value of
possession in the concept of holder
Pl,200,000. CRC likewise sold to the
by executing unequivocal acts of
spouses Rodriguez, a 700-square meter
repudiation amounting to ouster of
land (Lot B) which is adjacent to Lot A. Lot
Marciano, known to Marciano and
B has a present fair market value of
must be proven by clear and
P1,500,000.
convincing evidence. Only then would
his possession become adverse. The spouses Dela Cruz constructed a
house on Lot B, relying on their
(B). What rights, if any, does Ulpiano have
presentation of the CRC sales agent that it
against Marciano? Explain. (3%)
is the property they purchased. Only upon
the completion of their house did the
SUGGESTED ANSWER:
spouses Dela Cruz discovered that they
Although Ulpiano is a possessor in
had built on Lot B owned by the spouses
bad faith, because he knew he does
Rodriguez, not on Lot A that they
not own the land, he will lose the
purchased. They spent P 1 000,000 for the
three huts he built in bad faith and
house.
make an accounting of the fruits he
has gathered, he has the right to
As their lawyer, advise the spouses Dela
deduct from the value of the fruits
Cruz on their rights and obligations under
the expenses for
the given circumstances, and the
recourses
“Never Let The Odds Keep You From Pursuing What You Know In Your Heart You Were Meant To Do.”-Leroy Satchel Paige
Page 68 of 180
Civil Law Q&As (2007-2013) hectorchristopher@yahoo.com dbaratbateladot@gmail.com

and options open to them to protect their However, the builder cannot be
interests. (8%) obliged to buy the land if its value is
considerable more than that of the
SUGGESTED ANSWER: building.. In such case, he shall pay
reasonable rent of the owner of the
Based on the fact as stated, the
land does not choose to appropriate
spouses Dela Cruz as builders and the
the building or trees after proper
spouses Rodriguez as land owners,
indemnity (Art 448, Civil Code).
are both in good faith. The spouses
Dela Cruz are builder in good faith The house constructed by the
because before constructing the spouses Dela Cruz is considered as a
house they exercised due diligence useful expense, since it increased the
by asking the Agent of CRC the value of the lot. As such, should the
location of the lot A, and they relied spouses Rodriguez decides to
on the information given by the agent appropriate the house, the spouses
who is presumed to know the identity Dela Cruz are entitled to the right of
of the lot purchased by the Dela Cruz retention pending reimbursement of
spouses (Pleasantville v. CA, 253 the expenses they incurred or the
SCRA 10, 1996). On the other hand, increase in value which the thing may
there is no showing that the land have acquired by reason of the
owners, spouse Rodriguez acted in improvement (Art 546, Civil Code).
bad faith. The facts do not show that Thus, the spouses Dela Cruz may
the building was done with their demand P1,000,000.00 as payment of
knowledge and without opposition on the expenses in building the house or
their part (Art 453, Civil Code). The increase in value of the land because
good faith is always presumed (Art. of the house as a useful
527, Civil Code). improvement, as may be determined
by the court form the evidence
The owner of the land on which
presented during the trial (Depra v.
anything has been built, sown, or
Dumlao, 136 SCRA 475, 1985;
planted in good faith shall have the
Technogas Phils v. CA, 268 SCRA 5,
right:
1997).

to appropriate as his own the works


after payment of the indemnity
provided for in Art 546 and 548, or

to oblige the one who built to pay the


price of the land.

“Never Let The Odds Keep You From Pursuing What You Know In Your Heart You Were Meant To Do.”-Leroy Satchel Paige
Page 69 of 180
Civil Law Q&As (2007-2013) hectorchristopher@yahoo.com dbaratbateladot@gmail.com

Easement; Prescription; Acquisitive In 2006, Brand0 fenced off his property,


Prescription (2009) thereby blocking Andres' access to the
national highway. Andres demanded that
No. XI. TRUE or FALSE. Answer TRUE if part of the fence be removed to maintain
the statement is true, or FALSE if the his old access route to the highway
statement is false. Explain your answer in (pathway A), but Brando refused, claiming
not more than two (2) sentences. that there was another available pathway
(pathway B) for ingress and egress to the
(C). Acquisitive prescription of a negative
highway. Andres countered that pathway B
easement runs from the time the owner of
has defects, is circuitous, and is extremely
the dominant estate forbids, in a notarized
inconvenient to use.
document, the owner of the servient
estate from executing an act which would To settle their dispute, Andres and Brando
be lawful without the easement. (1%) hired Damian, a geodetic and civil
engineer, to survey and examine the two
SUGGESTED ANSWER:
pathways and the surrounding areas, and
True. In negative easements,
to determine the shortest and the least
acquisitive prescription runs from the
prejudicial way through the servient
moment the owner of the dominant
estates. After the survey, the engineer
estate forbade, by an instrument
concluded that pathway B is the longer
acknowledged before notary public,
route and will need improvements and
the owner of the servient estate from
repairs, but will not significantly affect the
executing an act which would be
use of Brando's property. On the other
lawful without the easement (Art.
hand, pathway A that had long been in
621, NCC).
place, is the shorter route but would
significantly affect the use of Brando's
property.

Easement; Right of Way (2013)


In light of the engineer's findings and the

No.VII.In 2005, Andres built a residential circumstances of the case, resolve the

house on a lot whose only access to the parties' right of way dispute. (6%)

national highway was a pathway crossing


SUGGESTED ANSWER:
Brando's property. Andres and others have
been using this pathway (pathway A) since
1980.

“Never Let The Odds Keep You From Pursuing What You Know In Your Heart You Were Meant To Do.”-Leroy Satchel Paige
Page 70 of 180
Civil Law Q&As (2007-2013) hectorchristopher@yahoo.com dbaratbateladot@gmail.com

Andres is not entitled to the (Pathway B). Second, the right of way
easement of right of way for Pathway obtained (Pathway A) is not the least
A. Pathway B must be used. prejudicial to Brando’s property, as
evidence by the reports of the
The owner of a dominant estate may geodetic and civil engineer.
validly obtain a compulsory right of
way only after he has established the When there is already an existing
existence of four requisites, to wit: adequate outlet from the dominant
estate to the public highway, even if
The (dominant) estate is surrounded the said outlet, for one reason or
by other immovables and is without another, be inconvenient, the need to
adequate outlet to a public highway; open up another servitude is entirely
unjustified (Costabella Corporation v.
After payment of the proper
CA, G.R. No. 80511, Jan 25, 1991).
indemnity;
The rule that the easement of right of
way shall be established at the point
The isolation was not due to the
least prejudicial to the servient
proprietor’s own acts; and
estate is controlling (Quimen v.

The right of way claimed is at a point Quimen and CA, G.R. No. 112331, May

least prejudicial to the servient 29, 1996).

estate, and insofar as consistent with


(Note: It is not clear from the problem if
this rule, where the distance from the
there exists an easement in favor of the
dominant estate to the public
lot belonging to Andres and if Brando’s lot
highway maybe the shortest (Art 650,
is burdened as a servient estate by a right
civil Code).
of way as a servient estate. If there is

However, the Supreme Court has such an easement burdening Brando’s lot,

consistently ruled that in case both was it created as legal easement or as a

criteria cannot be complied with, the voluntary easement. If the used pathway

right of way shall be established at was only a tolerance, then Brando may

the point least prejudicial to the close it. Andres must ask for the

servient estate. constitution of a legal easement through


Brando’s lot by proving the four requisites
The first and fourth requisites are not required by Art 649 and 65, Civil Code).
complied with. First, there is another
available outlet to the national
highway

“Never Let The Odds Keep You From Pursuing What You Know In Your Heart You Were Meant To Do.”-Leroy Satchel Paige
Page 71 of 180
Civil Law Q&As (2007-2013) hectorchristopher@yahoo.com dbaratbateladot@gmail.com

Easement; Right of Way (2010) easement or servitude, even if the


deed of sale is silent on the matter.
No.XIII. Franz was the owner of Lot E which
was surrounded by four (4) lots one of The vendee of the property in which a
which – Lot C – he also owned. He servitude or easement exists cannot
promised Ava that if she bought Lot E, he close or put obstructions thereon to
would give her a right of way in Lot C. prevent the dominant estate from
using it.
Convinced, Ava bought Lot E and, as
promised, Franz gave her a right of way in
Ava’s working abroad for more than
Lot C.
ten (10) years should not be
construed as non-user, because it
Ava cultivated Lot E and used the right of
cannot be implied from the fact that
way granted by Franz.
she or those she left behind to

Ava later found gainful employment cultivate the lot no longer use the

abroad. On her return after more than 10 right of way.

years, the right of way was no longer


available to her because Franz had in the Note: Since a right of way is a

meantime sold Lot C to Julia who had it discontinuous easement, the period

fenced. of ten years of non-user, shall be


computed from the day it ceased to
(A). Does Ava have a right to demand from be used under Act 6341 (2) CC.
Julia the activation of her right of way?
Renunciation or waiver of an
Explain. (2.5%)
easement must be specific, clear,

SUGGESTED ANSWER: express and made in a public


instrument in accordance of Art 1358
Yes. Ava has the right to demand of the New Civil Code.
from Julia the activation of the right ALTERNATIVE ANSWER:
of way, for the following reasons:
Yes. Ava has the right to demand
The easement of the right of way is a from Julia the activation of the right
real right which attaches to, and is of way. A voluntary easement of right
inseperable from, the estate to which of way, like any other contract, could
it belongs. be extinguished only by mutual
agreement or by renunciation of the
The sale of the property includes the owner of the dominant estate. Also,
like any other

“Never Let The Odds Keep You From Pursuing What You Know In Your Heart You Were Meant To Do.”-Leroy Satchel Paige
Page 72 of 180
Civil Law Q&As (2007-2013) hectorchristopher@yahoo.com dbaratbateladot@gmail.com

contract, an easement is generally her in lot C if Ava purchase lot E. The


effective between parties, their heirs promise was not reduced to writing
and assignees, except in case where (Obra v. Baldria, 529 SCRA 621
the rights and obligations arising [2007]). Hence, it was not or could
from the contract are not not have been registered as to warn
transmissible by their nature, or by buyers of lot C about the existence of
stipulations or by provision of law the easement on the property. Not
(Unisource Commercial v. Chung, 593 having been annotated on the TCT to
SCRA 530 [2009]). lot C, the buyer acquired lot C free
from such right of way granted to
(B). Assuming Ava opts to demand a right Ava.
of way from any of the owners of Lots A, B,
and D, can she do that? Explain. (2.5%)

SUGGESTED ANSWER: Hidden Treasure (2008)

Yes. Ava has the option to demand a No. VIII. Adam, a building contractor, was
right of way on any of the remaining engaged by Blas to construct a house on a
lots of Franz more so after Franz sold lot which he (Blas) owns. While digging on
lot C to Julia. The essential elements the lot in order to lay down the foudation
of a legal right of way under Art 649 of the house, Adam hit a very hard object.
and 650 of the New Civil Code are It turned out to be the vault of the old
complied with. Banco de las Islas Filipinas. Using a
detonation device, Adam was able to open
ALTERNATIVE ANSWER:
the vault containing old notes and coins
Yes. Ava has the option to demand a which were in circulation during the
right of way from the other lots. The Spanish era. While the notes and coins are
law provides that whenever a piece of no longer legal tender, they were valued
land acquired by sale, exchange or at P100 million because of their historical
partition is surrounded by other value and the coins silver nickel content.
estates of the vendor, exchanger, or The following filed legal claims over the
co-owner, he shall be obliged to grant notes and coins:
a right of way without indemnity (Art
652, NCC). (i). Adam, as finder;

ALTERNATIVE ANSWER: (ii). Blas, as owner of the property where


they were found;
No. There was merely a promise to
Ava that a right of way shall be
granted to
“Never Let The Odds Keep You From Pursuing What You Know In Your Heart You Were Meant To Do.”-Leroy Satchel Paige
Page 73 of 180
Civil Law Q&As (2007-2013) hectorchristopher@yahoo.com dbaratbateladot@gmail.com

(iii). Bank of the Philippine Islands, as present case, Adam, as finder, and
successor-in-interest of the owner of the Blas, as owner of the land, are
vault; and entitled to share 50-50 in the
treasure. The government can only
(iv). The Philippine Government because claim if it can establish that the notes
of their historical value. and coins are of interest to science or
the arts, then it must pay just price of
(A). Who owns the notes and coins? (4%)
the things found, to be divided
equally between Adam and Blas (Art.
SUGGESTED ANSWER:
438, Civil Code).

The notes and coins are no longer


(B). Assuming that either or both Adam
owned by the Bank of the Philippine
and Blas are adjudged as owners, will the
Islands, which has either lost or
notes and coins be deemed part of their
abandoned the vault and its contents,
absolute community or conjugal
and it has not taken any effort to
partnership of gains with their respective
search, locate or recover the vault. In
spouses? (2%)
any case, since the vault is in actual
possession of Adam, BPI may
SUGGESTED ANSWER:
attempt, in a judicial action to
recover, to rebut the presumption of If either or both Adam and Blas are
ownership in favor of Adam and Blas adjudged as owners, the notes and
(Art. 433, Civil Code). Hidden coins shall be deemed part of their
treasure is any hidden and unknown absolute community or conjugal
deposit of money, jewelry, or other partnership of gains with their
precious objects, the lawful respective spouses (Art. 117, par 4,
ownership of which does not appear. FC).
Given the age and importance of the
items found, it would be safe to
consider the vault, notes and coins
abandoned by BPI and its Mortgage; Public or Private
predecessor (Art. 439, Civil Code). It Instrument (2013)
belongs to the owner of the land on
No.VI. Lito obtained a loan of P1,000,000
which it is found. When the discovery
from Ferdie, payable within one year. To
is made on the property of another,
secure payment, Lito executed a chattel
or of the State and by chance, one-
mortgage on a Toyota Avanza and a real
half of it shall belong to the finder
estate mortgage on a 200-square meter
who is not a trespasser (Art. 438,
piece of property.
Civil Code). In the

“Never Let The Odds Keep You From Pursuing What You Know In Your Heart You Were Meant To Do.”-Leroy Satchel Paige
Page 74 of 180
Civil Law Q&As (2007-2013) hectorchristopher@yahoo.com dbaratbateladot@gmail.com

Would it be legally significant - from the exceeds Five Hundred pesos


point of view of validity and enforceability (P500.00) must appear in writing,
- if the loan and the mortgages were in even in private one. However, the
public or private instruments? (6%) requirement is not for validity of the
contract, but only for its greater
SUGGESTED ANSWER: efficacy.

From the point of view of validity and With regard to the chattel mortgage,
enforceability, there would be legal Art. 1508, the Chattel Mortgage Law,
significance if the mortgage was in a requires an affidavit of good faith
public or private instrument. As for stating that the chattel mortgage is
the loan, there is no legal significance supposed to stand as security of the
except of interest were charged on loan; thus, for the validity of the
the loan, in which case, the charging chattel mortgage, it must be in a
of interest must be in writing. public document and recorded in the
Chattel Mortgage Register in the
A contract of loan is a real contract
Register of Deeds. A real estate
and is perfected upon delivery of the
mortgage, under the provisions of
object of the obligation (Art 1934,
Art. 2125 of the Civil Code, requires
Civil Code). Thus, a contract of loan is
that in order that a mortgage may be
valid and enforceable even if it is
validly constituted the document in
neither in a private nor in a public
which it appears be recorded. If the
document.
instrument is not recorded, the
mortgage is nevertheless valid and
As a rule, contracts shall be
binding between the parties. Hence,
obligatory in whatever form they may
for validity of both chattel and real
have been entered into provided all
estate mortgages, they must appear
the essential requisites for their
in a public instrument. But the
validity are present. With regards to
purpose of enforceability, it is
its enforceability, a contact of loan is
submitted that the form of the
not among those enumerated under
contract, whether in a public or
Art. 1403 (2) of the Civil Code, which
private document, would be
are covered by the Statute of Frauds.
immaterial (Mobil Oil v. Diocaresa, 29
SCRA 656, 1969).
It is important to note that under Art.
1358 of the Civil Code, all the other
Also, under Art 1358, acts and
contracts where the amount involved
contracts which have for their object
the creation or transmission of real
rights over immovable property must
be in a public
“Never Let The Odds Keep You From Pursuing What You Know In Your Heart You Were Meant To Do.”-Leroy Satchel Paige
Page 75 of 180
Civil Law Q&As (2007-2013) hectorchristopher@yahoo.com dbaratbateladot@gmail.com

document for greater efficacy and a right. Possession may be the real
real estate mortgage is a real right right of possession or jus
over immovable property. possessiones or it can be merely the
right to possess or jus possedendi,
which are among the basic rights of
ownership. If the real right of
Occupation vs. Possession (2007)
possession is possession in the
concept of owner, but subject to
No.I. Distinguish the following concepts:
certain limitations, it may ripen into

(A). Occupation v. possession. (5%) full ownership of the thing or


property right
SUGGESTED ANSWER: through acquisitive prescription
depending on whether it is a case of
Occupation is an original mode of ordinary or extraordinary prescription
acquiring ownership (Art. 712, NCC). and whether the property is movable
Things appropriable by nature which or immovable.
are without an owner, such as
animals that are the object of hunting
and fishing, Ownership; Co-Ownership (2009)
hidden treasure and abandoned
movables, are acquired by occupation No. XI. TRUE or FALSE. Answer TRUE if
(Art. 713, NCC). However, ownership the statement is true, or FALSE if the
of a piece of land cannot be acquired statement is false. Explain your answer in
by occupation (Art. 714, NCC). not more than two (2) sentences.

ALTERNATIVE ANSWER: (D). The renunciation by a co-owner of his


undivided share in the co-owned property
Occupation is a mode of acquiring in lieu of the performance of his obligation
dominion by the seizure of corporeal to contribute to taxes and expenses for
things which have no owner, with the the
intention of acquiring the ownership preservation of the property constitutes
thereof. It is an original mode of dacion en pago. (1%)
acquiring ownership upon seizure of
a res nullius by the occupant who has SUGGESTED ANSWER:
the intention to become the owner True, Under the Civil Code, a co-
thereof. Possession, on the other owner may renounce his share in the
hand, is the holding of the thing or an co-owned property in lieu of paying
enjoyment of a for his share in the taxes and
expenses for the preservation of the
co-owned property.
“Never Let The Odds Keep You From Pursuing What You Know In Your Heart You Were Meant To Do.”-Leroy Satchel Paige
Page 76 of 180
Civil Law Q&As (2007-2013) hectorchristopher@yahoo.com dbaratbateladot@gmail.com

In effect, there is dacion en pago SUGGESTED ANSWER:


because the co-owner is discharging
his monetary obligation by paying it Yes, Cathy can lawfully ask for the

with his non-monetary interest in the demolition of Bobby's house. Where

co-owned property. The fact that he there are two or more heirs, the

is giving up his entire interest simply whole estate of the decedent, is,

means that he is accepting the value before partition, owned in common

of his interest as equivalent to his by such heirs, subject to the payment

share in the taxes and expenses of of debts of the deceased (Art. 1078,

preservation. Civil Code), Under the rules on co-


ownership, "none of the co-owners
shall, without the consent of the

Ownership; Co-Ownership (2008) others make alterations in the thing


owned in common, even though
No. VI. Alex died without a will, leaving benefits for all would results
only an undeveloped and untitled lot in therefrom." In Cruz v. Catapang, G.R.
Tagiug City. He is survived by his wife and No. 164110, 12 Feb., 2008, the Court
4 children. His wife told the children that held that "alterations include any act
she is waiving her share in the property, of strict dominion or ownership such
and allowed Bobby, the eldest son who as construction of a house." In the
was about to get married, to construct his present case, of Alex is the real
house on ¼ of the lot, without however owner of the undeveloped and
obtaining the consent of his siblings. After untitled lot in Taguig, co-ownership is
settlement of Alex's estate and partition created among his wife and four
among the heirs, it was discovered that children over said property upon his
Bobby's house was constructed on the death. Since the construction of the
portion allocated to his sister, Cathy asked house by Bobby was done without
Bobby to demolish his house and vacate obtaining the consent of his siblings,
the portion alloted to her. In leiu of the alteration effected is illegal.
demolition, Bobby offered to purchase Bobby is considered to be in bad faith
from Cathy the lot portion on which his and as a sanction for his conduct, he
house was constructed. At that time, the can be compelled by Cathy to
house constructed was valued at demolish or remove the structure at
P350.000. his own expense.

(A). Can Cathy lawfully ask for demolition (B). Can Bobby legally insist on purchasing
of Bobby's house? (3%) the land? (2%)

“Never Let The Odds Keep You From Pursuing What You Know In Your Heart You Were Meant To Do.”-Leroy Satchel Paige
Page 77 of 180
Civil Law Q&As (2007-2013) hectorchristopher@yahoo.com dbaratbateladot@gmail.com

SUGGESTED ANSWER: nature and object to remain at a fixed


place on a river, lake or coast." Since
No. Bobby cannot legally insist on the floating platform is a petroleum
purchasing the land. Being in bad operation facility, it is intended to
faith, he has no option to pay for the remain permanently where it is
price of the lot (Art. 450, Civil Code). situated, even if it is tethered to a
ship which is anchored to the seabed.

ALTERNATIVE ANSWER:
Property; Movable or Immovable
(2007)
The platform is a movable property
because it is attached to a movable
No.II. Manila Petroleum Co. owned and
property, i.e. the vessel which was
operated a petroleum operation facility off
merely anchored to the seabed. The
the coast of Manila. The facility was
fact that the vessel is merely
located on a floating platform made of
anchored to the sea bed only shows
wood and metal, upon which was
that it is not intended to remain at a
permanently attached the heavy
fixed place; hence, it remains a
equipment for the petroleum operations
movable property. If the intention
and living quarters of the crew. The
was to make the platform stay
floating platform likewise contained a
permanent where it was moored, it
garden area, where trees, plants and
would not have been simply tethered
flowers were planted. The platform was
to a vessel but itself anchored to the
tethered to a ship, the MV 101, which was
seabed.
anchored to the seabed.

(B). Are the equipment and living quarters


Please briefly give the reason for your
movable or immovable property?
answers. (10%)

SUGGESTED ANSWER:
(A).Is the platform movable or immovable
property?
The thing and living quarters of the
crew are immovable property under
SUGGESTED ANSWER:
Art. 415
NCC, classifies as an immovable
The platform is an immovable
"everything attached to an
property under Art. 415 (9) NCC,
immovable in a fixed manner, in such
which provides that "docks and
a way that it cannot be separated
structures which, though floating, are
therefrom without breaking the
intended by their
material or deterioration of
“Never Let The Odds Keep You From Pursuing What You Know In Your Heart You Were Meant To Do.”-Leroy Satchel Paige
Page 78 of 180
Civil Law Q&As (2007-2013) hectorchristopher@yahoo.com dbaratbateladot@gmail.com

the object." Both the equipment and The trees, plants and flowers planted
the living quarters are permanently in the garden area of the platform are
attached to the platform which is also immovable property under Art. 415
an immovable. The equipment can (2) NCC which classifies as an
also be classified as an immovable immovable property "trees, plants
property under Art. 415 (5) NCC and growing fruits, while they are
because such equipment are attached to the land or form an
"machinery, receptacles, instruments integral part of an immovable, the
or implements intended by the owner petroleum operation facility.
of the tenement for an industry or
works which may be carried on in a ALTERNATIVE ANSWER:

building or on a piece of land and


The trees, plants and flowers planted
which tend directly to meet the needs
in the garden area of the platform are
of the industry or works." It is
movable property because they are
logically assumed that the petroleum
not permanently attached t the land
industry may be carried on in a
and do not form an integral part of an
building or on a piece of land and the
immovable. The platform is not an
platform is analogous to a building.
immovable property for the same

ALTERNATIVE ANSWER: reason already given in the


Alternative Answer to Item (a) above.
The equipment and living quarters of
the crew are movable properties Land Titles and Deeds
since they are attached to a platform
which is also a movable property, Acquisition of Lands; Sale of Real
because it is simply attached to a Property to an Alien (2009)
vessel is likewise a movable property
since it was merely anchored on the No.XIX. In 1972, Luciano de la Cruz sold to

seabed only shows that it is not Chua Chung Chun, a Chinese citizen, a

intended to remain at a fixed place; parcel of land in Binondo. Chua died in

hence, it remains a movable property. 1990, leaving behind his wife and three
children, one of whom, Julian, is a
(C). Are the trees, plants and flowers naturalized Filipino citizen. Six years after
immovable or movable property? Chua’s death, the heirs executed an
extrajudicial settlement of estate, and the
SUGGESTED ANSWER: parcel of land was allocated to Julian. In

“Never Let The Odds Keep You From Pursuing What You Know In Your Heart You Were Meant To Do.”-Leroy Satchel Paige
Page 79 of 180
Civil Law Q&As (2007-2013) hectorchristopher@yahoo.com dbaratbateladot@gmail.com

2007, Luciano filed suit to recover the land Non-Registrable Properties (2007)
he sold to Chua, alleging that the sale was
void because it contravened the No.IV. (B). What properties are not

Constitution which prohibits the sale of registrable? (5%)

private lands to aliens. Julian moved to


Supply this information.
dismiss the suit on grounds of pari delicto,
laches and acquisitive prescription. Decide
SUGGESTED ANSWER:
the case with reasons. (4%)

The following properties are not


SUGGESTED ANSWER:
registrable:

The case must be dismissed. Julian,


(1.) Properties of the Public
who is a naturialized Filipino citizen dominion;
and to whom the property was
allocated in a n extra-judicial (2.) Properties for public use or public
partition of the estate, is now the service;
owner of the property. The defect in
(3.) Inalienable lands of the public
ownership of the property of Julian’s
domain;
alien father has already been cured
by its transfer to Julian. It has been
(4.) Military installations, civil and
validated by the transfer of the
quasi-public lands; and
property to a Filipino citizen. Hence,
there is no more violation of the
(5.) All lands not classified as
Constitution because the subject real
alienable and disposable.
property is now owned by a Filipino
citizen (Halili v. CA, 287 SCRA 465, ALTERNATIVE ANSWER:
[1998]). Further, after the lapse of 35
year, laches has set in and the motion (1). Properties of public dominium
to dismiss may be granted, for the intended for public use, like roads,
failure of Luciano to question the canals, rivers, torrents, ports and
ownership of Chua before its transfer bridges constructed by the State,
of ownership to Julian. banks, shores, roadsteads, and the like,
are incapable of private appropriation,
much less registration (Art. 420 NCC).
This includes public markets, public
plazas, municipal
streets and public
buildings

“Never Let The Odds Keep You From Pursuing What You Know In Your Heart You Were Meant To Do.”-Leroy Satchel Paige
Page 80 of 180
Civil Law Q&As (2007-2013) hectorchristopher@yahoo.com dbaratbateladot@gmail.com

(Municipality of Antipolo v. Zapanta, (7.) Lands reclaimed by the


133 SCRA 820, 1986; Martinez v. CA, government from the sea, lakes, or
56 SCRA 647, 1974; Navera v. Quicho, other bodies of water are disposed or
5 SCRA 454, 1962). acquisible only by lease and not
otherwise, under the Public Land Act.
(2.) Lands proclaimed or classified as
forest, timberlands, mineral lands
and national parks. Under Sec 2, Art
XII, Constitution of the Philippines, Prescription; Acquisitive Prescription

these lands are inalienable. (2008)

(3.) Lands that are reserved by law or No. VII. Anthony bought a piece of untitled

Presidential proclamation for military, agricultural land from Bert. Bert, in turn,

civic or quasi-public purpose, Under acquired the property by forging carlo's

Sec 88, Chapter XII of the Public Land signature in a deed of sale over the

Act, such lands shall be inalienable property. Carlo had been in possession of

and shall not be subject to the property for 8 years, declared it for tax

occupation, entry, sale, lease or other purposes, and religiously paid all taxes

disposition. due on the property. Anthony is not aware


of the defect in Bert's title, but has been in
(4.) In general, all lands of the public actual physical possession of the property
domain that has not been classified from the time he bought it from Bert, who
as alienable and disposable under the had never been in possession of the
Public Land Act. property for one year.

(5.) Lands that form part of the (A). Can Anthony acquire ownership of the
seabed, riverbed or lakebed. These property by acquisitive prescription? How
lands are not susceptible to private many more years does he have possess it
appropriation. to acquire ownership? (2%)

(6.) Foreshore lands is that strip of SUGGESTED ANSWER:


land that lies between the high and
low water marks and alternately wet Yes, Anthony can acquire ownership

and dry according to the flow of the of the property through acquisitive

tide belong to the public domain, and prescription. In the present case,

can only be acquired by lease if not Anthony is a buyer/possessor in good

needed by the government for public


or quasi-public purposes.

“Never Let The Odds Keep You From Pursuing What You Know In Your Heart You Were Meant To Do.”-Leroy Satchel Paige
Page 81 of 180
Civil Law Q&As (2007-2013) hectorchristopher@yahoo.com dbaratbateladot@gmail.com

faith because he was not aware of shall have a right to a part of the
the defect in Bert's title (Art. 526, expenses of cultivation, and to a part
Civil Code). As such, Anthony can of the net harvest of the standing
acquire ownership and other real crops, both in proportion to the time
rights over immovable property of the possession (Art 545, Civil
through open, continuous possession Code).
of 10 years (Art. 1134, Civil Code).
Anthony needs nine
more years of possession, in addition
Prescription; Judicially Foreclosed
to his one (1) year of possession in
Real Property Mortgage (2012)
good faith.

No.IX.a) Does the right to request for the


(B).If Carlo is able to legally recover his
issuance of a writ of possession over a
property, can he require Anthony to
foreclosed real property prescribe in five
account for all the fruits he has harvested
(5) years? (5%)
from the property while in possession?
(2%)
SUGGESTED ANSWER:

SUGGESTED ANSWER: Yes, it prescribes in five (5) years. If


the real property mortgaged is
If Carlo is able to legally recover his
judicially foreclosed, the action for
property, he cannot require Anthony
judicial foreclosure should be filed
to account for all the fruits harvested
within a period of ten (10) years. The
from the property. Anthony is entitled
request for issuance of a writ of
to the fruits harvested in good faith
possession should be filed upon
before his possession was legally
motion of the winning bidder within
interrupted (Art. 544, Civil Code).
five (5) years after the judgment of
foreclosure. The writ of possession is
(C).If there are standing crops on the
an order commanding the sheriff to
property when Carlo recovers possession,
place a person named therein in
can Carlo appropriate them? (2%)
possession of real property (BPI v.

SUGGESTED ANSWER: Icot. G.R. No. 168081, Oct 12, 2009).

Yes, Carlos can appropriate only a


portion of the standing crops on the
property once he recovers
possession. Anthony being a
possessor in good faith,

“Never Let The Odds Keep You From Pursuing What You Know In Your Heart You Were Meant To Do.”-Leroy Satchel Paige
Page 82 of 180
Civil Law Q&As (2007-2013) hectorchristopher@yahoo.com dbaratbateladot@gmail.com

Purchaser in Good Faith; Mortgaged of Deeds under Act. 3344 and obtained a
Property (2008) tax declaration in its name.

No. XIX. Juliet offered to sell her house and (A). Was Dehlma a purchaser in good
lot, together with all the furniture and faith? (2%)
appliances therein to Dehlma. Before
agreeing to purchase the property, SUGGESTED ANSWER:

Dehlma went to the Register of Deeds to


Yes, Dehlma is a purchaser in good
verify Juliet's title. She discovered that
faith. In the present case, before
while the property was registered in
Dehlma bought the property, she
Juliet's name under the Land Registration
went to the Register of Deeds to
Act, as amended by the Property
verify Juliet's title. When she
Registration Decree, it property, Dehlma
discovered that the property was
told Juliet to redeem the property from
mortgaged to Elaine, she gave an
Elaine, and gave her an advance payment
advance payment so that Juliet could
to be used for purposes of realesing the
release the mortgage. It was only
mortgage on the property. When the
after the mortgage was released and
mortgage was released, Juliet executed a
free from the claims of other persons
Deed of Absolute Sale over the property
that Dehlma bought the property.
which was duly registered with the
Thus, Dehlma is a purchaser in good
Registry of Deeds, and a new TCT was
faith (Mathay v. CA, G.R. No. 115788,
issued in Dehlma's name. Dehlma
17 Sept, 1998).
immediately took possession over the
house and lot and the movables therein.
(B). Who as between Dehlma and XYZ Bank
Thereafter, Dehlma went to theAssessor's
has a better right to the house and lot? (2%)
Office to get a new tax declaration under
her name. She was surprised to find out SUGGESTED ANSWER:
that the property was already declared for
tax purposes in the name of XYZ Bank Between Dehlma and XYZ Bank,
which had foreclosed the mortgage on the Dehlma has a better right to the
property before it was sold to her. XYZ house and lot. After the release of
Bank was also the purchaser in the the mortgage, the Deed of Absolute
foreclosure sale of the property. At that Sale was registered and a new title
time, the property was still unregistered was issued in Dehlma's name. Act
but XYZ Bank registered the Sheriff's Deed 3344 is applicable exclusively to
of Conveyance in the day book of the instruments resulting from
Register agreement of parties thereto and
does not apply to deeds of a sheriff
conveying to a
“Never Let The Odds Keep You From Pursuing What You Know In Your Heart You Were Meant To Do.”-Leroy Satchel Paige
Page 83 of 180
Civil Law Q&As (2007-2013) hectorchristopher@yahoo.com dbaratbateladot@gmail.com

purchaser unregistered lands sold to land registration and acquisition of title to


him under execution (Williams v. land. The manual should include the
Suñer, 49 Phil. ,534). following items:

(C). Who owns the movables inside the (A). What is the governing law? (5%)
house? (2%)
SUGGESTED ANSWER:
SUGGESTED ANSWER:
The governing law is the Land
Dehlma owns the movables because Registration Act as amended by
when she acquired the house and lot Property Registration Decree (Act
from Juliet, all the furniture and 496 as amended by PD 1529).
appliances therein were included in
the sale. As owner of the real [Note: It is respectfully recommended

property, Dehlma also owns the that full credit be given to examinees

movables found therein (Art. 542, who did not give the exact title or

Civil Code). number of the law but merely stated


a description of the law.]

ALTERNATIVE ANSWER:
Registration; Governing Law (2007)
In general, the governing law relating
No.IV. Bedrock Land & Property to registration and acquisition of title
Development Corp. is a development to land is Act 496 of 1902 as
company engaged in developing and amended by PD 1529, otherwise
selling subdivisions, condominium units known as Property Registration
and industrial estates. In order to Decree of June 11, 1978.
replenish its inventories, it embarked on
an aggressive land banking program. It (1.) Chapter III-I governs original

employed "scouts" who roam all over the registration of land title under the

Philippines to look for and conduct Torrens System by voluntary ordinary

investigations on prospective sites for judicial proceedings.

acquisition and development, whether


(2.) Chapter II-II governs compulsory
developed, semi-developed or raw land.
registration of lands through
The management of Bedrock asks you as
cadastral proceedings.
the company counsel to prepare a manual
containing a summary of the pertinent
laws and regulations relating to

“Never Let The Odds Keep You From Pursuing What You Know In Your Heart You Were Meant To Do.”-Leroy Satchel Paige
Page 84 of 180
Civil Law Q&As (2007-2013) hectorchristopher@yahoo.com dbaratbateladot@gmail.com

(3.) Section 103 governs registration Replacement of lost or destroyed


of homestead, sales, free patent owner's duplicate certificate of title.
under CA No. 141, as amended,
otherwise known as the Public Land (9.) R.A. No. 26 governs judicial

Act. reconstitution of lost or destroyed


originals of the certificate of title.
(4.) Section 104 governs registration
of (10.) R.A. No. 6732 governs
certificates of land transfers, administrative reconstitution of lost
emancipation patents and or destroyed original certificates of
Certificates of Land Ownership Award title.
(CLOA) under Comprehensive Land
Reform Law. (11.) Section 113 governs the
registration of instruments affecting
(5.) Chapter V governs the unregistered private lands.
registration of land dealings on
registered land like conveyances, (12.) Section 117 governs
transfers, mortgages, leases, powers "consultas," where the Register of
of attorney, trusts and similar Deeds refuses to register a deed or
contracts inter vivos. when he is in doubt as to what action
to take on an instrument presented
(6.) Chapter V-II governs the for registration.
registration of involuntary dealings
on registered land like attachments,
adverse claims, enforcement of liens
on registered land, notices of lis Registration; Party Who First took
Possession (2013)
pendens. (7.) Chapter VI governs the
registration of judgments, orders and
No.IX.Rica petitioned for the annulment of
partitions, condemnation in eminent
her ten-year old marriage to Richard.
domain proceedings, judicial and
Richard hired Atty. Cruz to represent him
extra-judicial settlement of estates.
in the proceedings. In payment for Atty.
Cruz's acceptance and legal fees, Richard
(8.) Sections 107, 108 and 109 govern
conveyed to Atty. Cruz a parcel of land in
petitions and actions after original
Taguig that he recently purchased with his
registration like: (a).Compulsory
lotto winnings. The transfer documents
surrender of withheld owner's
were duly signed and Atty. Cruz
duplicate certificate of title;

Amendment and alteration of


certificate of title;
“Never Let The Odds Keep You From Pursuing What You Know In Your Heart You Were Meant To Do.”-Leroy Satchel Paige
Page 85 of 180
Civil Law Q&As (2007-2013) hectorchristopher@yahoo.com dbaratbateladot@gmail.com

immediately took possession by fencing the liquidation of the absolute


off the property's entire perimeter. community or conjugal partnership of
the spouses as the case may be (Art.
Desperately needing money to pay for his 50 in relation to Art 43 of the Family
mounting legal fees and his other needs Code). Richard purchased the land
and despite the transfer to Atty. Cruz, with his lotto winnings during the
Richard offered the same parcel of land for pendency of the suit for annulment
sale to the spouses Garcia. After and on the assumption that the
inspection of the land, the spouses parties are governed by the regime of
considered it a good investment and absolute community or conjugal
purchased it from Richard. Immediately partnership, winnings from gambling
after the sale, the or betting will form part thereof.
spouses Garcia commenced the Also, since the land is part of the
construction of a three-story building over absolute community or conjugal
the land, but they were prevented from partnership of Richard and Rica, it
doing this by Atty. Cruz who claimed he may not be sold or alienated without
has a better right in light of the prior the consent of the latter and any
conveyance in his favor. disposition or encumbrance of the
property of the community or the
Is Atty. Cruz's claim correct? (8%)
conjugal property without the
consent of the other spouse is void
SUGGESTED ANSWER:
(Art 96 and Art 124, Family Code).

No. Atty. Cruz is not correct. At first


glance, it may appear that Atty. Cruz
is the one who has the better right
Registration; Requisites; Proof (2013)
because he first took possession of
the property. However, a lawyer is No.X. Manuel was born on 12 March 1940
prohibited under Art 1491 of the Civil in a 1 000-square meter property where
Code from acquiring the property and he grew up helping his father, Michael,
rights which may be the object of any cultivate the land. Michael has lived on the
litigation in which they may take part property since the land was opened for
by virtue of their profession. While settlement at about the time of the
the suit is for annulment of marriage Commonwealth government in 1935, but
and it may be urged that the land for some reason never secured any title to
itself is not the object of the the property other than a tax declaration
litigation, the annulment of marriage, in his name. He has held the property
if granted, will carry with it through

“Never Let The Odds Keep You From Pursuing What You Know In Your Heart You Were Meant To Do.”-Leroy Satchel Paige
Page 86 of 180
Civil Law Q&As (2007-2013) hectorchristopher@yahoo.com dbaratbateladot@gmail.com

the years in the concept of an owner and ownership since June 12, 1945, or
his stay was uncontested by others. He earlier. However, it is only necessary
has also conscientiously and continuously that the land is already declared A &
paid the realty taxes on the land. D land “at the time for application for
registration is filed” (Malabanan v.
Michael died in 2000 and Manuel - as Republic, G.R. No. 180067, June 30,
Michael’s only son and heir -now wants to 2009).
secure and register title to the land in his
own name. He consults you for legal Manuel could also invoke Sec 14 (2)
advice as he wants to perfect his title to of the same Decree, which allows
the land and secure its registration in his registration through ordinary
name. acquisitive prescription for thirty
years, provided, however, that the
What are the laws that you need to land is “patrimonial” in character, i.e.
consider in advising Manuel on how he can already declared by the government
perfect his title and register the land in his (a) as A & D, and (b) no longer
name? Explain the relevance of these laws needed for public use or public
to your projected course of action. (4%) service (Malabanan, supra).

SUGGESTED ANSWER: Manuel could also file an application


for
(Note: With all due respect, it is “confirmation of imperfect or
recommended that the examiner accept incomplete title’ through “judicial
and give full credit to any of the answers legalization” under Sec. 48 (b) of CA
given in each of the following no. 141, or the Public Land Act (PLA).
paragraphs.) But, as held in Malabanan, there is no
substantial difference between this
I would advice Manuel to file an provision and Sec 14 (1) of the PRD.
application for registration under Sec Both refer to agricultural lands
14 of Pres. Decree No. 1529, or the already classified as alienable and
Property Registration Decree (PRD), disposable at the time the application
specifically Sec14 (1) which requires is filed, and require possession and
(a) that the land applied for forms occupation since June 12, 1945. The
part of the alienable and disposable only difference is that under the PRD,
(A & D) portion of the public domain, there already exists a title which is to
and (b) that the applicant has been in be confirmed, whereas under the
open, continuous and notorious PLA, the presumption is that land is
possession and occupation thereof still public land (Republic v.
under bona fide claim of

“Never Let The Odds Keep You From Pursuing What You Know In Your Heart You Were Meant To Do.”-Leroy Satchel Paige
Page 87 of 180
Civil Law Q&As (2007-2013) hectorchristopher@yahoo.com dbaratbateladot@gmail.com

Aquino, G.R. No. L-33983, January 27, Manuel has a the burden to overcome
1983). the presumption of State ownership
by “well-nigh incontrovertible”
Manuel may also invoke “vested evidence (Ong v. Republic, G.R. No.
rights’ acquired under Rep. Act. No. 175746, March 12, 2008).
1942, dated June 2, 1957, which Accordingly, he must show that ht
amended Sec. 48 (b) of the PLA by eland is already classified as A & D
providing for a prescriptive period of “at the time the application for
thirty years for judicial confirmation registration is filed” and that he has
of imperfect title. It must only be been in “possession and occupation
demonstrated that possession and thereof” in the manner required by
occupation commenced on January law since June 12, 1945, or earlier.
24, 1947 and the 30-year period was
completed prior to the effectivity of Manuel may tack his possession to
PD No. 1073 on January 25, 1977. PD that of his predecessor-in-interest
No. 1073 now requires possession (Michael) by the testimony of
and occupation since June 12, 1945 disinterested and knowledgeable
(Republic v. Espinosa, G.R. No. eyewitnesses. Overt acts of
171514, July 18, 2012). possession may consist in introducing
valuable improvements like fencing
Another alternative is for Manuel to the land, constructing a residential
secure title through administrative house thereon, cultivating the land
proceedings under the homestead or and planting fruit bearing trees,
free patent provisions of the PLA. The declaring the land for taxation
title issued has the same efficacy and purposes and paying realty taxes, all
validity as a title issued through of which are corroborative proof of
judicial proceedings, but with the possession.
limitations that the land cannot be
sold or disposed of within five years To identify the land, he must submit
from the issuance of patent (Sec. the tracing cloth plan or a duly-
118, CA No. 141, as amended). certified blueprint or whiteprint copy
thereof (Director of Lands v. Reyes,
What do you have to prove to secure G.R. No. L-27594, November 28,
Manuel's objectives and what 1975; Director of Lands v. CA and
documentation are necessary? (4%) Iglesia ni Cristo, G.R. No. L-56613,
March 14, 1988).
SUGGESTED ANSWER:

To show the classification of the land


as A & D, the application must be
accompanied by (1) a CENRO or
PENRO

“Never Let The Odds Keep You From Pursuing What You Know In Your Heart You Were Meant To Do.”-Leroy Satchel Paige
Page 88 of 180
Civil Law Q&As (2007-2013) hectorchristopher@yahoo.com dbaratbateladot@gmail.com

certification; and (2) a certified true


copy of the original classification I will institute the following actions
approved by the DENR Secretary against Atty. Tan:
(Republic v. Bantigue, G.R.No.
162322, March 14, 2012). A (a). A civil action for damage for the
presidential or legislative act may fraudulent transfer of the title in his
also be considered. name and to recover the value of the
property;

(b). An action against the National


Remedies; Fraud; Rights of Innocent
Treasurer for compensation from the
Purchaser (2009)
State Assurance Fund which is set
aside by law to pay those who lose
No.IX. Before migrating to Canada in 1992,
their land suffer damages as a
the spouses Teodoro and Anita entrusted
consequence of the operation of the
all their legal papers and documents to
Torrens system;
their nephew, Atty. Tan. Taking advantage
of the situation, Atty. Tan forged a deed of
(c). A criminal action for forgery or
sale, making it appear that he had bought
falsification of public document;
the couple’s property in Quezon City. In
2000, he succeeded in obtaining a TCT
(d). A complaint with the Supreme
over the property in his name.
Court/Integrated Bar of the
Subsequently, Atty. Tan sold the same
Philippines to disbar or suspend him
property to Luis, who built an auto repair
or other disciplinary action for
shop on the property. In 2004, Luis
violation or the Code of Professional
registered the deed of conveyance, and
Ethics.
title over the property was transferred in
his name.
Any action against Luis will not

In 2006, the spouses Teodoro and Anita prosper because he is an innocent

came to the Philippines for a visit and purchaser for value. The Title to the

discovered what had happened to their land he bought was already in the

property. They immediately hire you as name of the person who sold the

lawyer. What action or actions will you property to him, and there is nothing

institute in order to vindicate their rights? on the title which will make him

Explain fully. (4%) suspect about the fraud committed


by Atty. Tan.
SUGGESTED ANSWER:

“Never Let The Odds Keep You From Pursuing What You Know In Your Heart You Were Meant To Do.”-Leroy Satchel Paige
Page 89 of 180
Civil Law Q&As (2007-2013) hectorchristopher@yahoo.com dbaratbateladot@gmail.com

Contracts Rescission of Contract; Fortuitous


Event (2008)

Contract to Sell vs. Conditional


No.XVIII. AB Corp. entered into a contract
Contract of Sale (2012)
with XY Corp. whereby the former agreed

No.X.a) A contract to sell is the same as a to construct the research and laboratory

conditional contract of sale. Do you agree? facilities of the latter. Under the terms of

Explain your answer. (5%) the contract, AB Corp. agreed to complete


the facility in 18 months, at the total
SUGGESTED ANSWER: contract price of P10 million. XY Corp. paid
50% of the total contract price, the
No. A contract to sell is a species of
balance to be paid upon completion of the
conditional sale. The contract to sell
work. The work stated immediately, but
does not sell a thing or property; it
AB Corp. later experienced work slippage
sells the right to buy property. A
because of labor unrest in his company. AB
conditional sale is a sale subject to
Corp.'s employees claimed that they are
the happening or performance of a
not being paid on time; hence, the work
condition, such as payment of the full
slowdown. As of the 17th month, work was
purchase price, or the performance of
only 45% completed. AB Corp. asked for
other prestation to give, to do or not
extension of time, claiming that its labor
to do. Compliance with the condition
problems is a case of fortuitous event, but
automatically gives the right to the
this was denied by XY Corp. When it
vendee to demand the delivery of the
became certain that the contruction could
object of the sale. In a contract to
not be finished on time, XY Corp. sent
sell, however, the compliance with
written notice cancelling the contract, and
the condition does not automatically
requiring AB Corp. to immediately vacate
sell the property to the vendee. It the premises.
merely gives the vendee the right to
compel the vendor to execute the (A). Can the labor unrest be considered a
deed of absolute sale. fortuitous event? (1%)

SUGGESTED ANSWER:

No. The labor unrest cannot be


considered a fortuitous event under
Art. 1174 of the Civil Code. A
fortuitous event should occur
independent of the

“Never Let The Odds Keep You From Pursuing What You Know In Your Heart You Were Meant To Do.”-Leroy Satchel Paige
Page 90 of 180
Civil Law Q&As (2007-2013) hectorchristopher@yahoo.com dbaratbateladot@gmail.com

will of the debtor or without his statement is false. Explain your answer in
participation or aggravation (Paras, not more than two (2) sentences.
Civil Code Annotated, vol. IV, 2000
ed., p 159). As mentioned in the (A). A clause in an arbitration contract

facts, labor unrest of the employees granting one of the parties the power to

was caused by AB Corp.'s failure to choose more arbitrators than the other

pay its employees on time. renders the arbitration contract void. (1%)

(B). Can XY Corp. unilaterrally and SUGGESTED ANSWER:

immediately cancel the contract? (2%) True. The Civil Code provides that
“Any clause giving one of the parties
SUGGESTED ANSWER: power to choose more arbitrators
than the other is void and of no
No, XY Corp. cannot unilaterally and effect” (Art 2045, NCC).
immediately cancel the contract. In
Obligations
the absence of any stipulation for
automatic rescission, rescission must
Extinguishment; Compensation
be judicial (Art. 1191, Civil Code). (2009)

(C). Must AB Corp. return the 50% No.XV. Sarah had a deposit in a savings
downpayment? (2%) account with Filipino Universal Bank in the
amount of five million pesos
SUGGESTED ANSWER:
(P5,000,000.00). To buy a new car, she
obtained a loan from the same bank in the
AB Corp. need not return the 50%
amount of P1,200,000.00, payable in
down payment because 45% of the
twelve monthly installments. Sarah issued
work was already completed,
in favor of the bank post-dated checks,
otherwise, XY Corp. would be unjustly
each in the amount of P100,000.00, to
enriching itself at the expense of AB
cover the twelve monthly installment
Corp.
payments. On the third, fourth and fifth
months, the corresponding checks
bounced.
Stipulation; Arbitration Clause (2009)
The bank then declared the whole
No. XI. TRUE or FALSE. Answer TRUE if obligation due, and proceeded to deduct
the statement is true, or FALSE if the the
amount of one million pesos
(P1,000,000.00) from Sarah’s deposit after
notice to her that this is a form of
“Never Let The Odds Keep You From Pursuing What You Know In Your Heart You Were Meant To Do.”-Leroy Satchel Paige
Page 91 of 180
Civil Law Q&As (2007-2013) hectorchristopher@yahoo.com dbaratbateladot@gmail.com

compensation allowed by law. Is the bank Extinguishment; Compensation


(2008)
correct? Explain. (4%)

No. XV. Eduardo was granted a loan by


SUGGESTED ANSWER:
XYZ Bank for the purpose of improving a
No, the bank is not correct. While the
building which XYZ leased from him.
Bank is correct about the applicability
Eduardo, executed the promissory note
of compensation, it was not correct
("PN") in favor of the bank, with his friend
as to the amount compensated.
Recardo as co-signatory. In the PN, they

A bank deposit is a contract of loan, both acknowledged that they are

where the depositor is the creditor "individually and collectively" liable and

and the bank the debtor. Since Sarah waived the need for prior demand. To

is also the debtor of the bank with secure the PN, Recardo executed a real

respect to the loan, both are mutually estate mortgage on his own property.

principal debtors and creditors of When Eduardo defaulted on the PN, XYZ

each other. Both obligation are due, stopped payment of rentals on the

demandable and liquidated but only building on the ground that legal

up to the extent of P300,000.00 compensation had set in. Since there was

(covering the unpaid third, fourth and still a balance due on the PN after

fifth monthly installments). The applying the rentals, XYZ foreclosed the

entire one million was not yet due real estate mortgage over Recardo's

because the loan has no acceleration property. Recardo opposed the foreclosure

clause in case of default. And since on the ground that he is only a co-

there is no retention or controversy signatory; that no demand was made

commenced by third person and upon him for payment, and assuming he is

communicated in due time to the liable, his liability should not go beyond

debtor, then all the requisites of legal half the balance of the loan. Further,

compensation are present but only up Recardo said that when the bank invoked

to the amount of P300,000.00. The compensation between the reantals and

bank, therefore, may deduct the amount of the loan, it amounted to a

P300,000.00 from Sarah’s bank new contract or novation, and had the

deposit by way of compensation. effect of extinguishing the security since


he did not give his consent (as owner of
the property under the real estate
mortgage) thereto.

(A). Can XYZ Bank validly assert legal


compensation? (2%)

“Never Let The Odds Keep You From Pursuing What You Know In Your Heart You Were Meant To Do.”-Leroy Satchel Paige
Page 92 of 180
Civil Law Q&As (2007-2013) hectorchristopher@yahoo.com dbaratbateladot@gmail.com

SUGGESTED ANSWER: ground that legal compensation had set


in. Since there was still a balance due on
Yes, XYZ Bank can validly assert legal the PN after applying the rentals, XYZ
compensation. In the present case, foreclosed the real estate mortgage over
all of the elements of legal Recardo's property. Recardo opposed the
compensation are present: (1) XYZ foreclosure on the ground that he is only a
Bank is the creditor of Eduardo while co-signatory; that no demand was made
Eduardo is the lessor of XYZ Bank; (2) upon him for payment, and assuming he is
both debts consist in a sum of money, liable, his liability should not go beyond
or if the things due are consumable, half the balance of the loan. Further,
they be of the same kind, and also of Recardo said that when the bank invoked
the same quality if the latter has compensation between the reantals and
been stated; (3) the two debts be the amount of the loan, it amounted to a
due; (4) they be liquidated and new contract or novation, and had the
demandable, and (5) over neither of effect of extinguishing the security since
them there be any retention or he did not give his consent (as owner of
controversy, commenced by third the property under the real estate
persons and communicated in due mortgage) thereto.
time to the debtor (Art. 1279, Civil
Code). (C). Does Recardo have basis under the
Civil Code for claiming that the original
contract was novated? (2%)

Extinguishment; Novation (2008) SUGGESTED ANSWER:

No. XV. Eduardo was granted a loan by No. Recardo has no basis for claiming
XYZ Bank for the purpose of improving a novation of the original contract
building which XYZ leased from him. when the bank invoked compensation
Eduardo, executed the promissory note because there was simply partial
("PN") in favor of the bank, with his friend compensation (Art. 1290, Civil Code)
Recardo as co-signatory. In the PN, they and this would not bar the bank from
both acknowledged that they are recovering the remaining balance of
"individually and collectively" liable and the obligation.
waived the need for prior demand. To
secure the PN, Recardo executed a real ALTERNATIVE ANSWER:
estate mortgage on his own property.
When Eduardo defaulted on the PN, XYZ No. In order that an obligation may

stopped payment of rentals on the be extinguished by another, it is

building on the imperative


“Never Let The Odds Keep You From Pursuing What You Know In Your Heart You Were Meant To Do.”-Leroy Satchel Paige
Page 93 of 180
Civil Law Q&As (2007-2013) hectorchristopher@yahoo.com dbaratbateladot@gmail.com

that it be so declared in unequivocal or creditors (Philippine Airlines v. CA


terms, or that the old and new and Amelia Tan, G.R. No. L-49188,
obligations be on every point 1990). Mere delivery of checks does
compatible with each other. Novation not discharge the obligation under a
is never presumed (Art. 1292, Civil judgment. A check shall produce the
Code). effect of payment only when they
have been cashed or where through
the fault of the creditor they have
been impaired (Art 1249, Civil Code).
Extinguishment; Payment of Check
(2013) However, it is not necessary that the
right of redemption be exercised by
No.VI. Lito obtained a loan of P1,000,000
delivery of legal tender. A check may
from Ferdie, payable within one year. To
be used for the exercise of right of
secure payment, Lito executed a chattel
redemption, the same being a right
mortgage on a Toyota Avanza and a real
and not an obligation. The tender of a
estate mortgage on a 200-square meter
check is sufficient to compel
piece of property.
redemption but is not in itself a
payment that relieves the
Lito's failure to pay led to the extra-judicial
redemptioner from his liability to pay
foreclosure of the mortgaged real
the redemption price (Biana v.
property. Within a year from foreclosure,
Gimenez, G.R. No. 132768, Sept 9,
Lito tendered a manager's check to Ferdie
2005, citing Fortunado v. CA).
to redeem the property. Ferdie refused to
accept payment on the ground that he
Redemption within the period allowed
wanted payment in cash: the check does
by law is not a matter of intent but a
not qualify as legal tender and does not
question of payment or valid tender
include the interest payment. Is Ferdie's
of full redemption prices within the
refusal justified? (4%)
said period. Whether redemption is
being made under Art. 3135 or under
SUGGESTED ANSWER:
the General Banking Law, the

A check, whether a manager’s check mortgagor or his assignee is required

or an ordinary check is not legal to tender payment to make said

tender, and an offer of a check in redemption valid (Heirs of

payment of a debt is not a valid Quisumbing v. PNB and SLDC, G.R.

tender of payment and may be No. 178242, Jan 20, 2009).

refused receipt by the oblige

“Never Let The Odds Keep You From Pursuing What You Know In Your Heart You Were Meant To Do.”-Leroy Satchel Paige
Page 94 of 180
Civil Law Q&As (2007-2013) hectorchristopher@yahoo.com dbaratbateladot@gmail.com

Moreover, Ferdie’s refusal was there has been extraordinary deflation


justified on the ground that the since 1998, and therefore, Felipe should
amount tendered does not include pay him the value of the debt at the time
interest. In order to effect the it was incurred. Felipe refused to pay him
redemption of the foreclosed again, claiming that Gustavo is estopped
property, the payment to the from raising the issue of legal tender,
purchaser must include the following having accepted the check in March, and
sums: (a) the bid price; (b) the that it was Gustavo's negligence in not
interest on the bid price, computed at depositing the check immediately that
one per centum (1%) per month; and caused the check to become stale.
(c) the assessments and taxes, if any,
paid by the purchaser with the same (A). Can Gustavo now raised the issue that

rate of interest (Sec 28, 1997 Rules of the cashier's check is not legal tender?

Civil Procedure). Unless there is an (2%)

express stipulation to that effect, the


SUGGESTED ANSWER:
creditor cannot be compelled to
receive partial payment of the
No. Gustavo previously accepted a
prestation (Art. 1248, Civil Code).
check as payment. It was his fault
why the check became stale. He is
now estopped from raising the issue

Extinguishment; Payment of Check; that a cashier's check is not legal

Legal Tender (2008) tender.

No. XVII. Felipe borrowed $100 from (B). Can Felipe validly refuse to pay

Gustavo in 1998, when the Phil P - US$ Gustavo again? (2%)

exchange rate was P56 - US$1. On March


SUGGESTED ANSWER:
1, 2008, Felipe tendered to Gustavo a
cashier's check in the amount of P4,135 in
Yes, Felipe can refuse to pay Gustavo,
payment of his US$ 100 debt, based on
who allowed the check to become
the Phil P - US$ exchange rat at that time.
stale. Although a check is not legal
Gustavo accepted the check, but forgot to
tender (Belisario v. Natividad. 60 Phil
deposit it until Sept. 12, 2008. His bank
156), there are instances when a
refused to accepted the check because it
check produces the effects of
had become stale. Gustavo now wants
payment, for example: (a) when the
Felipe to pay him in cash the amount of
creditor is in estoppel or he had
P5,600. Claiming that the previous
previously promised he would accept
payment was not in legal tender, and that
a check (Paras, Civil

“Never Let The Odds Keep You From Pursuing What You Know In Your Heart You Were Meant To Do.”-Leroy Satchel Paige
Page 95 of 180
Civil Law Q&As (2007-2013) hectorchristopher@yahoo.com dbaratbateladot@gmail.com

Code Annotated, Vol IV, 2000 ed., p. secure the PN, Recardo executed a real
394); (b) when the check has lost its estate mortgage on his own property.
value because of the fault of the When Eduardo defaulted on the PN, XYZ
creditor (Art. 1249, 2nd par.),as when stopped payment of rentals on the
he was unreasonably delayed in building on the ground that legal
presenting the check for payment compensation had set in. Since there was
(PNB v. Seeto, G.R. No, L-4388, 13 still a balance due on the PN after
August 1952). applying the rentals, XYZ foreclosed the
real estate mortgage over Recardo's
(C). Can Felipe compel Gustavo to receive property. Recardo opposed the foreclosure
US$100 instead? (1%) on the ground that he is only a co-
signatory; that no demand was made
SUGGESTED ANSWER:
upon him for payment, and assuming he is
liable, his liability should not go beyond
Felipe cannot compel Gustavo to
half the balance of the loan. Further,
receive US$100 because under RA
Recardo said that when the bank invoked
529, payment of loans should be at
compensation between the reantals and
Philippine currency at the rate of
the amount of the loan, it amounted to a
exchange prevailing at the time of
new contract or novation, and had the
the stipulated date of payment.
effect of extinguishing the security since
Felipe could only compel Gustavo to
he did not give his consent (as owner of
receive US$ 100 if they stipulated
the property under the real estate
that obligation be paid in foreign
mortgage) thereto.
currency (R.A. 4100).

(B). Can Recardo's property be foreclosed


to pay the full balance of the loan? (2%)

Liability; Solidary Liability (2008)


SUGGESTED ANSWER:

No. XV. Eduardo was granted a loan by


Yes, Recardo's property can be
XYZ Bank for the purpose of improving a
foreclosed to pay the full balance of
building which XYZ leased from him.
the loan because when he signed as
Eduardo, executed the promissory note
co-signatory in the promissory note,
("PN") in favor of the bank, with his friend
he acknowledged he is solidarily
Recardo as co-signatory. In the PN, they
liable with Eduardo. In solidary
both acknowledged that they are
obligations, a creditor has the right
"individually and collectively" liable and
to demand full payment of the
waived the need for prior demand. To
obligation from any of

“Never Let The Odds Keep You From Pursuing What You Know In Your Heart You Were Meant To Do.”-Leroy Satchel Paige
Page 96 of 180
Civil Law Q&As (2007-2013) hectorchristopher@yahoo.com dbaratbateladot@gmail.com

the solidary debtors (Art. 1207, Civil latter, is obliged to continue the same
Code). until the termination of the affair and
its incidents, or to require the person
concerned to substitute him, if the
owner is in a position to do so (Art.
Obligations; Without Agreement
2144, NCC).
(2007)

Second example, a case of solutio


No.V. What are obligations without an
indebiti may also give rise to an
agreement"? Give five examples of
obligation without an agreement.
situations giving rise to this type of
This refers to the obligation to return
obligations? (10%)
which arises when something is
SUGGESTED ANSWER: received when there is no right to
demand it, and it was unduly
"Obligations without an agreement" delivered through mistake (Art. 2154,
are obligations that do not arise from NCC).
contract such as those arising from:
1. delicts; 2. quasi-delicts; 3. solutio Third example, is when without the
indebiti; 4. negotiorum gestio; and 5. knowledge of the person obliged to
all other obligations arising from law. give support, it is given by a stranger,
the latter shall have a right to claim
ALTERNATIVE ANSWER: the same from the former, unless it
appears that he gave it out of piety
"Obligations without an agreement" and without intention of being repaid
refer to the juridical relation of quasi- (Art. 2164, NCC).
contract which arise from certain
lawful, voluntary and unilateral acts Fourth example, is when through
to the end that no one shall be accident or other causes a person is
unjustly enriched or benefited at the injured or becomes seriously ill, and
expense of another. (Art. 2142, NCC) he is treated or helped while he is not
in a condition to give consent to a
First Example of an obligation contract, he shall be liable to pay for
without an agreement is a case of the services of the physician or other
negotiorum gestio, whereby one who person aiding him, unless the service
voluntarily takes charge of the has been rendered out of pure
agency or management of the generosity (Art. 2167, NCC).
business or property of another
without any power from the

“Never Let The Odds Keep You From Pursuing What You Know In Your Heart You Were Meant To Do.”-Leroy Satchel Paige
Page 97 of 180
Civil Law Q&As (2007-2013) hectorchristopher@yahoo.com dbaratbateladot@gmail.com

Fifth instance of an obligation ought not, in equity and good


without an agreement is when the conscience, to hold (Heirs of Lorenzo
person obliged to support an orphan Yap v. CA, 371 Phil 523, 1991). The
or an insane or other indigent person following are examples of
unjustly refuses to give support to constructive trust: 1. Art. 1456 NCC
the latter, any third person may which provides: "If property is
furnish support to the needy acquired through mistake or fraud,
individual, with right of the person obtaining it is, by force of
reimbursement from the person law considered a trustee of an
obliged to give support. The implied trust for the benefit of the
provisions of this article apply when person for whom the property
the father or mother of a child under comes." 2. Art 1451 NCC which
eighteen years of age unjustly provides: "When land passes by
refuses to support him (Art. 2166, succession through any person and
NCC). he causes the legal title to be put in
the name of another, a trust is
Trust established by implication of law for
the benefit of the true owner." 3. Art
Trust De Son Tort (2007) 1454 NCC which provides: "If an
absolute conveyance of property is
No.III. Explain the following concepts and made in order to secure the
doctrines and give an example of each: performance of an obligation of the
grantor toward the grantee, a trust
(A). concept of trust de son
by virtue of law is established. If the
tort (constructive trust) (5%)
fulfillment of the obligation is offered
by the grantor when it becomes due,
SUGGESTED ANSWER:
he may demand the reconveyance of
the property to him." 4. Art 1455 NCC
A constructive trust is a trust NOT
which provides: "When any trustee,
created by any word or phrase, either
guardian or any person holding a
expressly or impliedly, evincing a
fiduciary relationship uses trust funds
direct intention to create a trust, but
for the purchase of property and
is one that arises in order to satisfy
causes conveyance to be made to him
the demands of justice. It does not
or to third person, a trust is
come about by agreement or
established by operation of law in
intention but mainly operation of law
favor of the person to whom the
and construed as a trust against one
funds belong."
who, by fraud, duress or abuse of
confidence, obtains or holds the legal
right to property which he
“Never Let The Odds Keep You From Pursuing What You Know In Your Heart You Were Meant To Do.”-Leroy Satchel Paige
Page 98 of 180
Civil Law Q&As (2007-2013) hectorchristopher@yahoo.com dbaratbateladot@gmail.com

Sales advanced for her employees (biyaheros).


She required them to surrender TCT of
Condominium Act; Partition of a their properties and to execute the
Condominium (2009) corresponding Deeds of Sale in her favor.
Domeng Bandong was not required to post
No.XVIII. The Ifugao Arms is a
any security but when Eulalia discovered
condominium project in Baguio City. A
that he incurred shortage in cattle
strong earthquake occurred which left
procurement operation, he was required to
huge cracks in the outer walls of the
execute a Deed of Sale over a parcel of
building. As a result, a number of
land in favor of Eulalia. She sold the
condominium units were rendered unfit for
property to her grandneice Jocelyn who
use. May Edwin, owner of one of the
thereafter instituted an action for
condominium units affected, legally sue
ejectment against the Spouses Bandong.
for partition by sale of the whole project?
Explain. (4%) To assert their right, Spouses Bandong
filed an action for annulment of sale
SUGGESTED ANSWER:
against Eulalia and Jocelyn alleging that
Yes, Edwin may legally sue for
there was no sale intended but only
partition by sale of the whole
equitable mortgage for the purpose of
condominium project under the
securing the shortage incurred by Domeng
following conditions:
in the amount of P 70, 000.00 while
the damage or destruction caused by
employed as "biyahero" by Eulalia. Was
the earthquake has rendered one-half
the Deed of Sale between Domeng and
(1/2) or more of the units therein
Eulalia a contract of sale or an equitable
untenantable, and (b) that the
mortgage? Explain. (5%)
condominium owners holding an
aggregate of more than thirty SUGGESTED ANSWER:
percent (30%) interests of the
common areas are opposed to the The contract between Domeng
restoration of the condominium Bandong and Eulalia was an equitable
project (Sec 8 [b], Republic mortgage rather than a contract of
Act No. 472 “Condominium Act”). sale. The purported deed of sale was
actually intended to merely secure
the payment of the shortage incurred
Mortgage; Equitable Mortgage (2012) by Domeng in the conduct of the
cattle-buying operations. Under Art
No.VI. (b) Eulalia was engaged in the 1602, Civil Code,
business of buying and selling large cattle.
In order to secure the financial capital, she

“Never Let The Odds Keep You From Pursuing What You Know In Your Heart You Were Meant To Do.”-Leroy Satchel Paige
Page 99 of 180
Civil Law Q&As (2007-2013) hectorchristopher@yahoo.com dbaratbateladot@gmail.com

the contract shall be presumed to be offering P800,000 in ready cash for the
an equitable mortgage when it may land. When Roberto confirmed that he
be fairly inferred that the real could pay in cash as soon as Sergio could
intention of the parties is simply to get the documentation ready, Sergio
secure the payment of a debt or the decided to withdraw his offer to Marcelo,
performance of any other obligation. hoping to just explain matters to his
The present transaction was clearly friend. Marcelo, however, objected when
intended to just secure the shortage the withdrawal was communicated to him,
incurred by Eulalia because Bandung taking the position that they have a firm
remained in possession of the and binding agreement that Sergio cannot
property inspite of the execution of simply walk away from because he has an
the sale. option to buy that is duly supported by a
duly accepted valuable consideration.

Does Marcelo have a cause of action


Option Contract; Liquor & “Pulutan”
against Sergio? (5%)
as Consideration (2013)

SUGGESTED ANSWER:
No.III.Sergio is the registered owner of a
500-square meter land. His friend, Yes. Marcelo has a cause of action
Marcelo, who has long been interested in against Sergio.
the property, succeeded in persuading
Sergio to sell it to him. On June 2, 2012, Under Art. 1324, when the offerer has
they agreed on the purchase price of allowed the offeree a certain period
P600,000 and that Sergio would give to accept, the offer may be
Marcelo up to June30, 2012 within which withdrawn at any time before
to raise the amount. Marcelo, in a light acceptance by communicating such
tone usual between them, said that they withdrawal, except when the option
should seal their agreement through a is founded upon consideration, as
case of Jack Daniels Black and P5,000 something paid or promised.
"pulutan" money which he immediately
handed to Sergio and which the latter An accepted unilateral promise to
accepted. The friends then sat down and buy or sell a determinate thing for a
drank the first bottle from the case of price certain is binding upon him if
bourbon. the promise is supported by a
consideration distinct from the price
On June 15, 2013, Sergio learned of (Art. 1479).
another buyer, Roberto, who was

“Never Let The Odds Keep You From Pursuing What You Know In Your Heart You Were Meant To Do.”-Leroy Satchel Paige
Page 100 of 180
Civil Law Q&As (2007-2013) hectorchristopher@yahoo.com dbaratbateladot@gmail.com

Consideration in an option contract The Statute of Frauds covers an


may be anything of value,, unlike in agreement for the sale of real
sale where it must be the price property or of an interest therein.
certain in money or its equivalent Such agreement is unenforceable by
(San Miguel Properties Inc. v. action, unless the same, or some note
Spouses Huang, G.R. No. 137290, July or memorandum, thereof, be in
31, 2000). writing, (Art. 1403 (e), Civil Code).
Here, Marcelo and Sergio merely
Here, the case of Jack Daniels Black entered into an Option Contract,
and the P5,000.00 “pulutan” money which refers to a unilateral promise
was a consideration to “seal their to buy or sell, which need not be in
agreement,” an agreement that writing to be enforceable (Sanchez v.
Marcelo is given until June 30, 2012 Rigos, G.R. No. L-25494, June 14,
to buy the parcel of land. There is 1972, citing Atkins, Kroll and Co. Inc.
also no showing that such v. Cua Hian Tek and Southwestern
consideration will be considered part Sugar & Molasses Co. v. Atlantic Gulf
of the purchase price. Thus, Sergio’s & Pacific Co.).
unilateral withdrawal of the offer
violated the Option Contract between ALTERNATIVE ANSWER:
him and Marcelo.
No. Sergio’s claim has no legal basis.
Can Sergio claim that whatever they might
have agreed upon cannot be enforced The contract of sale has already been

because any agreement relating to the partially executed which takes it

sale of real property must be supported by outside the ambit of the Statute of

evidence in writing and they never Frauds is applicable only to executory

reduced their agreement to writing? (3%) contracts, not to contracts that are
totally or partially performed
SUGGESTED ANSWER: (Carbonnel v. Poncio, G.R. No. L-
11231, May 12, 1958).
No. Sergio’s claim has no legal basis.

The contract at issue in the present


case is the option contract, not the Right of First Refusal; Lessee; Effect
contract of sale for the real property. (2008)
Therefore, Art. 1403 does not apply.
No.XVI. Dux leased his house to Iris for a
period of 2 years, at the rate of
P25,000.00
“Never Let The Odds Keep You From Pursuing What You Know In Your Heart You Were Meant To Do.”-Leroy Satchel Paige
Page 101 of 180
Civil Law Q&As (2007-2013) hectorchristopher@yahoo.com dbaratbateladot@gmail.com

monthly, payable annually in advance. refusal. This makes the mother a


The contract stipulated that it may be buyer in bad faith, hence giving more
renewed for another 2-year period upon ground for rescission of the sale to
mutual agreement of the parties. The her (Equatorial Realty, et al. v.
contract also granted Iris the right of first Mayfair Theater, G.R. No. 106063, 21
refusal to purchase the property at any Nov. 1996).
time during the lease, if Dux decides to
sell the property at the same price that ALTERNATIVE ANSWER:

the property is offered for sale to a third


No, Iris cannot seek rescission of the
party. Twenty-three months after execution
sale of the property to Dux’s mother
of the lease contract, Dux sold breach of
because the sale is not one of those
her right of first refusal. Dux said there
rescissible contracts under Art. 1381
was no breach because the property was
of the Civil Code.
sold to his mother who is not a third party.
Iris filed an action to rescind the sale and
(B). Will the alternative prayer for
to compel Dux to sell the property to her
extension of the lease prosper? (2%)
at the same price. Alternatively, she asked
the court to extend the lease for another 2 SUGGESTED ANSWER:
years on the same terms.
No. The contract stipulated that it
(A). Can Iris seek rescission of the sale of may be renewed for another 2-year
the property to Dux's mother? (3%) period upon mutual agreement of the
parties. Contracts are binding
SUGGESTED ANSWER:
between the parties; validity or
compliance cannot be left to the will
Yes, because the right of first refusal
of one of the parties (Art. 1308, Civil
is included in the contract signed by
Code).
the parties. Only if the lessee failed
to exercise the right of first refusal
ALTERNATIVE ANSWER:
could the lessor lawfully sell the
subject property to others, under no It depends. The alternative prayer for
less than the same terms and the extension of the lease may
conditions previously offered to the prosper if
lessee. Granting that the mother is there is a stipulation in the contract
not a third party, this would make her of sale; (b) Dux's mother is aware of
privy to the agreement of Dux and the existing contract of lease; or (c)
Iris, aware of the right of first the lease is recorded in the Registry
of Property (Art. 1676, Civil Code).

“Never Let The Odds Keep You From Pursuing What You Know In Your Heart You Were Meant To Do.”-Leroy Satchel Paige
Page 102 of 180
Civil Law Q&As (2007-2013) hectorchristopher@yahoo.com dbaratbateladot@gmail.com

Lease reimbursed the value of the improvements


he introduced. (4%)

Builder; Good Faith; Useful


SUGGESTED ANSWER:
Improvements (2013)

Boboy’s claim that he is a builder in


No.IV.Anselmo is the registered owner of a
good faith has no basis. A builder in
land and a house that his friend Boboy
good faith is someone who occupies
occupied for a nominal rental and on the
the property in concept of an owner.
condition that Boboy would vacate the
The provisions on builder-planter-
property on demand. With Anselmo's
sower under the Civil Code cover
knowledge, Boboy introduced renovations
cases in which the builder, planter
consisting of an additional bedroom, a
and sower believe themselves to be
covered veranda, and a concrete block
owners of the land, or at least, to
fence, at his own expense.
have a claim of title thereto.

Subsequently, Anselmo needed the


As Boboy is a lessee of the property,
property as his residence and thus asked
even if he was paying nominal rental,
Boboy to vacate and turn it over to him.
Art. 1678, Civil Code, is applicable.
Boboy, despite an extension, failed to
Under this provision, if the lessee
vacate the property, forcing Anselmo to
makes, in good faith, useful
send him a written demand to vacate.
improvements which are suitable to

In his own written reply, Boboy signified the use for which the lease is

that he was ready to leave but Anselmo intended, without altering the form

must first reimburse him the value of the or substance of the property leased,

improvements he introduced on the the lessor upon the termination of

property as he is a builder in good faith. the lease, shall pay the lessee one-

Anselmo refused, insisting that Boboy half of the value of improvements at

cannot ask for reimbursement as he is a that time. Should the lessor refuse to

mere lessee. Boboy responded by reimburse said amount,

removing the improvements and leaving the lessee may remove the

the building in its original state. improvements, even though the


principal thing may suffer damage
(IVa) Resolve Boboy's claim that as a thereby.
builder in good faith, he should be

“Never Let The Odds Keep You From Pursuing What You Know In Your Heart You Were Meant To Do.”-Leroy Satchel Paige
Page 103 of 180
Civil Law Q&As (2007-2013) hectorchristopher@yahoo.com dbaratbateladot@gmail.com

(IVb) Can Boboy be held liable for lease contracts between Jude and his
damages for removing the improvements tenants? Explain your answer. (3%)
over Anselmo's objection? (4%)
SUGGESTED ANSWER:
SUGGESTED ANSWER: Yes, Ildefonso must respect the lease
contracts between Jude and his
No. Boboy cannot be held liable for tenants. While it is true that the said
damages. lease contracts were not registered
and annotated on the title to the
The lessor, Anselmo, refused to
property, Ildefonso is still not an
reimburse one-half of the value of the
innocent purchaser for value. He
improvements, so the lessee, Boboy,
ought to know the existence of the
may remove the same, even though
lease because the building was
the principal thing may suffer
already occupied by the tenants at
damage thereby. If in removing the
the time he bought it. Applying the
useful improvements Boboy caused
principle of caveat emptor, he should
more impairment in the property
have checked and known the status
leased than is necessary he will be
of the occupants of their right to
liable for damages (Art. 1678, Civil
occupy the building before buying it.
Code).

Agency
Lease; Caveat Emptor (2009)
Agency; Sale of a Real Property
through an Agent (2010)
No.VIII. Jude owned a building which he
had leased to several tenants. Without
No.XVI. X was the owner of an
informing his tenants, Jude sold the
unregistered parcel of land in Cabanatuan
building to Ildefonso. Thereafter, the latter
City. As she was abroad, she advised her
notified all the tenants that he is the new
sister Y via overseas call to sell the land
owner of the building. Ildefonso ordered
and sign a contract of sale on her behalf.
the tenants to vacate the premises within
thirty Y thus sold the land to B1 on March 31,
days from notice because he had other 2001 and executed a deed of absolute
plans for the building. The tenants refused sale on behalf of X. B1 fully paid the
to vacate, insisting that they will only do purchase price.
so when the term of their lease shall have
expired. Is Ildefonso bound to respect the

“Never Let The Odds Keep You From Pursuing What You Know In Your Heart You Were Meant To Do.”-Leroy Satchel Paige
Page 104 of 180
Civil Law Q&As (2007-2013) hectorchristopher@yahoo.com dbaratbateladot@gmail.com

B2, unaware of the sale of the land to B1, double sales of an immovable
signified to Y his interest to buy it but property, the ownership shall pertain
asked Y for her authority from X. Without to the person who is in good faith
informing X that she had sold the land to was first in possession and in the
B1, Y sought X for a written authority to absence thereof to the person who
sell. presents the oldest title, provide
there is good faith.
X e-mailed Y an authority to sell the land.
Y thereafter sold the land on May 1, 2001 In a case, the Supreme Court has held

to B2 on monthly installment basis for two that in a sale of real estate the

years, the first installment to be paid at execution of a notarial document of

the end of May 2001. sale is tantamount to delivery of the


possession of the property sold. The
Who between B1 and B2 has a better right ownership of the land therefore
over the land? Explain. (5%) pertains to the first buyer. It may also
be mentioned that under Art 3344 no
SUGGESTED ANSWER: instruments or deed
establishing,
B-2 has a better title. This is not a
case of double sale. Since the first transmitting,

sale was void. The law provides that acknowledging, modifying, or


when a sale of a piece of land or any extinguishing right to real property
interest therein is through an agent, not registered under Act 496 shall be
the authority of the latter shall be in valid except as between the parties.
writing; otherwise, the sale shall be Thus, the Deed of Sale of B-2 has no
void (Art 1874, NCC). The property binding effect on B-1.
was sold by Y to B1 wihtout any
written authority from the owner X. Partnership
Hence, the sale to B1 was void.
Liability; Liability of a Partner (2010)
ALTERNATIVE ANSWER:
No.XV. A, B, and C entered into a
Under the facts, B-1 has a better
partnership to operate a restaurant
right to the land. Given the fact that
business. When the restaurant had gone
the Deed of Sale in favor of B-1 and
past break-even stage and started to
B-2 are not inscribed in the Registry
garner considerable profits, C died. A and
of Deeds, the case is governed by Art
B continued the business without
1544 of the New Civil Code which
dissolving the partnership. They in fact
provides that in case of
opened a branch of the restaurant,
incurring obligations in the process.
Creditors started
“Never Let The Odds Keep You From Pursuing What You Know In Your Heart You Were Meant To Do.”-Leroy Satchel Paige
Page 105 of 180
Civil Law Q&As (2007-2013) hectorchristopher@yahoo.com dbaratbateladot@gmail.com

demanding for the payment of their individual properties shall be subject


obligations. first to the payment of his separate
debts (Art 1835. NCC).
(A). Who are liable for the settlement of
the partnership’s obligations? Explain?
(3%)
Oral Partnership (2009)
SUGGESTED ANSWER:
No.I. TRUE or FALSE. Answer TRUE if the
The two remaining partners, A and B, statement is true, or FALSE if the
are liable. When any partner dies and statement is false. Explain your answer in
the business is continued without any not more than two (2) sentences.
settlement of accounts as between
him or his estate, the surviving (C). An oral partnership is valid. (1%)
partners are held liable for continuing
the business despite the death of C SUGGESTED ANSWER:
(Art 1841, 1785, par 2, and Art 1833 TRUE. Partnership is a consensual
of NCC). contract, hence, it is valid even
though not in writing.
(B).What are the creditors’ recourse/s?
Explain. (3%) ALTERNATIVE ANSWER:
TRUE. An oral is a consensual of the
SUGGESTED ANSWER:
partnership is valid even though not
in writing. However, If it involves
Creditors can file the appropriate
contribution of an immovable
actions, for instance, an action for
property or a real right, an oral
collection of sum of money against
contract of partnership is void. In
the “partnership at will” and if there
such a case, the contract of
are no sufficient funds, the creditors
partnership to be valid, must be in a
may go after the private properties of
public instrument ( Art. 1771 ,NCC ),
A and B (Art 816, NCC). Creditors may
and the inventory of said property
also sue the estate of C. The estate is
signed by the parties must be
not excused from the liabilities of the
attached to said public instrument
partnership even if C is dead already
(Art. 1773, NCC).
but only up to the time that he
remained a partner (Art 1829, 1835,
ALTERNATIVE ANSWER:
par 2, NCC; Testate Estate of Mota v.
Serra, 47 Phil 464 [1925]). However,
the liability of C’s

“Never Let The Odds Keep You From Pursuing What You Know In Your Heart You Were Meant To Do.”-Leroy Satchel Paige
Page 106 of 180
Civil Law Q&As (2007-2013) hectorchristopher@yahoo.com dbaratbateladot@gmail.com

TRUE. Partnership is a consensual Yes, he is not entitled to the return of


contract, hence, it is valid even his contribution to the capital of the
though not in writing. The oral partnership, but only to the net
contract of partnership is also valid profits from the partnership business
even if an immovable property or real during the life of the partnership
right is contributed thereto. While period. If he is a limited partner,
the law, in such a case, requires the however, he may ask for the return of
partnership to be in a public his contributions as provided in Art
document, the law does not expressly 1856 and 1857, Civil Code.
declare the contract void if not
executed in the required form (Article Commodatum & Mutuum
1409 (7 ,NCC ). And there being
Mutuum; Interest; Solutio Indebiti
nothing in the law from which it can
(2012)
be inferred that the said requirement
is prohibitory or mandatory (Article 5, No.VI.a) Siga-an granted a loan to
NCC), the said oral contract of Villanueva in the amount of P 540, 000.00.
partnership must also be valid. The Such agreement was not reduced to
interested party may simply require writing. Siga-an demanded interest which
the contract to be made into a public was paid by Villanueva in cash and
document in order to comply with the checks. The total amount Villanueva paid
required form (Article 1357, NCC). accumulated to P 1, 200, 000.00. Upon
The purpose of the law in requiring a advice of her lawyer, Villanueva
public document is simply to notify demanded for the return of the excess
the public about the contribution. amount of P 660, 000.00 which was
ignored by Siga-an.

Share; Demand during the Existence Is the payment of interest valid? Explain.
of Partnership (2012) (3%)

No.X.b) A partner cannot demand the SUGGESTED ANSWER:


return of his share (contribution) during
the existence of a partnership. Do you No, Art. 1956, Civil Code, provides
agree? Explain your answer. (5%) that “no interest shall be due unless
it has been expressly stipulated in
SUGGESTED ANSWER: writing.”

Is solution indebiti applicable? Explain.


(2%)

“Never Let The Odds Keep You From Pursuing What You Know In Your Heart You Were Meant To Do.”-Leroy Satchel Paige
Page 107 of 180
Civil Law Q&As (2007-2013) hectorchristopher@yahoo.com dbaratbateladot@gmail.com

SUGGESTED ANSWER: enforceable ( Article 1403 [2] b,


NCC).The validity of the contract
Yes, Solutio Indebiti is should be distinguished from its
applicable
enforceability .
because Villanueva Overpaid by
P600,000.00 representing interest Surety
payment which is not due. He can,
therefore, demand its return. Surety (2010)

No.III. Define, Enumerate or Explain.


Guaranty
(2% each)

Guaranty (2009)
(A). What is the difference between
"guaranty" and "suretyship"?
No.I. TRUE or FALSE. Answer TRUE if the
statement is true, or FALSE if the
SUGGESTED ANSWER:
statement is false. Explain your answer in
not more than two (2) sentences. Guaranty and Suretyship
distinguished

(D). An oral promise of guaranty is valid


(1)The obligation in guaranty is
and binding. (1%)
secondary; whereas, in suretyship, it
is primary.
SUGGESTED ANSWER :
In guranty, the undertaking is to pay
FALSE. An oral contract of guaranty, if the principal debtor cannot pay;
being a special promise to answer for whereas, in suretyship, the
the debt of undertaking is to pay if the principal
another, is unenforceable unless in debtor does not pay .
writing (Article 1403 [2] b, NCC ).
In guranty, the guarantor is entitled
to the benefit of excussion; whereas,
ALTERNATIVE ANSWER:
in
TRUE. An oral promise of guaranty is
suretyship the surety is not
valid and binding. While the contract
entitled.
is valid, however ,it is unenforceable
because it is not writing . Being a Liability in guaranty depends upon
special promise answer for the debt, an independent agreement to pay the
or miscarriage of another, the Statute obligations of the principal if he fails
of Frauds requires it to be in writing to do so; whereas, in suretyship, the
to be surety assumes liability as a regular
party.

“Never Let The Odds Keep You From Pursuing What You Know In Your Heart You Were Meant To Do.”-Leroy Satchel Paige
Page 108 of 180
Civil Law Q&As (2007-2013) hectorchristopher@yahoo.com dbaratbateladot@gmail.com

(5)The Guarantor insures the (B). Will your answer to [a] be the same if
solvency of the principal debtor; the contract stipulates that upon failure of
whereas, the surety insures the debt. Rosario to redeem the ring on due date,
Jennifer may immediately sell the ring and
(6)In a guaranty, the guarantor is
appropriate the entire proceeds thereof for
subsidiarlty liable; whereas, in a
herself as full payment of the loan?
suretyship, the surety binds himself
Reasons. (3%)
solidarity with the principal debtor
(Art 2047, Civil Code). SUGGESTED ANSWER:
No, my answer will be different. While
Pledge
the contract of pledge is valid, the
stipulation authorizing the pledgee to
Pledge; Pactum Commissorium (2009)
immediately sell the thing pledged is
void under Art 2088 of the New Civil
No.XVII. Rosario obtained a loan of
Code, which provides that “the
P100,000.00 from Jennifer, and pledged
creditor cannot appropriate the
her diamond ring. The contract signed by
things given by way of pledge or
the parties stipulated that if Rosario is
mortgage, or dispose of them xxx.”
unable to redeem the ring on due date,
Jennifer cannot immediately sell by
she will execute a document in favor of
herself the thing pledged. It must be
Jennifer providing that the ring shall
foreclosed by selling it at a public
automatically be considered full payment
auction in accordance with the
of the loan.
procedure under Art 2112 of the New

(A). Is the contract valid? Explain. (3%) Civil Code.

Torts and Damages


SUGGESTED ANSWER:
The contract is valid because Rosario Damages (2012)
has to execute a document in favor of
Jennifer to transfer the ownership of No.I. a) Roberto was in Nikko Hotel when
the pledged ring to the latter. The he bumped into a friend who was then on
contract her way to a wedding reception being held
does not amount to pactum in said hotel. Roberto alleged that he was
commissorium because it does not then invited by his friend to join her at the
provide for the automatic wedding reception and carried the basket
appropriation by the pledgee of the full of fruits which she was bringing to the
thing pledged in case of default by affair. At the reception, the wedding
the pledgor.

“Never Let The Odds Keep You From Pursuing What You Know In Your Heart You Were Meant To Do.”-Leroy Satchel Paige
Page 109 of 180
Civil Law Q&As (2007-2013) hectorchristopher@yahoo.com dbaratbateladot@gmail.com

coordinator of the hotel noticed him and prosper. Otherwise, Roberto’s action
asked him, allegedly in a loud voice, to will not prosper.
leave as he was not in the guest list. He
The hotel is liable for the wrongful
retorted that he had been invited to the
acts of its employees.
affair by his friend, who however denied
doing so. Deeply embarrassed by the COMMENT:
incident, Roberto then sued the hotel for
damages under Articles 19 and 21 of the The facts of the problem are almost

Civil Code. Will Roberto’s action prosper? similar to the facts of Nikko Hotel

Explain. (5%) Manila Garden v. Reyes, G.R. No.


154259, Feb 28, 2005. In the said
SUGGESTED ANSWER: case, however, there is a categorical
finding that the hotel employee did
No. Roberto’s action will not prosper.
not, exposed the complainant to the
From the facts given in the problem,
ridicule, shame or embarrassment;
the wedding coordinator did not
hence, did not commit any abuse of
abuse her right when she asked him
right. The present problem makes no
to leave the wedding reception
statement of that finding. In the
because he was not in the guest list.
contrary, the problem states that it is
Hotel Nikko could not be held liable
a mere allegation.
for damages as its liable spring from
the liability of its employee (Nikko
Hotel Manila Garden v. Reyes, G.R.
No. 154259, Feb 28, 2005). Damages; Moral & Exemplary (2009)

ALTERNATIVE ANSWER: No.XIV. Rodolfo, married to Sharon, had an


illicit affair with his secretary, Nanette, a
It depends. While the hotel has the
19-year old girl, and begot a baby girl,
right to exclude an uninvited guest
Rona. Nanette sued Rodolfo for damages:
from the wedding reception, that
actual, for hospital and other medical
does not give the hotel the license to
expenses in delivering the child by
humiliate Roberto. If the wedding
caesarean section; moral, claiming that
coordinator of the hotel acted
Rodolfo promised to marry her,
wrongfully e.g. with the abuse of
representing that he was single when, in
right, unfairly, or in a matter that
fact, he was not; and exemplary, to teach
exposed Roberto to unnecessary
a lesson to like-minded Lotharios.
ridicule or shame, his action will

“Never Let The Odds Keep You From Pursuing What You Know In Your Heart You Were Meant To Do.”-Leroy Satchel Paige
Page 110 of 180
Civil Law Q&As (2007-2013) hectorchristopher@yahoo.com dbaratbateladot@gmail.com

(A). If you were the judge, would you Vinzons-Chato filed a Motion to Dismiss
award all the claims of Nanette? Explain. arguing that she cannot be held liable for
(3%) damages for acts she performed while in
the discharge of her duties as BIR
SUGGESTED ANSWER: Commissioner. Is she correct? Explain.
If Rodolfo's marriage could not have (5%)
been possibly known to Nanette or
there is no gross negligence on the SUGGESTED ANSWER:
part of Nanette, Rodolfo could be
Yes. As a general rule, a public officer
held liable for moral damages.
is not liable for acts performed in the
discharge of his duties. The
If there is gross negligence in a suit
exceptions are when he acted with
for quasi-delict, exemplary could be
malice, bad faith, or gross negligence
awarded.
in the performance of his duty, or
when his act is in violation of a
Constitutional guaranteed right and
Damages; Public Officers acting in the liberties of a person under Art32 of
Performance of their Duties (2012) the NCC.

No.II.a) Liwayway Vinzons-Chato was then The public officer is not automatically

the Commissioner of Internal Revenue considered to have violated the

while Fortune Tobacco Corporation is an rights or liberties of a person simply

entity engaged in the manufacture of because the rule the public officer

different brands of cigarettes, among issued was declared invalid by the

which are "Champion," "Hope," and "More" court. The complainant must still

cigarettes. allege and prove the particular injury


or prejudice he has suffered from the
Fortune filed a complaint against Vinzons- violation of his constitutional right by
Chato to recover damages for the alleged the issuance of the invalidated rule.
violation of its constitutional rights arising
The problem does not state any fact
from Vinzons-Chato’s issuance of Revenue
from which any malice, bad faith or
Memorandum Circular No. 37-934 (which
gross negligence on the part of
re-classified Fortune cigarettes as locally
Vinzons-Chato may be inferred, or the
manufactured with foreign brands and
particular injury or prejudice the
thereby imposed higher taxes), which the
complainant may have suffered as a
Supreme Court later declared invalid.
result of the violation of his
constitutional right. Hence, she

“Never Let The Odds Keep You From Pursuing What You Know In Your Heart You Were Meant To Do.”-Leroy Satchel Paige
Page 111 of 180
Civil Law Q&As (2007-2013) hectorchristopher@yahoo.com dbaratbateladot@gmail.com

cannot be held liable. The facts the driver and the defense of
presented are similar to facts of the diligence is not available.
case of Vinzons-Chato v. Fortune, G.R.
No. 141309, Dec 23, 2008. (B).Would your answer be the same if
Rommel was in the car at the time of the
accident? Explain. (2%)

Death Indemnity (2009) SUGGESTED ANSWER:


Yes, my answer would be the same.
No. X. Rommel’s private car, while being Rommel, who was in the car, shall be
driven by the regular family driver, liable for damages if he could have
Amado, hits a pedestrian causing the prevented the misfortune by the use
latter’s death. Rommel is not in the car of due diligence in supervising his
when the incident happened. driver but failed to exercise it (Art.
2184, NCC). In such case, his liability
(A). Is Rommel liable for damages to the
is solidary with his driver.
heirs of the deceased? Explain. (2%)
ALTERNATIVE ANSWER:
Yes, my answer will be the same
SUGGESTED ANSWER:
except that in such case the liability
Yes, Rommel may be held liable for
of the owner is not presumed. When
damages if he fails to prove that he
the owner is inside the vehicle, he
exercised the diligence of a good
becomes liable only when it is shown
father of a family (Art. 2180, par 5,
that he could have prevented the
NCC) in selecting and supervising his
misfortune by the use of due
family driver. The owner is presumed
diligence (Art. 2184, NCC). For the
liable unless he proves the defense of
owner to be held liable, the burden of
diligence. If the driver was
proving that he could have prevented
performing his assigned task when
the misfortune rests on the shoulder
the accident happened, Rommel shall
of the victim.
be solidarily liable with the driver.

In case the driver is convicted of


reckless imprudence and cannot pay Doctrine of Discovered Peril (Last
the civil liability, Rommel is Clear Chance) (2007)
subsidiarily liable for the damage
awarded against No.III. Explain the following concepts and
doctrines and give an example of each:

“Never Let The Odds Keep You From Pursuing What You Know In Your Heart You Were Meant To Do.”-Leroy Satchel Paige
Page 112 of 180
Civil Law Q&As (2007-2013) hectorchristopher@yahoo.com dbaratbateladot@gmail.com

(B). doctrine of discovered peril (last clear typhoon knocked down the fence of the
chance) (5%) pond and the iguana crawled out of the
gate of Primo’s residence. N, a neighbor
SUGGESTED ANSWER: who was passing by, started throwing
stones at the iguana, drawing the iguana
The doctrine of last clear chance
to move toward him. N panicked and ran
states that where the plaintiff was
but tripped on something and suffered a
guilty of prior or antecedent
broken leg.
negligence, but the defendant, who
had the ultimate opportunity to avoid Is anyone liable for N’s injuries? Explain.
the impending harm failed to do so, it (4%)
is the defendant who is liable for all
the consequences of the accident SUGGESTED ANSWER:
notwithstanding the prior negligence
No one is liable. The possessor of an
of the plaintiff. An example is where a
animal or whoever may make use of
person was riding a pony on a bridge
the same is responsible for the
and improperly pulled the pony to the
damage it may cause, although it may
wrong side when he saw a car
escape or be lost. This responsibility
coming. The driver of the car did not
shall cease only in case the damage
stop or change direction, and nearly
should come from force majeure or
hit the horse, and, the frightened
from the fault of the person who has
animal jumped to its death. The
suffered damage (Art 2183, NCC).
driver of the car is guilty of
negligence because he had a fair
opportunity to avoid the accident and
failed to avail himself of that
Liability; Special Parental Authority
opportunity. He is liable under the
(2010)
doctrine of last clear chance (Picart v.
Smith, 37 Phil. 809, 1918). No.XII. On May 5, 1989, 16-year old
Rozanno, who was issued a student
permit, drove to school a car, a gift from
Liability; Owner of a Pet; Fortuitous his parents. On even date, as his class was
Event (2010) scheduled to go on a field trip, his teacher
requested him to accommodate in his car,
No.XIV. Primo owns a pet iguana which he as he did, four (4) of his classmates
keeps in a man-made pond enclosed by a
fence situated in his residential lot. A

“Never Let The Odds Keep You From Pursuing What You Know In Your Heart You Were Meant To Do.”-Leroy Satchel Paige
Page 113 of 180
Civil Law Q&As (2007-2013) hectorchristopher@yahoo.com dbaratbateladot@gmail.com

because the van rented by the school was (B). How about the damage to the
too crowded. On the way to a museum jeepney? Explain. (2%)
which the students were scheduled to
visit, Rozanno made a wrong maneuver, SUGGESTED ANSWER:

causing a collision with a jeepney. One of


With respect to the damages caused
his classmates died. He and the three (3)
to the jeepney, only Rozanno should
others were badly injured.
be held liable because his negligence
or tortuous act was the sole,
(A). Who is liable for the death of
proximate and immediate cause
Rozanno’s classmate and the injuries
thereof.
suffered by Rozanno and his 3 other
classmates? Explain. (2%)
(C). Under the same facts, except the date
of occurrence of the incident, this time in
SUGGESTED ANSWER:
mid-1994, what would be your answer?
At the time the incident occurred in Explain. (2%)
May 1989, Rozanno was still a minor.
Being a minor, Art 218 of the Family SUGGESTED ANSWER:

Code applies. Pursuant to Art 218,


Since Rozanno was 16 years old in
the school, its administrators and
1989, if the incident happened
teachers shall be liable for the acts of
sometime in the middle of 1994,
minor Rozanno because of the special
Rozanno have been 21 years old at
parental authority and responsibility
the time. Hence, he was already of
that they exercise over him. The
legal age. The law reducing the age
authority applies to all authorized
of majority to 18 years took effect in
activities, whether inside or outside
December 1989.
the premises of the school, entity or
institution. The field trip on which Being of legal age, articles 218, 219,
occasion Rozanno drove the car, was and 221 of the Family Code are no
an authorized activity, and , thus, longer applicable. In such case, only
covered by the provision. Rozanno will be personally
Furthermore, the parents of Rozanno responsible for all the consequences
are subsidiarily liable pursuant to Art of his act unless his school or his
219 (FC), and principally liable under parents were themselves also
Art 221 (FC), if they are negligent. negligent and such negligence
contributed to the happening of the
incident. In that event, the school or
his parents are not liable under Art
218, 218 or 221 of the Family Code,
but will be
“Never Let The Odds Keep You From Pursuing What You Know In Your Heart You Were Meant To Do.”-Leroy Satchel Paige
Page 114 of 180
Civil Law Q&As (2007-2013) hectorchristopher@yahoo.com dbaratbateladot@gmail.com

liable under general provision on the court to justify the damages that your
Civil Code on quasi-delict. client claims? (8%)

SUGGESTED ANSWER:

Quasi-Delict; Claims; Requisites


(2013) I will the base the claim of my client
on quasi-delict under Art 2176 of the
No.II. A collision occurred at an Civil Code of the Philippines. The
intersection involving a bicycle and a requisites for a claim under quasi-
taxicab. Both the bicycle rider (a delict to prosper are as follows:
businessman then doing his morning
exercise) and the taxi driver claimed that Act or omission, there being fault or

the other was at fault. Based on the police negligence;

report, the bicycle crossed the intersection


Damage or injury; and
first but the taxicab, crossing at a fast clip
from the bicycle's left, could not brake in
Causal connection between the
time and hit the bicycle's rear wheel,
damage and the act or omission.
toppling it and throwing the bicycle rider
into the sidewalk 5 meters away. The case clearly involves quasi-delict
where my client, the bicycle rider,
The bicycle rider suffered a fractured right
suffered injury as a result of the
knee, sustained when he fell on his right
negligence of the over-speeding taxi
side on the concrete side walk. He was
driver, without fault on my client’s
hospitalized and was subsequently
part.
operated on, rendering him immobile for 3
weeks and requiring physical rehabilitation To prove actual damages aside from
for another 3 months. In his complaint for the testimony of client, I will present
damages, the rider prayed for the award his hospital and medical bills.
ofP1,000,000 actual damages,P200,000 Receipts paid on the rehabilitation
moral damages, P200,000 exemplary will also be presented. [The sentence
damages, P1 00,000 nominal damages in red should be replaced with the
and P50,000 attorney's fees. following sentence because he is a
businessman and not an employee.
Assuming the police report to be correct
Furthermore, I will present income
and as the lawyer for the bicycle rider,
tax returns, contracts and other
what evidence (documentary and
documents to prove unrealized
testimonial) and legal arguments will you
profits as a result of this temporary
present in
injury.] I will
“Never Let The Odds Keep You From Pursuing What You Know In Your Heart You Were Meant To Do.”-Leroy Satchel Paige
Page 115 of 180
Civil Law Q&As (2007-2013) hectorchristopher@yahoo.com dbaratbateladot@gmail.com

also call the attending physician to fraudulent, reckless, oppressive, or


testify as to the extent of the injuries malevolent manner. While the amount
suffered by my client, and to of exemplary damages may not be
corroborate the contents of the proved, the plaintiff must show that
medical documents. he is entitled to moral or
compensatory damages. In support of
Based on Art. 2202, in quasi-delicts, this, I will present the police report
the defendant shall be liable for all showing the circumstance under
damages which are the natural and which the accident took place, taking
probable consequences of the act or into account the actions of the
omission complained of. It is not parties. I will ask the officials who
necessary that the damages have responded to the accident to testify
been foreseen or could have been as to the conduct of the parties at the
foreseen by the defendant. time of the accident in order to
determine whether defendant was
Unlike actual damages, no proof of
guilty of gross negligence.
pecuniary loss is necessary in order
that moral, nominal, temperate Finally, attorney’s fees may be
liquidated or exemplary damages recovered when exemplary damages
may be adjudicated. The assessment are awarded (Art 2208, Civil Code).
is left to the discretion of the Court
(Art. 2216, Civil Code). There must be
proof pecuniary estimation, however.
Quasi Tort (2010)
Moral damages can be recovered by
my client under Articles 2219 and No.III. Define, Enumerate or Explain.

2200. Moral damages may be (2% each)

recovered in case of a quasi-delict


(B). Define quasi tort. Who are the persons
causing physical injuries.
liable under quasi torts and what are the
Additionally, it must be proved that
defenses available to them?
such damages were the proximate
result of the act complained of.
Note: It is recommended that the examiner
Medical certificates will be presented,
exercise leniency and liberality in grading
along with the testimony from my
the answers given to this question. The term
client and other eyewitness accounts,
quasi-tort is not part of legal developments
in order to support the award for
in civil law. In Philippine legal tradition,
moral damages.
quasi-

Exemplary damages may be granted


if the defendant acted in wanton,
“Never Let The Odds Keep You From Pursuing What You Know In Your Heart You Were Meant To Do.”-Leroy Satchel Paige
Page 116 of 180
Civil Law Q&As (2007-2013) hectorchristopher@yahoo.com dbaratbateladot@gmail.com

delict has been treated as the closest civil who can be held liable and their
law equivalent of the common law tort. In defenses would also apply.
fact, in a number of Supreme Court
Those liable for quasi-delict include:
decisions, the two terms have been
considered synonymous. In reality, Those tortfeasor or the person
however, the common law tort is much causing damage to another through
broader in scope than the civil law quasi- fault or negligence ( Article 2176 NCC
delict. In recent developments in common ); and
law, the concept of “quasi-torts” can be Persons vicariously liable under
considered as the closest common law Article 2180 (NCC ).
equivalent of the civil law concept of
quasi-delict. This is because it is argued
that the growing recognition of quasi-torts The defenses available include:

as a source of obligation is hinged on the


That the defendant was not negligent
acceptance at common law of the civil law
or that he exercised due diligence
principles of quasi-delict.
( Article 2176 NCC );

SUGGESTED ANSWER:
That although the defendant is

Quasi -tort is a legal concept negligent his negligence is not the

upholding the doctrine that some proximate cause of the injury ( Article

legal duty exists that cannot be 2179 NCC );

classified strictly as a personal duty


That the plaintiff's own negligence
(thus resulting in a tort), nor as a
was the immediate and proximate
contractual duty but rather some
cause of his injury ( Article 2179
other kind of duty recognizable by
NCC );
the law. ” Tort “ or ” Quasi-tort” is an
Anglo American or Common Law (d ) That the person vicariously liable
concept, while “Delict” or “Quasi- has observed all the diligence of a
Delict“ is a Civil Law concept good father of a family to prevent
(Wikipedia encyclopedia). damage ( Article 2180 NCC ); and

ALTERNATIVE ANSWER: That the cause of action has


prescribed after the lapse s (Article
Quasi -tort is considered as the
2179 NCC ).
equivalent of quasi-delict. Hence the
rules of the latter pertaining to The fact that the plaintiff had
persons committed contributory negligence is
a partial defense (Art 2179, NCC).

“Never Let The Odds Keep You From Pursuing What You Know In Your Heart You Were Meant To Do.”-Leroy Satchel Paige
Page 117 of 180
Civil Law Q&As (2007-2013) hectorchristopher@yahoo.com dbaratbateladot@gmail.com

MULTIPLE CHOICE per capita, and not by right of


representation (Art 975, Civil Code)
QUESTIONS
I. (2) How much is Dante's share in the net
2013 Taxation Law Exam estate? (1%)
MCQ (October 13, 2013) P150,000.
P200,000.
Armand died intestate. His full-blood P300,000.
brothers, Bobby and Conrad, and half- P400,000.
blood brothers, Danny, Edward and Floro, None of the above.
all predeceased him. The following are the
surviving relatives: SUGGESTED ANSWER:
1. Benny and Bonnie, legitimate children E. None of the above.
of Bobby; There is no showing that Danny is an
2. Cesar, legitimate child of Conrad;
illegitimate half-blood brother of
3. Dante, illegitimate child of Danny; Armand. In the absence of proof to
4. Ernie, adopted child of Edward; and the contrary, the law presumes that
5. Felix, grandson of Floro. the relationship is legitimate. Thus,
The net value of Armand's estate is Dante, an illegitimate child of Danny,
Pl,200,000. is barred from inheriting from
Armand pursuant to the “iron curtain
I. (1) How much do Benny and Bonnie rule” which disqualifies an
stand to inherit by right of representation? illegitimate child from inheriting ab
(1%) intestao from the legitimate children
P200,000 and relatives of his father or mother,
P300,000 and vice versa (Art 992, Civil Code).
P400,000
P150,000 I. (3) How much is Ernie's share in the net
None of the above. estate . (1%)
P 0.
SUGGESTED ANSWER: P400,000.
(E) None of the above. P150,000.
If all the brothers/sisters are P200,000.
disqualified to inherit, the None of the above.
nephews/nieces inherit SUGGESTED ANSWER:

“Never Let The Odds Keep You From Pursuing What You Know In Your Heart You Were Meant To Do.”-Leroy Satchel Paige
Page 118 of 180
Civil Law Q&As (2007-2013) hectorchristopher@yahoo.com dbaratbateladot@gmail.com

(A) 0 or (E) None of the above. Should the share of insolvent debtor C be
The legal relationship created by divided only between the two other
adoption is strictly between the remaining debtors, A and B? (1%)
adopter and the adopted. It does not Yes. Remission of D's share carries with it
extend to the relatives of either party total extinguishment of his obligation to
(Sayson v. CA, G.R. Nos. 89224-25, the benefit of the solidary debtors.
Jan 23, 1992). Yes. The Civil Code recognizes remission
(Note: “E. None of the above’” is as a mode of extinguishing an obligation.
another answer because Ernie has no This clearly applies to D.
share at all in the net estate). No. The rule is that gratuitous acts should
be restrictively construed, allowing only
I. (4) How much is Felix's share in the net the least transmission of rights.
estate? (1%) No, as the release of the share of one
P400,000. debtor would then increase the
P150,000. burden of the other debtors without
P300,000. their consent.
P0.
None of the above. SUGGESTED ANSWER:
(D). No, as the release of the share of
SUGGESTED ANSWER: one debtor would then increase the
(D). 0. Or (E) None of the above. burden of the other debtors without
In the collateral line, representation their consent. When one of the
is granted only to children of brother solidary debtors cannot, because of
or sisters, Felix is a grandson of a his insolvency, reimburse his share to
predeceased brother. the debtor paying the obligation,
(Note: “E. None of the above: is such share shall be borne by all his
another answer because Felix has no co-debtors, in proportion to the debt
share at all in the net estate) of each (Art 1217, Civil Code).
Additionally, D was released only
A, B, C and D are the solidary debtors of X from his share of P10,000.00 not from
for P40,000. X released D from the the solidary tie that binds him to A, B
payment of his share of PI 0,000. When and C.
the obligation became due and
demandable, C turned out to be insolvent. Amador obtained a loan of P300,000 from
Basilio payable on March25, 2012. As
security for the payment of his loan,
Amador constituted a mortgage on his
residential house and lot in Basilio's favor.
“Never Let The Odds Keep You From Pursuing What You Know In Your Heart You Were Meant To Do.”-Leroy Satchel Paige
Page 119 of 180
Civil Law Q&As (2007-2013) hectorchristopher@yahoo.com dbaratbateladot@gmail.com
(Note: “A” is not the correct answer
because it states that “all the assets
Cacho, a good friend of Amador,
of
guaranteed and obligated himself to pay
Basilio, in case Amador fails to pay his
loan at maturity.

(1) If Amador fails to pay Basilio his loan


on March 25, 2012, can Basilio compel
Cacho to pay? (1%)

No, Basilio cannot compel Cacho to pay


because as guarantor, Cacho can invoke
the principle of excussion, i.e., all the
assets of Basilio must first be exhausted.
No, Basilio cannot compel Cacho to
pay because Basilio has not
exhausted the available remedies
against Amador.
Yes, Basilio can compel Cacho to pay
because the nature of Cacho's
undertaking indicates that he has bound
himself solidarily with Amador.
Yes, Basilio can compel Cacho who bound
himself to unconditionally pay in case
Amador fails to pay; thus the benefit of
excussion will not apply.

SUGGESTED ANSWER:
No, Basilio cannot compel Cacho to
pay because Basilio has not
exhausted the available remedies
against Amador.
The guarantor cannot be compelled
to pay the creditor unless the latter
has exhausted all the property of the
debtor and has resorted to all the
legal remedies against the debtor
(Art. 2058, Civil Code)
ownership on the buyer, Diego, who
must therefore consent.
Basilio (the creditor) must first be
No, Basilio cannot foreclose the real estate
exhausted”)
mortgage. To deprive the new owner of
(2) If Amador sells his residential house
ownership and possession is unjust and
and lot to Diego, can Basilio foreclose the
inequitable.
real estate mortgage? (1%)

SUGGESTED ANSWER:
Yes, Basilio can foreclose the real estate
Yes, Basilio can foreclose the real
mortgage because real estate mortgage
estate mortgage. It is binding upon
creates a real right that attaches to the
Diego as the mortgage is embodied in
property.
a public instrument.
Yes, Basilio can foreclose the real
Since the mortgage is in a public
estate mortgage. It is binding upon
instrument, there is constructive
Diego as the mortgage is embodied in
notice to Diego, who is the buyer if
a public instrument.
the mortgaged property.
No, Basilio cannot foreclose the real
estate mortgage. The sale confers
ALTERNATIVE ANSWER:

“Never Let The Odds Keep You From Pursuing What You Know In Your Heart You Were Meant To Do.”-Leroy Satchel Paige
Page 120 of 180
Civil Law Q&As (2007-2013) hectorchristopher@yahoo.com dbaratbateladot@gmail.com

No, Basilio cannot foreclose the real No, Jose's refusal is not justified. The
estate mortgage. The sale confers expenses he incurred are useful for
ownership on the buyer, Diego, who the preservation of the thing loaned.
must therefore consent. It is Jose's obligation to shoulder
The mortgage is not registered, thus, these useful expenses.
cannot be binding against third
persons (Art. 2125, Civil Code) SUGGESTED ANSWER:
No, Jose's refusal is not justified. The
IV. Cruz lent Jose his car until Jose finished expenses he incurred are useful for
his Bar exams. Soon after Cruz delivered the preservation of the thing loaned.
the car, Jose brought it to Mitsubishi It is Jose's obligation to shoulder
Cubao for maintenance check up and these useful expenses.
incurred costs of P8,000. Seeing the car's In commodatum, the bailee is obliged
peeling and faded paint, Jose also had the to pay for the ordinary expenses for
car repainted for P10,000. Answer the two the use and preservation of the thing
questions below based on these common loaned (Art 1941, Civil Code).
facts. The bailee, Jose, has no right of
retention on the ground that the
IV. (1) After the bar exams, Cruz asked for bailor owes him something, even if it
the return of his car. Jose said he would may be by reason of expenses. He
return it as soon as Cruz has reimbursed can only retain it if he suffers
him for the car maintenance and damages by reason of a flaw or
repainting costs of P 18,000. defect in the thing loaned of which
Is Jose's refusal justified? (1%) the bailor knows (Art 1951, Civil
No, Jose's refusal is not justified. In this Code).
kind of contract, Jose is obliged to pay for
all the expenses incurred for the IV. (2) During the bar exam month, Jose
preservation of the thing loaned. lent the car to his girlfriend, Jolie, who
Yes, Jose's refusal is justified. He is obliged parked the car at the Mall of Asia's open
to pay for all the ordinary and parking lot, with the ignition key inside the
extraordinary expenses, but subject to car. Car thieves broke into and took the
reimbursement from Cruz. car.
Yes, Jose's refusal is justified. The principle
of unjust enrichment warrants the Is Jose liable to Cruz for the loss of the car
reimbursement of Jose's expenses. due to Jolie's negligence? (1%)

“Never Let The Odds Keep You From Pursuing What You Know In Your Heart You Were Meant To Do.”-Leroy Satchel Paige
Page 121 of 180
Civil Law Q&As (2007-2013) hectorchristopher@yahoo.com dbaratbateladot@gmail.com

No, Jose is not liable to Cruz as the loss Securities and Exchange Commission,
was not due to his fault or negligence. designated L and 0 as managing partners;
No, Jose is not liable to Cruz. In the L was liable only to the extent of his
absence of any prohibition, Jose could lend capital contribution; and P was not liable
the car to Jolie. Since the loss was due to for losses.
force majeure, neither Jose nor Jolie is In 2006, the partnership earned a net
liable. profit of P800,000. In the same year, P
Yes, Jose is liable to Cruz. Since Jose engaged in a different business with the
lent the car to Jolie without Cruz's consent of all the partners. However, in
consent, Jose must bear the 2007, the
consequent loss of the car. partnership incurred a net loss of
Yes, Jose is liable to Cruz. The contract P500,000. In 2008,the partners dissolved
between them is personal in nature. Jose the partnership. The proceeds of the sale
can neither lend nor lease the car to a of partnership assets were insufficient to
third person. settle its obligation. After liquidation, the
partnership had an unpaid liability
SUGGESTED ANSWER: ofP300,000.
Yes, Jose is liable to Cruz. Since Jose
lent the car to Jolie without Cruz's V. (l) Assuming that the just and equitable
consent, Jose must bear the share of the industrial partner, P, in the
consequent loss of the car. profit in 2006 amounted to P1 00,000, how
The bailee is liable for the loss of the much is the share of 0, a limited partner,
thing, even if it should be through a in the P800,000 net profit? (1%)
fortuitous event if he lends or leases P160,000.
the thing to a third person, who is not P175,000.
a member of his household (Art 1942, P280,000.
Civil Code). P200,000.
None of the above.
V. In 2005, L, M, N, 0 and P formed a
partnership. L, M and N were capitalist SUGGESTED ANSWER:
partners who contributed P500,000 each, (C) P280,000.
while 0, a limited partner, contributed P1 , First, deduct the share of P from the
000,000. P joined as an industrial partner, profits. P800,000 less P100,000 is
contributing only his services. The Articles P700,000. Next, get the share of O by
of Partnership, registered with the following the proportion that the
shares of L, M, N, O is 1:1:1:2,
respectively.

“Never Let The Odds Keep You From Pursuing What You Know In Your Heart You Were Meant To Do.”-Leroy Satchel Paige
Page 122 of 180
Civil Law Q&As (2007-2013) hectorchristopher@yahoo.com dbaratbateladot@gmail.com

No. P is not liable because there is a valid


V. (2) In 2007, how much is the share of 0, stipulation exempting him from losses.
a limited partner, in the net loss of Since the other partners allowed him to
P500,000? (1%) engage in an outside business activity, the
P 0. stipulation absolving P from liability is
P1 00,000. valid. For 0, it is basic that a limited
P125,000. partner is liable only up to the extent of
P200,000. his capital contribution.
None of the above. Yes. The stipulations exempting P and L
from losses are not binding upon the
SUGGESTED ANSWER: creditors. 0 is likewise liable because the
(D) P200,000 partnership was not formed in accordance
A limited partner shall not become with the requirements of a limited
liable a s a general partner unless, in partnership.
addition to the exercise of his rights No. The Civil Code allows the partners to
and powers as a limited partner, he stipulate that a partner shall not be liable
takes part in the control of the for losses. The registration of the Articles
business (Art 1948, Civil Code). In the of Partnership embodying such
absence of stipulation as to profits stipulations serves as constructive notice
and losses, the share of each partner to the partnership creditors.(E) None of
in the losses shall be proportionate to the above is completely accurate.
what he may have contributed (Art None of the above is completely
1797). accurate.

V. (3) Can the partnership creditors hold L, SUGGESTED ANSWER:


0 and Pliable after all the assets of the None of the above is completely
partnership are exhausted? (1%) accurate.
Yes. The stipulation exempting P from VI. Gary is a tobacco trader and also a
losses is valid only among the partners. L lending investor. He sold tobacco leaves to
is liable because the agreement limiting Homer for delivery within a month,
his liability to his capital contribution is not although the period for delivery was not
valid insofar as the creditors are guaranteed. Despite Gary's efforts to
concerned. Having taken part in the deliver on time, transportation problems
management of the partnership, 0 is liable and government red tape hindered his
as capitalist partner. efforts and he could only deliver after 30
days.

“Never Let The Odds Keep You From Pursuing What You Know In Your Heart You Were Meant To Do.”-Leroy Satchel Paige
Page 123 of 180
Civil Law Q&As (2007-2013) hectorchristopher@yahoo.com dbaratbateladot@gmail.com

Homer refused to accept the late delivery No. Homer was not justified in refusing to
and to pay on the ground that the agreed accept the tobacco leaves. There was no
term had not been complied with. term in the contract but a mixed condition.
As lending investor, Gary granted a The fulfillment of the condition did not
Pl,000,000 loan to Isaac to be paid within depend purely on Gary's will but on other
two years from execution of the contract. factors, e.g., the shipping company and
As security for the loan, Isaac promised to the government. Homer should comply
deliver to Gary his Toyota Innova within with his obligation.
seven (7) days, but Isaac failed to do so.
Gary was thus compelled to demand SUGGESTED ANSWER:
payment for the loan before the end of the No. Homer was not justified in
agreed two-year term. refusing to accept the tobacco
leaves. He consented to the terms
VI. (l) Was Homer justified in refusing to and conditions of the sale and must
accept the tobacco leaves? (1%) abide by it. Obligations arising from
Yes. Homer was justified in refusing to contract have the force of law
accept the tobacco leaves. The delivery between the contracting parties.
was to be made within a month. Gary's It is clear under the facts that the
promise of delivery on a "best effort" basis period of delivery of the tobacco
made the delivery uncertain. The term, leaves was not guaranteed. Gary
therefore, was ambiguous. anticipated other factors which may
No. Homer was not justified in prevent him from making the delivery
refusing to accept the tobacco within a month. True enough,
leaves. He consented to the terms transportation problems and
and conditions of the sale and must government red tape did. Such slight
abide by it. Obligations arising from delay was, thus, excusable.
contract have the force of law Obligations arising from contract
between the contracting parties. have the force of law between the
Yes. Homer was justified in his refusal to contracting parties and should be
accept the delivery. The contract complied with in good faith (Art.
contemplates an obligation with a term. 1160, Civil Code)
Since the delivery was made after 30
days, contrary to the terms agreed upon, VI. (2) Can Gary compel Isaac to pay his
Gary could not insist that Homer accept loan even before the end of the two-year
the tobacco leaves. period? (1%)

“Never Let The Odds Keep You From Pursuing What You Know In Your Heart You Were Meant To Do.”-Leroy Satchel Paige
Page 124 of 180
Civil Law Q&As (2007-2013) hectorchristopher@yahoo.com dbaratbateladot@gmail.com

Yes, Gary can compel Isaac to Under Art 1198 (2) of the Civil Code,
immediately pay the loan. Non- the debtor shall lose every right to
compliance with the promised make use of the period when he does
guaranty or security renders the not furnish to the creditor the
obligation immediately demandable. guaranties or securities which he has
Isaac lost his right to make use of the promised.
period.
Yes, Gary can compel Isaac to immediately VII. Lito was a commercial pilot who flew
pay the loan. The delivery of the Toyota for Pacific-Micronesian Air. In 1998, he was
Innova is a condition for the loan. Isaac's the co-pilot of the airline's Flight MA916
failure to deliver the car violated the that mysteriously disappeared two hours
condition upon which the loan was after take-off from Agana, Guam,
granted. It is but fair for Gary to demand presumably over the Pacific Ocean. No
immediate payment. trace of the plane and its 105 passengers
No, Gary cannot compel Isaac to and crew was ever found despite diligent
immediately pay the loan. The delivery of search; Lito himself was never heard of
the car as security for the loan is an again. Lito left behind his wife, Lita, and
accessory contract; the principal contract their two children.
is still the P 1,000,000 loan. Thus, Isaac
can still make use of the period. In 2008, Lita met and married Jaime. They
No, Gary cannot compel Isaac to now have a child of their own.
immediately pay the loan. Equity dictates While on a tour with her former high
that Gary should have granted a school classmates in a remote province of
reasonable extension of time for Isaac to China in 2010, Lita was surprised to see
deliver his Toyota Innova. It would be Lito or somebody who looked exactly like
unfair and him, but she was sure it was Lito because
burdensome for Isaac to pay of the extreme surprise that registered in
the P 1,000,000 simplybecause the
his face when he also saw her. Shocked,
promised security was not delivered.
she immediately fled to her hotel and post
haste returned to the country the next
SUGGESTED ANSWER: day. Lita now comes to you for legal
Yes, Gary can compel Isaac to advice. She asks you the following
immediately pay the loan. Non- questions:
compliance with the promised
guaranty or security renders the VII. (l) If Lito is alive, what is the status of
obligation immediately demandable. his marriage to Lita? (1%)
Isaac lost his right to make use of the
period.

“Never Let The Odds Keep You From Pursuing What You Know In Your Heart You Were Meant To Do.”-Leroy Satchel Paige
Page 125 of 180
Civil Law Q&As (2007-2013) hectorchristopher@yahoo.com dbaratbateladot@gmail.com

The marriage subsists because the marital The marriage is valid. After an absence of
bond has not been terminated by death. more than 10 years, Lito is already
The marriage was terminated when Lita presumed dead for all purposes.
married Jaime. The marriage is void. Lito's mere absence,
The marriage subsists because Lita's however lengthy, is insufficient to
marriage to Jaime is void. authorize Lita to contract a subsequent
The marriage is terminated because Lito is marriage.
presumed dead after his plane has been The marriage is void. If Lito is indeed alive,
missing for more than 4 years. his marriage to Lita was never dissolved
The marriage can be formally declared and they can resume their marital
terminated if Lito would not resurface. relations at any time.

SUGGESTED ANSWER: SUGGESTED ANSWER:


The marriage subsists because Lita's Any answer is correct.
marriage to Jaime is void. Under Art 390 of the Civil Code, after
For the purpose of contracting the an absence of seven years, it being
subsequent marriage under Art 41 of unknown whether or not the
the Family Code, the spouse present absentee still lives, he shall be
must institute a summary proceeding presumed dead doe all purposes,
as provided in the Family Code for except for those of succession. This
the declaration of presumptive death provision was not repealed by the
of the absentee, without prejudice to present Family Code. Applying this to
the effect of the reappearance of the the problem, (A) may be correct. (B)
absent spouse. may also be correct. (C) and
may also be correct under Art 41 of
the Family Code.
VII. (2) If Lito is alive, what is the status of
Lita's marriage to Jaime? (1%) VIII.Which of the following actions or
defenses are meritorious: (1%)
The marriage is valid because Lita's
marriage to Lito was terminated upon An action for recovery of down
Lito's disappearance for more than seven payment paid under a rescinded oral
years. sale of real property.
A defense in an action for ejectment that
the lessor verbally promised to extend or
renew the lease.

“Never Let The Odds Keep You From Pursuing What You Know In Your Heart You Were Meant To Do.”-Leroy Satchel Paige
Page 126 of 180
Civil Law Q&As (2007-2013) hectorchristopher@yahoo.com dbaratbateladot@gmail.com

An action for payment of sum of money


filed against one who orally promised to ALTERNATIVE ANSWER:
answer another's debt in case the latter (E) None of the above.
defaults. The recovery of the down payments
A defense in an action for damages that should be made in the same action
the debtor has sufficient, but unliquidated for rescission. Otherwise, it would be
assets to satisfy the credit acquired when a ground for dismissal under Rule 2,
it becomes due. Sec 4 of Rules of Court.
None of the above. Lease of a real property is covered by
the Statute of Frauds. Furthermore, it
SUGGESTED ANSWER: also consists of interest in real
An action for recovery of down property. Hence, it must be in
payment paid under a rescinded oral writing. (Art 1403, Civil Code)
sale of real property. A contract of guaranty is a promise to
An oral sale of real property is an answer for the debt of another and
unenforceable contract under the hence, it is also covered by the
Statute of Frauds. Since, in the Statute of Frauds. It must be in
problem, the vendee paid down writing before it can be enforced in a
payment, it takes it out of the ambit court action. (Art 1403, Civil Code)
of Statute of Frauds. The rescission The fact that a debtor has
here must be in the sense of unliquidated assets does not excuse
resolution of the reciprocal obligation him from paying his debt.
arising from the contract of sale. If In the technical meaning of rescission
rescinded (resolved) by the vendee under Art 1191 of the Civil Code will
on account of the vendors’ failure to be adhered to, then there is no
deliver the thing sold, the parties will absolutely correct answer. Hence,
go back to their status prior to the letter E is also a possible answer.
contract. If the vendor refuses to
return the down payment, then the IX. Betty entrusted to her agent, Aida,
vendee can file an action to recover several pieces of jewelry to be sold on
the down payment. commission with the express obligation to
If, on the other hand, the vendor and turn over to Betty the proceeds of the
the vendee mutually agree to rescind sale, or to return the jewelries if not sold in
i.e. cancel the contract, the vendee a month's time. Instead of selling the
likewise can file an action for the jewelries, Aida pawned them with the
recovery of the down payment on the
basis of solution indebiti.

“Never Let The Odds Keep You From Pursuing What You Know In Your Heart You Were Meant To Do.”-Leroy Satchel Paige
Page 127 of 180
Civil Law Q&As (2007-2013) hectorchristopher@yahoo.com dbaratbateladot@gmail.com

Tambunting Pawnshop, and used the I will rule in favor of Tambunting. Its good
money for herself. Aida failed to redeem faith takes precedence over the right of
the pawned jewelries and after a month, Betty to recover the jewelries.
Betty discovered what Aida had done. I will rule in favor of Tambunting. Good
Betty brought criminal charges which faith is always presumed. Tambunting's
resulted in Aida's conviction for estafa. lawful acquisition in the ordinary course of
business coupled with good faith gives it
Betty thereafter filed an action against legal right over the jewelries.
Tambunting Pawnshop for the recovery of
the jewelries. Tambunting raised the SUGGESTED ANSWER:
defense of ownership, additionally arguing I will rule in favor of Betty. My ruling
that it is duly licensed to engage in the is based on the Civil Code provision
pawnshop and lending business, and that that one who has lost any movable or
it accepted the mortgage of the jewelry in has been unlawfully deprived thereof
good faith and in the regular course of its may recover it from the person in
business. possession of the same. Tam
bunting's claim of good faith is
If you were the judge, how will you decide inconsequential.
the case? (1%)
Although possession of movable
I will rule in favor of Betty. My ruling property acquired in good faith is
is based on the Civil Code provision equivalent to a title, nevertheless,
that one who has lost any movable or one who has lost any movable or has
has been unlawfully deprived thereof been unlawfully deprived thereof may
may recover it from the person in recover it from the person in
possession of the same. Tam possession of the same. Betty has
bunting's claim of good faith is been deprived unlawfully of her
inconsequential. jewelries by the estafa committed by
I will rule in favor of Betty. Tambunting's Aida. The pledge of the said jewelries
claim of good faith pales into by Aida to Tambunting pawnshop is
insignificance in light of the unlawful void because the pledgor is not the
deprivation of the jewelries. However, owner (Art 2085
equity dictates that Tambunting must be (2), Civil Code). Tambunting’s claim of
reimbursed for the pawn value of the good faith is inconsequential,
jewelries. because, aside from good faith,
Tambunting must prove also that it
acquired the jewelries at a public sale
in order to be able to retain the
jewelries until reimbursed by
“Never Let The Odds Keep You From Pursuing What You Know In Your Heart You Were Meant To Do.”-Leroy Satchel Paige
Page 128 of 180
Civil Law Q&As (2007-2013) hectorchristopher@yahoo.com dbaratbateladot@gmail.com

Betty the amount of loan including If you were the judge, would you grant
interest (Art 559, Civil Code). Arlene's motion? (1%)

The only exception the law allows is Yes, I will grant the motion because the
when there is acquisition in good lease contract between Arlene and Janet
faith of the possessor at a public was not in writing, hence, Janet may not
sale, in which case, the owner cannot enforce any right arising from the same
obtain its return without reimbursing contract.
the price (Dizon v. Suntay, 47 SCRA No, I will not grant the motion because to
160, Sept 29, 1972). allow Arlene to retain the advance
payments would amount to unjust
X. Arlene owns a row of apartment houses enrichment.
in Kamuning, Quezon City. She agreed to Yes, I will grant the motion because the
lease Apartment No. 1 to Janet for a period action for recovery is premature; Janet
of 18 months at the rate of P10,000 per should first secure a judicial rescission of
month. The lease was not covered by any the contract of lease.
contract. Janet promptly gave Arlene two No. I will not grant the motion
months deposit and 18 checks covering because the cause of action does not
the rental payment for 18 months. This seek to enforce any right under the
show of good faith prompted Arlene to contract of lease.
promise Janet that should Arlene decide to
sell the property, she would give Janet the SUGGESTED ANSWER:
right of first refusal. No. I will not grant the motion
X. (1) Not long after Janet moved in, she because the cause of action does not
received news that her application for a seek to enforce any right under the
Master of Laws scholarship at King's contract of lease.
College in London had been approved. Janet is not asking for the continued
Since her acceptance of the scholarship use of the leased premises. Moreover,
entailed a transfer of residence, Janet the contract is aside the ambit of the
asked Arlene to return the advance rental Statute of Frauds as the same has
payments she made. Arlene refused, already been partially performed.
prompting Janet to file an action to recover
the payments. Arlene filed a motion to X. (2)Assume that Janet decided not to
dismiss, claiming that the lease on which accept the scholarship and continued
the action is based, is unenforceable. leasing Apartment No. 1. Midway through

“Never Let The Odds Keep You From Pursuing What You Know In Your Heart You Were Meant To Do.”-Leroy Satchel Paige
Page 129 of 180
Civil Law Q&As (2007-2013) hectorchristopher@yahoo.com dbaratbateladot@gmail.com

the lease period, Arlene decided to sell Frauds under Art 1403 (2)(e) of the
Apartment No. 1 to Jun in breach of her Civil Code. It must be in writing in
promise to Janet to grant her the right of order to be enforceable.
first refusal. Thus, Janet filed an action
seeking the recognition of her right of first
refusal, the payment of damages for the 2012 Taxation Law Exam
violation of this right, and the rescission of
MCQ (October 14, 2012)
the sale between Arlene and Jun.

Which of the following is NOT included in


Is Janet's action meritorious? (1%)
the attributes of juridical capacity?
Juridical capacity is inherent in
Yes, under the Civil Code, a promise to buy
every natural person, and therefore
and sell a determinate thing is reciprocally
it is not acquired.
demandable.
Juridical capacity is lost only
No, the promise to buy and sell a
through death.
determinate thing was not supported by a
Juridical capacity is the fitness to
consideration.
be the subject of legal relations.
Yes, Janet's right of first refusal was clearly
Juridical capacity cannot exist
violated when the property was not
without capacity to act.
offered for sale to her before it was sold to
Jun.
Which of the following is NOT a restriction
No, a right of first refusal involves an
on one’s capacity to act?
interest over real property that must
Minority
be embodied in a written contract to
Marriage
be enforceable.
Deaf-mute
None of the above.
Civil Interdiction

SUGGESTED ANSWER:
SUGGESTED ANSWER:
No, a right of first refusal involves an
This question should be disregarded.
interest over real property that must
(NOTE: There is no correct answer among
be embodied in a written contract to
the choices given. All choices are
be enforceable.
restrictions on one’s capacity to act. While
The right of first refusal involves a
Marriage is the only one not mentioned in
transfer of interest in the real
Articles 38 and 39 of the NCC as a
property. As such, it is covered by the
restriction on
Statute of

“Never Let The Odds Keep You From Pursuing What You Know In Your Heart You Were Meant To Do.”-Leroy Satchel Paige
Page 130 of 180
Civil Law Q&As (2007-2013) hectorchristopher@yahoo.com dbaratbateladot@gmail.com

capacity to act, it restricts the capacity of Yes, the will is not valid under
a married person in cases of adoption.) Philippine law.
No, attestation clause is not an
This attribute or incident of a case act of the testator.
determine whether it is a conflict-of-laws No, the governing law is
case or one covered by domestic law. Spanish law.
Cause of action
Foreign element Note: The facts do not state the Law
Jurisdiction observed by the testator in executing his
Forum non conveniens will. He could have observed Spanish Law
or Philippine Law (see comment of
The capacity of an heir to succeed shall be Tolentino to Art. 815 NCC in
governed by the: 3Tolentino117, 1992). If he observed
national law of the decedent’s heirs Spanish Law, the opposition is not correct
law of the country where the because the will is valid under Spanish
decedent was a resident at the Law, hence choice (d) is the correct
time of his death answer. If he observed Philippine Law, the
national law of the person who opposition is still not correct because
died Philippine Law does not require the
law of the country where the testator to sign the Attestation Clause of
properties of the decedent are his will, said clause not being his act. In
located. such case, choice (c) is the correct
answer).
Atty. BUKO, a Filipino, executed a will while
he was in Spain. The attestation clause of Ramon, a Filipino, executed a will in
the said will does not contain Buko’s Manila, where he left his house and
signature. It is valid under Spanish law. At located in BP Homes Parañaque in favor of
its probate in Manila, it is being opposed his
on the ground that the attestation clause Filipino son, Ramgen. Ramon’s other
does not contain BUKO’s signature. children RJ and Ramona, both Turkish
Is the opposition correct? Choose the best nationals, are disputing the bequest to
answer.. Ramgen. They plotted to kill Ramgen.
Yes, because it is a fatal defect. Ramon learned of the plot, so he tore his
will in two pieces out of anger. Which
statement is most accurate?
The mere act of Ramon Sr. is
immaterial because the will is still
readable.
“Never Let The Odds Keep You From Pursuing What You Know In Your Heart You Were Meant To Do.”-Leroy Satchel Paige
Page 131 of 180
Civil Law Q&As (2007-2013) hectorchristopher@yahoo.com dbaratbateladot@gmail.com

The mere act of tearing the will


amounts to revocation. A Japanese national and a Filipino national
The tearing of the will may entered into a contract for services in
amount to revocation if Thailand. The services will be rendered in
coupled with intent of revoking Singapore. In case of breach, what law will
it. govern?
The act of tearing the will is Thailand law
material. Philippine law
Singapore law
Even if the applicable law is a foreign law, Japanese law
a count in the Philippines may be
constrained to apply Philippine law under Pedro (Filipino) and his wife Jane
any of the following instances, except: (American) executed a joint will in Canada,
when the foreign law, judgment or where such joint will is valid. In case the
contract is contrary to a sound and joint will is probated in Japan, what law will
important public policy of the govern the formalities of the joint will?
forum; American law
when the property subject of Philippine law
the case is located outside of Canadian law
the Philippines; Japanese law
when the foreign law or
judgment is penal in nature; A French national revokes his will in Japan
when the foreign law is procedural where he is domiciled. He then changed
in nature. his domicile to the Philippines where he
died. The revocation of his will in Japan is
If a will is executed by a testator who was valid under Japanese law but invalid under
born a Filipino citizen but became Philippine law. The affected heir is a
naturalized Japanese citizen at the time of Malaysian national residing in the
his death, what law will govern its Philippines. What law will apply?
testamentary provisions if the will is Japanese law
executed in China and the property being Philippine law
disposed is located in Indonesia? French law
Chinese law Malaysian law
Philippine law
Indonesia law
Japanese law

“Never Let The Odds Keep You From Pursuing What You Know In Your Heart You Were Meant To Do.”-Leroy Satchel Paige
Page 132 of 180
Civil Law Q&As (2007-2013) hectorchristopher@yahoo.com dbaratbateladot@gmail.com

In the absence of contrary stipulation in a obligation, but Bill fails or refuses to pay,
marriage settlement, property relations of what law will govern?
Filipino spouses shall be governed by --- American law
Philippines laws Philippine law
Law of the place where the spouses Australian law
reside Japanese law
Law of the place where the (Facts for item numbers 15-18)
properties are situated In 1989, Charice (Filipina) and Justine
Law of the place where they were (American), were married in the
married. Philippines. In 1990, they separated and
Justine went to Las Vegas where he
The will of a Filipino executed in a foreign obtained a divorce in the same year. He
country --- then married another Filipina, Lea, in
cannot be probated in the Canada on January 1, 1992. They had two
Philippines; (2) sons, James and John (who were both
may be probated in the Philippines born in 1992). In 1993, after failing to hear
provided that properties in the from Justine, Charice married Bugoy (a
estate are located in the Filipino), by whom she had a daughter,
Philippines; Regine. In 2009, Regine married James
cannot be probated before the (son of Justine with Lea) in California,
death of the testator; where such marriage is valid.
may be probated in the
Philippines provided it was What is the current status of the marriage
executed in accordance with of Charice and Justine under Philippine
the laws of the place where the laws?
will was executed. Valid
Void
Pedro (Filipino and Bill (American) entered Voidable
into a contract in Australia, whereby it was Dissolved
agreed that Pedro will build a commercial (Note: While Art 26 of the FC does not
building for Bill in the Philippines, and in categorically provide that the first
payment for the construction, Bill will marriage is dissolved by the divorce
transfer and convey his cattle ranch obtained by the foreign spouse abroad,
located in Japan in favor of Pedro. In case but provides that such divorce merely
Pedro performs his gives the Filipino spouse the capacity to
contract a second marriage, it

“Never Let The Odds Keep You From Pursuing What You Know In Your Heart You Were Meant To Do.”-Leroy Satchel Paige
Page 133 of 180
Civil Law Q&As (2007-2013) hectorchristopher@yahoo.com dbaratbateladot@gmail.com

is believed that the dissolution of the first examinations unless he marries Princess.
marriage us the necessary consequence As a consequence of the threat, Ricky
of the foreign divorce.) married Princess. Can the marriage be
annulled on the ground of intimidation
What id the status of the marriage under Article 45 of the Family Code?
between Charice and Bugoy under Choose the best answer.
Philippine laws? Yes, because without the threat,
Valid Ricky would not have married
Void Princess.
Voidable Yes, because the threat to enforce
Unenforceable the claim of Princess vitiates the
consent of Ricky in contracting the
What is the status of the marriage marriage.
between Charice and Bugoy under No, because the threat made
Philippine laws? by Marforth is just and legal.
Valid No, because Marforth is not a party
Void to the contract of marriage
Voidable between Princess and Ricky.
Unenforceable
Audrey, single, bought a parcel of land in
What is the status of the marriage Malolos City from Franco for P 1Million. A
between Regine and James under contract was executed between them
Philippine laws? which already vested upon Audrey full
Valid ownership of the property, although
Void payable in monthly installments for a
Voidable period of four (4) years. One (1) year after
Unenforceable the execution of the contract, Audrey got
married to Arnel. They executed a
Ricky and Princess were sweethearts. marriage settlement whereby they agreed
Princess became pregnant. Knowing that that their properties shall be governed by
Ricky is preparing for the examinations, the regime of conjugal partnership of
Marforth, a lawyer and cousin of Princess, gains. Thereafter, subsequent installments
threatened Ricky with the filing of a were paid from the conjugal partnership
complaint for immorality in the Supreme funds. Is the land conjugal or paraphernal?
Court, thus preventing him from taking

“Never Let The Odds Keep You From Pursuing What You Know In Your Heart You Were Meant To Do.”-Leroy Satchel Paige
Page 134 of 180
Civil Law Q&As (2007-2013) hectorchristopher@yahoo.com dbaratbateladot@gmail.com

The land is conjugal because the the laws of such consular official. Under
installments were paid from the Philippine law, what is the status of the
conjugal partnership funds. marriage of Agay and Topacio? Choose the
The land is paraphernal best answer.
because ownership thereof was Void, because the consular
acquired before the marriage. official only has authority to
The land is both conjugal and solemnize marriages between
paraphernal funds of installments Filipinos.
were paid from both the personal Valid, because according to the
funds of Audrey and the conjugal laws of Australia, such consular
partnership funds. official has authority to celebrate
The land is paraphernal because it the marriage.
was Audrey who purchased the Voidable, because there is an
same. irregularity in the authority of the
consular official to solemnize
Ernesto donated a mobile phone worth P marriages.
32,000 to Hubert orally and delivered the Valid, because such marriage is
unit to Hubert who accepted. Which recognized as valid in the place
statement is most accurate? where it was celebrated.
The donation is void and
Ernesto may get mobile phone (Note: The issues in the problem is
back. whether or not the fact that one of the
The donation is void but Ernesto parties to the marriage was an alien
cannot get the mobile phone back. constituted absence of authority or mere
The donation is voidable and may irregularity of authority. The problem only
be anulled. give the choice, letter (a), in case it is
The donation is valid. interpreted as absence of authority. The
problem does not give a choice in case it
Agay, a Filipino citizen and Topacio, an is interpreted as an irregularity thereby
Australian citizen, got married in the making all the other answers wrong).
consular office of the Philippines in
Australia. According to the laws of
Australia, a marriage solemnized by a Separation of property between spouses
consular official is valid, provided that during the marriage may take place only:
such marriage is celebrated in accordance by agreement of the spouses.
with

“Never Let The Odds Keep You From Pursuing What You Know In Your Heart You Were Meant To Do.”-Leroy Satchel Paige
Page 135 of 180
Civil Law Q&As (2007-2013) hectorchristopher@yahoo.com dbaratbateladot@gmail.com

If one of the spouses has given No trial shall be held without the 6-
ground for legal separation. month cooling off period being
Upon order of the court. observed.
If one spouse has abandoned the The spouses will be entitled to live
other. separately upon the start of the
trial.
The husband may impugn the legitimacy The prosecuting attorney has
of his child but not on the ground that: to conduct his own
the wife is suspected of investigation.
infidelity.
the husband had a serious illness A husband by chance discovered hidden
that prevented him from engaging treasure on the paraphernal property of
in sexual intercourse. his wife. Who owns the discovered
they were living apart. treasure?
he is physically incapable of sexual The half pertaining to the husband
intercourse. (finder) belongs to the conjugal
partnership.
A marriage is void if: The half pertaining to the wife (as
solemnized with a marriage license owner) belongs to the conjugal
issued without complying with the partnership.
required 10-day posting. One half shall belong to the
solemnized by a minister whom the husband as finder and the other
parties believe to have the half shall belong to the wife as
authority. owner of the property.
between parties both 23 years of a and b
age but without parental advice.
none of the above Which of the following marriages is void
for reasons of public policy?
In legal separation, which is not correct? Between brothers and sisters,
The aggrieved spouse may file the whether of the full or half blood.
action within five (5) years from the Between step-parents and step
time of the occurrence of the children.
cause. Between parents-in-law and
children-in-law.
b and c

“Never Let The Odds Keep You From Pursuing What You Know In Your Heart You Were Meant To Do.”-Leroy Satchel Paige
Page 136 of 180
Civil Law Q&As (2007-2013) hectorchristopher@yahoo.com dbaratbateladot@gmail.com

The following constitute the different Children conceived or born outside


circumstances or case of fraud which will a valid marriage.
serves as ground for the annulment of a Children born under a valid
marriage, except? marriage, which was later declared
Non-disclosure of the previous void because of the psychological
conviction by final judgment of the incapacity of either or both of the
other party of a crime involving spouses.
moral turpitude. Children conceived and born
Concealment of a sexually- outside a valid marriage.
transmissible disease, regardless of Children born under a valid
its nature, existing at the time of marriage, but the parents later
the marriage. obtained a legal separation.
Concealment of drug addiction,
habitual alcoholism, homosexuality An illegitimate child may use the surname
or lesbianism existing at the time of his father when his filiation is
of marriage. established in any of the following
Concealment by the wife or the instances, except:
husband of the fact of sexual Filiation has been recognized by
relations prior to the marriage. the father through the record of
birth appearing in the civil register
Which of the following is not a requisite for Admission of filiation by the father
a valid donation propter nuptias? in a public document.
The donation must be made before Private handwritten instrument is
the celebration of the marriage. made by the father acknowledging
The donation shall be his filiation.
automatically revoked in case Affidavit by the mother stating
of non-celebration of the the name of his true father.
marriage.
The donation must be made in Under RA 8043, an adopter is required to
consideration of the marriage. be at least ____ years old and ____ years
The donation must be made in older than the child to be adopted at the
favor of one or both of the future time of the application unless the adopter
spouses. is the parent by nature of the child.
30 and 15
Who are illegitimate children? 27 and 16

“Never Let The Odds Keep You From Pursuing What You Know In Your Heart You Were Meant To Do.”-Leroy Satchel Paige
Page 137 of 180
Civil Law Q&As (2007-2013) hectorchristopher@yahoo.com dbaratbateladot@gmail.com

50 and 10 Subjects the child or allows him to


18 and 15 be subjected to acts of
lasciviousness.
Under RA 8043, a child qualified to be
adopted is any person below _____ years 37. Which of the following statements is
old. wrong?
18 The possessor in bad faith shall
21 reimburse the fruits received and
15 those which the legitimate
16 possessor could have received.
The possessor in bad faith has
Which of the following DOES NOT result in right of reimbursement for
permanent termination of parental necessary expenses and those for
authority? the production, gathering and
Death of the parents. preservation of the fruits.
Death of the child. The possessor in bad faith is not
Emancipation of the child. entitled to a refund of ornamental
Conviction of the parents of a expenses.
crime which carries with it the The possessor in bad faith is
penalty of civil interdiction. entitled to a refund of useful
expenses.
The court, in an action filed for the
purpose, may suspend parental authority Which phrase most accurately completes
if the parent or the person exercising the statement – The expenses incurred in
parental authority commits any of the improvements for the luxury or mere
following acts, except: pleasure shall not be refunded to thew
Treats the child with excessive possessor in bad faith:
harshness or cruelty. but he may remove the objects
Gives the child corrupting orders, for which such expenses have
counsel or example. been incurred, provided that
Compels the child to take up a the thing suffers no injury
course in college against thereby, and that the lawful
his/her will. possessor does not prefer to
retain them.

“Never Let The Odds Keep You From Pursuing What You Know In Your Heart You Were Meant To Do.”-Leroy Satchel Paige
Page 138 of 180
Civil Law Q&As (2007-2013) hectorchristopher@yahoo.com dbaratbateladot@gmail.com

and he may not remove the objects of the dominion over the property as
for which such expenses have been owner. What action is being referred to?
incurred. Accion publiciana
and he may not remove the objects Accion reinvindicatoria
for which such expenses have been Accion interdictal
incurred, unless he pays the value Quieting of Title
they may have at the time he
entered into possession. A summary action to recover physical or
but he may remove the objects for material possession only and must be
which such expenses have been brought within one (1) year from the time
incurred. the cause of action arises. What action is
being referred to?
The following are the limitations on the Accion publiciana
right of ownership imposed by the owner Accion reinvindicatoria
himself, except: Accion interdictal
Will/Succession Quieting of Title
Mortgage
Pledge The following things are property of public
Lease dominion, except:
ports and bridges constructed by
A plenary action for the recovery of the the State.
possession of real estate, upon mere vehicles and weapons of the Armed
allegation and proof of a better right Forces of the Philippines.
thereto, and without allegation of proof of rivers.
title. This action can only be brought after lands reclaimed by the state from
the expiration of one (1) year. What action the sea.
is being referred to?
Accion publiciana SUGGESTED ANSWER:
Accion reinvindicatoria This question should be disregarded
Accion interdictal because there is no correct answer.
Quieting of Title (Note: At first glance, one gets the
impression that vehicles and weapons of
Action to recover real property based on the AFP are not property of the public
ownership. Here, the object is the domain. But they are actually property of
recovery the public dominion under the second
paragraph of Art 420 of

“Never Let The Odds Keep You From Pursuing What You Know In Your Heart You Were Meant To Do.”-Leroy Satchel Paige
Page 139 of 180
Civil Law Q&As (2007-2013) hectorchristopher@yahoo.com dbaratbateladot@gmail.com

the NCC. Property of the state which are the donee accepts the donation.
not for public use but are intended for the donor executes the deed of
some public service are properties of the donation.
public dominion. While the vehicles and the donor knows of the donee’s
weapons of the AFP are not for public use, acceptance even if the latter
they are used for the defense of the State has not received the copy of
which is a public service.) the deed of donation.
the donee confirms that the donor
44. Which of the following statements is has learned the former’s
wrong? acceptance.
patrimonial property of the
state, when no longer intended The following are the elements of an
for public use or for public obligation, except:
service, shall become property Juridical/Legal Tie
of public dominion. Active subject
all property of the State, which is Passive subject
not of public dominion, is Consideration
patrimonial property.
The property of provinces, cities It is a conduct that may consist of giving,
and municipalities is divided into doing, or not doing something.
property for public use and Obligation
patrimonial property. Juridical necessity
Property is either of public Prestation
dominion or of private ownership. Contract
It is a juridical relation arising from lawful,
The following cannot ask for the reduction voluntary and unilateral acts based on the
of inofficious donation, except: principle that no one should unjustly
Creditors of the deceased enrich himself at the expense of another.
Devisees or legatees Quasi-contract
Compulsory heirs of the donor Quasi-delict
The surviving spouse of the donee. Cotract
Delict
Donation is perfected from the moment
--- The following are the elements of quasi-
delict, except:

“Never Let The Odds Keep You From Pursuing What You Know In Your Heart You Were Meant To Do.”-Leroy Satchel Paige
Page 140 of 180
Civil Law Q&As (2007-2013) hectorchristopher@yahoo.com dbaratbateladot@gmail.com

Act or omission It is an international evasion of the faithful


Fault/negligence performance of the obligation.
Damage/injury Negligence
Pre-existing contract Fraud
Delay
A debtor is liable for damages in case of Mistake
delay if he is guilty of any of the following,
except: The following are the requisites of
default (mora) fortuitous event, except:
mistake Cause is independent of the will of
negligence (culpa) the debtor.
breach through contravention of b) The
event
the tenor thereof
is unforeseeable/unavoidable.
Occurrence renders it absolutely
This term refers to a delay on the part of
impossible for the debtor to fulfill
both the debtor and creditor in reciprocal
his obligation in a normal manner;
obligations.
impossibility must be absolute not
Mora accipiendi
partial, otherwise not force
Mora solvendi
majeure.
Compensation morae
Debtor contributed to the
Solution indibiti
aggravation of the injury to the
creditor.
The following are the requisites of mora
solvendi, except:
A debtor may still be held liable for loss or
Obligation pertains to the debtor
damages even if it was caused by a
and is determinate, due,
fortuitous event in any of the following
demandable, and liquidated.
instances, except:
Obligation was performed on
The debtor is guilty of dolo, malice
its maturity date.
or bad faith, has promised the
There is judicial or extrajudicial
same thing to two or more persons
demand by the creditor.
who do not have the same interest.
Failure of the debtor to comply with
The debtor contributed to the loss.
such demand.
The thing to be delivered is
generic.

“Never Let The Odds Keep You From Pursuing What You Know In Your Heart You Were Meant To Do.”-Leroy Satchel Paige
Page 141 of 180
Civil Law Q&As (2007-2013) hectorchristopher@yahoo.com dbaratbateladot@gmail.com

The creditor is guilty of fraud, No, because a motion to dismiss is


negligence or delay or if he a prohibited pleading.
contravened the tenor of the Yes, because Fermin and Toti
obligation. should also pay their share of the
obligation.
Buko, Fermin and Toti bound themselves
solidarily to pay Ayee the amount of P Buko, Fermin and Toti are solidarily
5,000.00. Suppose Buko paid the debtors of Ayee. Twelve (12) years after
obligation, what is his right as against his the obligation became due and
co-debtors? demandable, Buko paid Ayee and later on
Buko cas ask for asked for reimbursement of Fermin’s and
reimbursement from Fermin Toti’s shares. Is Buko correct? Why?
and Toti. No, because the obligation has
Buko can sue Fermin and Toti for already prescribed.
damages. Yes, because the obligation is
Buko can sue for rescission. solidary.
Buko can claim a refund from Ayee. No, because in solidary obligation
any one of the solidary debtors can
Buko, Fermin and Toti bound themselves pay the entire debt.
solidarily to pay Ayee the sum of P Yes, because Fermin and Toti will be
10,000.00. When the obligation became unduly enriched at the expense of
due and demandable, Ayee sued Buko for Buko.
the payment of the P 10,000.00. Buko
moved to dismiss on the ground that there Buko, Fermin and Toti are solidary
was failure to implead Fermin and Toti who debtors under a loan obligation of P
are indispensable parties. Will the motion 300,000.00 which has fallen due. The
to dismiss prosper? Why? creditor has, however, condoned Fermin’s
Yes, because Fermin and Toti entire share in the debt. Since Toti has
should have been impleaded as become insolvent, the creditor makes a
their obligation is solidary. demand on Buko to pay the debt. How
No, because the creditor may much, if any, may Buko be compelled to
proceed against any one of the pay?
solidary debtors or some or all P 200.000.00
of them simultaneously. P 300,000.00
P 100,000.00

“Never Let The Odds Keep You From Pursuing What You Know In Your Heart You Were Meant To Do.”-Leroy Satchel Paige
Page 142 of 180
Civil Law Q&As (2007-2013) hectorchristopher@yahoo.com dbaratbateladot@gmail.com

P 150,000.00 That they be liquidated and


demandable.
Dina bought a car from Jai and delivered a
check in payment of the same. Has Dina 63. Which of the following statements is
paid the obligation? Why? correct?
No, not yet. The delivery of All contracts are perfected by mere
promissory notes payable to consent.
order, or bills of exchange or All contracts are perfected by
other mercantile documents delivery of the object.
shall produce the effect of All contracts are required to be in
payment only when they have writing.
been cashed, or when through All contracts are required to
the fault of the creditor they have a valid consideration.
have been impaired.
Yes, because a check is a valid It is a principle which holds that parties
legal tender of payment. are bound not only by what has been
It depends. If the check is a expressly provided for in the contract but
manager’s check or cashier’s check also to the natural consequences that flow
it will produce the effect of out of such agreement.
payment. Obligatory force of contracts
If it’s an ordinary check, no Mutuality of contracts
payment. Autonomy of contracts
Yes, because a check is as good as Relativity of contracts
cash.
The following are the requisites of legal It is a principle which holds that contracts
compensation, except: must be binding to both parties and its
That each of the obligors is bound validity and effectivity can never be left to
principally and that he be the same the will of one of the parties.
time a principal creditor of the Obligatory force of contracts
other. Mutuality of contracts
That both debts consist in a sum of Autonomy of contracts
money, or if the things due are Relativity of contracts
consumable, they be the same
kind, and also of the same quality if It refers to the rule that a contract
the latter has been stated. is binding not only
between
That the two (2) debts are not
yet due.

“Never Let The Odds Keep You From Pursuing What You Know In Your Heart You Were Meant To Do.”-Leroy Satchel Paige
Page 143 of 180
Civil Law Q&As (2007-2013) hectorchristopher@yahoo.com dbaratbateladot@gmail.com

parties but extends to the heirs, Acceptance of the offer by the


successors in interest, and assignees of offeree.
the parties, provided that the contract Qualified/conditional acceptance of
involved transmissible rights by their the offer, which becomes counter-
nature, or by stipulation or by law. offer.
Obligatory force of contracts Subject matter becomes
Mutuality of contracts illegal/impossible before
Autonomy of contracts acceptance is communicated.
Relativity of contracts
70. Which of the following statements is
It is rule which holds that the freedom of correct?
the parties to contract includes the Offers in interrelated contracts are
freedom to stipulate, provided the perfected upon consent.
stipulations are not contrary to law, Offers in interrelated contracts
morals, good customs, public order or require a single acceptance.
public policy. c) Business
Obligatory force of contracts
advertisements
Mutuality of contracts
are definite offers that require
Autonomy of contracts
specific acceptance.
Relativity of contracts
Advertisements for Bidders are
only invitations to make
The following are the ways by which
proposals and the advertiser is
innominate contracts are regulated, not bound to accept the
except:
highest/lowest bidder, unless it
By the stipulation of the parties.
appears otherwise.
By the general principles of
quasi-contracts and delicts
71. The following are solemn contracts
By the rules governing the most
(Contracts which must appear in writing),
analogous nominate contracts.
except:
By the customs of the place.
Donations of real estate or of
An offer becomes ineffective on any of the
movables if the value exceeds P
following grounds, except:
5,000.00.
Death, civil interdiction,
Stipulation to pay interest in loans.
insanity/insolvency of either party
Sale of land through an agent
before acceptance is conveyed.
(authority must be in writing).

“Never Let The Odds Keep You From Pursuing What You Know In Your Heart You Were Meant To Do.”-Leroy Satchel Paige
Page 144 of 180
Civil Law Q&As (2007-2013) hectorchristopher@yahoo.com dbaratbateladot@gmail.com

Construction contract of a The following are the characteristics of a


building. voidable contract, except:
Effective until set aside.
The following are rescissible contracts, May be assailed/attacked only in an
except: action for that purpose.
Entered into by guardian whenever Can be confirmed or ratified.
ward suffers damage more than ¼ Can be assailed only by either
of value of property. party.
Agreed upon in representation of
absentees, if absentee suffers The following are void contracts, except:
lesion by more than ¼ of value of Pactum commissorium
property. Pactum de non alienando
Contracts where fraud is Pactum leonina
committed on creditor (accion Pacto de retro
pauliana).
Contracts entered into by The borrower in a contract of loan or
minors. mutuum must pay interest to the lender.
If there is an agreement in
The following are the requisites before a writing to the effect.
contract entered into in fraud of creditors As a matter of course.
may be rescinded, except: If the amount borrowed is very
There must be credited existing large.
prior to the celebration of the If the lender so demands at the
contract. maturity date.
There must be fraud, or at least,
the intent to commit fraud to the The liability of the school, its administrators
prejudice of the creditor seeking and teachers, or the individual, entity or
rescission. institution engaged in child care over the
The creditor cannot in any legal minor child or damage caused by the acts or
manner collect his credit omissions of the unemancipated minor while
(subsidiary character of rescission) under their supervision, instruction or
The object of the contract must custody shall be:
be legally in the possession of Joint and subsidiary
a 3rd person in good faith. Principal and solidary

“Never Let The Odds Keep You From Pursuing What You Know In Your Heart You Were Meant To Do.”-Leroy Satchel Paige
Page 145 of 180
Civil Law Q&As (2007-2013) hectorchristopher@yahoo.com dbaratbateladot@gmail.com

Principal and joint quasi-contract


Subsidiary and solidary. civil
natural
The creditor has the right to the fruits of
the thing from the time: Consent was given by one in
the thing is delivered. representation of another but without
the obligation to deliver the authority. The contract is:
things arises. voidable
the contract is perfected. rescissible
the fruits are delivered. void
unenforceable
If one of the parties to the contract is Michael Fermin, without the authority of
without juridical capacity, the contract is: Pascual Lacas, owner of a car, sold the
voidable same car in the name of Mr. Lacas to Atty.
rescissible Buko. The contract between Atty. Buko and
void Mr. Lacas is ---
unenforceable void because of the absence of
consent from the owner, Mr. Lacas.
When both parties to the contract are valid because all of the essential
minors, the contract is: requisites of a contract are present.
voidable unenforceable because Michael
rescissible Fermin had no authority but he
void sold the car in the name of Mr.
unenforceable Lacas, the owner.
rescissible because the contract
When the consent of one of the parties caused lesion to Atty. Buko.
was vitiated, the contract is:
voidable Which of the following contracts is void?
rescissible An oral sale of a parcel of land.
void A sale of land by an agent in a
unenforceable public instrument where his
authority from the principal is
An obligation which is based on equity and oral.
natural law is known as: A donation of a wrist watch worth P
pure 4,500.00.

“Never Let The Odds Keep You From Pursuing What You Know In Your Heart You Were Meant To Do.”-Leroy Satchel Paige
Page 146 of 180
Civil Law Q&As (2007-2013) hectorchristopher@yahoo.com dbaratbateladot@gmail.com

A relatively simulated contract for fulfillment of the obligation


even if he has not tendered
Which of the following expresses a correct payment of the purchase price.
principle of law? Choose the best answer. The contract between the parties is
Failure to disclose facts when there rescissible.
is a duty to reveal them, does not The contract between the
constitute fraud. parties is subject to ratification
Violence or intimidation does not by the parties.
render a contract annullable if
employed not by a contracting 88. Which of the following statements is
party but by a third person. wrong?
A threat to enforce one’s claim Creditors are protected in cases of
through competent authority, if the contracts intended to defraud
claim is legal or just, does not them.
vitiate consent. Contracts take effect only between
Absolute simulation of a the parties, their assign and heirs,
contract always results in a except in case where the rights and
void contract. obligations arising from the
contract are not transmissible by
Aligada orally offered to sell his two- their nature, or by stipulation or by
hectare rice land to Balane for P 10Million. provision of law.
The offer was orally accepted. By If a contract should contain some
agreement, the land was to be delivered stipulation in favor of a third
(through execution of a notarized Deed of person, he may demand its
Sale) and the price was to be paid exactly fulfillment provided he
one-month from their oral agreement. communicated his acceptance to
Which statement is most accurate? the obligor before its revocation.
If Aligada refuses to deliver the In contracts creating real
land on the agreed date despite rights, third persons who come
payment by Balane, the latter may into possession of the object of
not successfully sue Aligada the contract are not bound
because the contract is oral. thereby.
If Aligada refused to deliver the
land, Balane may successfully sue Which phrase most accurately completes
the statement – Any third person who
induces another to violate his contract:

“Never Let The Odds Keep You From Pursuing What You Know In Your Heart You Were Meant To Do.”-Leroy Satchel Paige
Page 147 of 180
Civil Law Q&As (2007-2013) hectorchristopher@yahoo.com dbaratbateladot@gmail.com

shall be liable for damages only if


he is a party to the same contract. The attestation clause contains the
shall be liable for damages to following, except:
the other contracting party. the number of pages used;
shall not be liable for damages to that the testator signed or caused
the other contracting party. another to sign the will and every
shall not be liable for damages if page thereof in the presence of the
the parties are in pari delicto. instrumental witnesses;
notary public;
The requisites of succession are as follows, the instrumental witnesses
except: witnessed and signed the will and
Death of decedent all the pages thereof in the
Transmissible estate presence of the testator and one
Existence and capacity of another.
successor, designated by decedent
or law The following are the formalities required
Payment of Taxes in the execution of holographic will,
except:
The characteristics of succession are as Entirely written;
follows, except: Dated;
It is a legal contract. Signed by testator himself
Only property, rights and Notarized by a notary public.
obligations to the extent of the
value of the inheritance are The following are the grounds for
transmitted. disallowance of wills, except:
The transmission takes place only The formalities required by law
at the time of death. have not been complied with.
The transmission takes place either The testator was insane or
by will or by operation of law. mentally incapable of making will.
The will was executed through
The following rights are extinguished by force or under duress, or influence
death, except: of fear or threats.
Legal support The will contains an attestation
Parental authority clause.
Right to inherit
Agency

“Never Let The Odds Keep You From Pursuing What You Know In Your Heart You Were Meant To Do.”-Leroy Satchel Paige
Page 148 of 180
Civil Law Q&As (2007-2013) hectorchristopher@yahoo.com dbaratbateladot@gmail.com

It is the omission in the testator’s will of consideration, which gives him the right to
one, some or all of the compulsory heirs in buy certain merchandise or specified
direct line, whether living at the time of property, from another person, at anytime
execution of the will or born after the within the agreed period, at a fixed price.
death of the testator. What principle is What contract is being referred to?
being referred to? Option Contract
reserva troncal Contract to Sell
preterition Contract of Sale
fideicommissary Lease
disposicion captatoria
Any disposition made upon the condition Which of the following contracts of sale is
that the heir shall make some provision in void?
his will in favor of the testator or of any Sale of EGM’s car by KRP, EGM’s
other person shall be void. Here, both the agent, whose authority is not
condition and the disposition are void. reduced into writing.
What principle is being referred to? Sale of EGM’s piece of land by
reserva troncal KRP, EGM’s agent, whose
preterition authority is not reduced into
fideicommissary writing.
disposicion captatoria Sale of EGM’s car by KRP, a person
stranger to EGM, without
Which phrase most accurately completes EGM’s consent or authority.
the statement – If at the time the contract Sale of EGM’s piece of land by
of sale is perfected, the thing which is the KRP, a person stranger to EGM,
object of the contract has been entirely without EGM’s consent or authority.
lost:
the buyer bears the risk of loss.
the contract shall be without 2011 Taxation Law
any effect.
Exam MCQ (November
the seller bears the risk of loss.
the buyer may withdraw from the 13, 2011)
contract.
(1)When does a declaration of absence of
a missing person take effect?
A contract granting a privilege to a
person, for which he has paid a Immediately from the issuance of
the declaration of absence.

“Never Let The Odds Keep You From Pursuing What You Know In Your Heart You Were Meant To Do.”-Leroy Satchel Paige
Page 149 of 180
Civil Law Q&As (2007-2013) hectorchristopher@yahoo.com dbaratbateladot@gmail.com

3 months after the publication of accepted the substitution. Later, however,


the declaration of absence. the new debtor became insolvent and
6 months after the publication defaulted in his obligation. What is the
of the declaration of absence. effect of the new debtor’s default upon the
15 days from the issuance of the original debtor?
declaration of absence. The original debtor is freed of
liability since novation took
The authority that school administrators place and this relieved him of
exercise over school children under their his obligation.
supervision, instruction, or custody is The original debtor shall pay or
called perform the obligation with
legal parental authority. recourse to the new debtor.
substitute parental authority. The original debtor remains liable
ordinary parental authority. since he gave no consent to the
special parental authority. substitution.
The original debtor shall pay or
Can future inheritance be the subject of a perform 50% of the obligation to
contract of sale? avoid unjust enrichment on his
No, since it will put the predecessor part.
at the risk of harm from a tempted
buyer, contrary to public policy. Lennie bought a business class ticket from
Yes, since the death of the Alta Airlines. As she checked in, the
decedent is certain to occur. manager downgraded her to economy on
No, since the seller owns no the ground that a Congressman had to be
inheritance while his accommodated in the business class.
predecessor lives. Lennie suffered the discomfort and
Yes, but on the condition that the embarrassment of the downgrade. She
amount of the inheritance can only sued the airlines for quasi-delict but Alta
be ascertained after the obligations Airlines countered that, since her travel
of the estate have been paid. was governed by a contract between
Upon the proposal of a third person, a new them, no quasi-delict could arise. Is the
debtor substituted the original debtor airline correct?
without the latter’s consent. The creditor No, the breach of contract may
in fact be tortious as when it is
tainted as in this case with

“Never Let The Odds Keep You From Pursuing What You Know In Your Heart You Were Meant To Do.”-Leroy Satchel Paige
Page 150 of 180
Civil Law Q&As (2007-2013) hectorchristopher@yahoo.com dbaratbateladot@gmail.com

arbitrariness, gross bad faith, No, since the marriage did not take
and malice. place.
No, denying Lennie the comfort Yes, since all the requisites of a
and amenities of the business class donation of an immovable are
as provided in the ticket is a present.
tortious act. No, since the donation and its
Yes, since the facts show a breach acceptance are not in a public
of contract, not a quasi-delict. instrument.
Yes, since quasi-delict presupposes Yes, since X freely donated the
the absence of a pre- property to Y who became its
existing contractual relation owner.
between the parties.
Rene and Lily got married after a brief
Which of the following is an indispensable courtship. After one month, Lily discovered
requirement in an action for "quieting of that while Rene presented himself as a
title" involving real property? The plaintiff macho man he was actually gay. He would
must not go to bed with her. He kept obscene
be in actual possession of the magazines of nude men and always
property. sought the company of handsome boys.
be the registered owner of the What legal remedy does Lily have?
property. She can file an action for
have legal or equitable title to annulment of marriage on
the property. ground of fraud.
be the beneficial owner of the She can seek a declaration of
property. nullity of the marriage based on
Rene’s psychological incapacity.
X and Y were to marry in 3 months. She can go abroad and file for
Meantime, to express his affection, X divorce in a country that can grant
donated a house and lot to Y, which it.
donation X wrote in a letter to Y. Y wrote She has none since she had the
back, accepting the donation and took opportunity to examine the goods
possession of the property. Before the and freely entered into the
wedding, however, Y suddenly died of marriage.
heart attack. Can Y’s heirs get the
property? Lucio executed a simple deed of donation
of P50 million on time deposit with a bank
in favor of A, B, C, D, and E,

“Never Let The Odds Keep You From Pursuing What You Know In Your Heart You Were Meant To Do.”-Leroy Satchel Paige
Page 151 of 180
Civil Law Q&As (2007-2013) hectorchristopher@yahoo.com dbaratbateladot@gmail.com

without indicating the share of each No, since there was no impediment
donee. All the donees accepted the to Raul selling his inheritance to a
donation in writing. A, one of the donees, stranger.
died. Will B, C, D, and E get A’s share in
the money? When one exercises a right recognized by
Yes, accretion will automatically law, knowing that he thereby causes an
apply to the joint-donees in equal injustice to another, the latter is entitled
shares. to recover damages. This is known as the
Yes, since the donor’s intention is principle of
to give the whole of P50 million to res ipsa loquitur.
the jointdonees in equal shares. damnum absque injuria.
No, A"s share will revert to the vicarious liability.
donor because accretion applies abuse of rights.
only if the joint-donees are
spouses. Which of the following is NOT a basis for
No, A’s share goes to his heirs rendering a disinheritance defective or
since the donation did not imperfect?
provide for reversion to donor. Its cause comes from the guilt
of a spouse in a legal
Raul, Ester, and Rufus inherited a 10- separation case, the innocent-
hectare land from their father. Before the spouse having died.
land could be partitioned, however, Raul The truth of its cause is denied and
sold his hereditary right to Raffy, a not sufficiently proved by evidence.
stranger to the family, for P5 million. Do Its cause is not authorized by the
Ester and Rufus have a remedy for law.
keeping the land within their family? Its cause is not specified.
Yes, they may be subrogated to
Raffy’s right by reimbursing to Manuel came to Manila and married
him within the required time Marianne. Unknown to Marianne, Manuel
what he paid Raul. had been previously convicted in Palawan
Yes, they may be subrogated to of theft and served time for it. After
Raffy’s right provided they buy him Marianne learned of his previous
out before he registers the sale. conviction, she stopped living with him.
No, they can be subrogated to Can Marianne seek the annulment of the
Raffy’s right only with his
conformity.

“Never Let The Odds Keep You From Pursuing What You Know In Your Heart You Were Meant To Do.”-Leroy Satchel Paige
Page 152 of 180
Civil Law Q&As (2007-2013) hectorchristopher@yahoo.com dbaratbateladot@gmail.com

marriage based on Manuel’s nondisclosure since Philippine law does not


of his previous crime? recognize divorce.
No, since the assumption is that All the children are legitimate since
marriage forgives all past wrongs. they were born of the same father
Yes, since the non-disclosure of and mother.
that crime is the equivalent of
fraud, which is a ground for Who can make a donation?
annulment. All persons who can enter into
No, in case of doubt, the law must contracts and dispose of their
be construed to preserve the property.
institution of marriage. All persons who are of legal age
No, since Manuel already served and suffer from no civil interdiction.
the penalty for his crime. All persons who can make a last
will and testament.
Arthur and Helen, both Filipinos, got All persons, whether natural or
married and had 2 children. Arthur later artificial, who own property.
worked in Rome where he acquired Italian
citizenship. He got a divorce from Helen in The liability of the partners, including
Rome but, on returning to the Philippines, industrial partners for partnership
he realized his mistake, asked forgiveness contracts entered into in its name and for
of his wife, and resumed living with her. its account, when all partnership assets
They had 2 more children. What is the have been exhausted is
status of their 4 children? Pro-rata.
The children born before the Joint.
divorce are legitimate but Solidary.
those born after it are not Voluntary.
since Arthur got the divorce
when he had ceased to be a When can a missing person who left
Filipino. someone to administer his property be
The divorce rendered illegitimate declared an absentee by the court? When
the children born before it since the he has been missing for
marriage that begot them had 2 years from the receipt of the last
been nullified. news about him.
The children born before and after 7 years from the receipt of the last
the divorce are all legitimate news about him.

“Never Let The Odds Keep You From Pursuing What You Know In Your Heart You Were Meant To Do.”-Leroy Satchel Paige
Page 153 of 180
Civil Law Q&As (2007-2013) hectorchristopher@yahoo.com dbaratbateladot@gmail.com

10 years from the receipt of the principal two days after the principal died,
last news about him. an event that neither the agent nor the
5 years from the receipt of the buyer knew at the time of the sale. What
last news about him. is the standing of the sale?
Voidable.
Which of the following claims against the Valid.
debtor enjoys preference over the others Void.
with respect to his specific immovable Unenforceable.
property and real rights?
Unpaid price of real property sold, Spouses A and B leased a piece of land
upon the immovable property. belonging to B's parents for 25 years. The
Mortgage credits recorded in the spouses built their house on it worth
registry of property, upon the P300,000.00. Subsequently, in a case that
mortgaged real estate. C filed against A and B, the court found
Taxes due, upon the land or the latter liable to C for P200,000.00.
building. When the sheriff was attaching their
Expenses for the preservation and house for the satisfaction of the judgment,
improvement of property, when the A and B claimed that it was exempt from
law authorizes reimbursement, execution, being a family home. Is this
upon the preserved or improved claim correct?
immovable. Yes, because while B’s parents own
the land, they agreed to have their
When bilateral contracts are vitiated with daughter build her family home on
vices of consent, they are rendered it.
rescissible. No, because there is no judicial
void. declaration that it is a family home.
unenforceable. No, since the land does not
voidable. belong to A and B, it cannot
qualify as a family home.
An agent, authorized by a special power of Yes, because the A and B’s family
attorney to sell a land belonging to the actually lives in that house.
principal succeeded in selling the same to
a buyer according to the instructions given Solomon sold his coconut plantation to
the agent. The agent executed the deed of Aragon, Inc. for P100 million, payable in
absolute sale on behalf of his installments of P10 million per month with
6% interest per annum. Solomon married

“Never Let The Odds Keep You From Pursuing What You Know In Your Heart You Were Meant To Do.”-Leroy Satchel Paige
Page 154 of 180
Civil Law Q&As (2007-2013) hectorchristopher@yahoo.com dbaratbateladot@gmail.com

Lorna after 5 months and they chose When A and B married, they chose
conjugal partnership of gains to govern conjugal partnership of gains to govern
their property relations. When they their property relations. After 3 years, B
married, Aragon had an unpaid balance of succeeded in getting her marriage to A
P50 million plus interest in Solomon’s annulled on ground of the latter’s
favor. To whom will Aragon’s monthly psychological incapacity. What liquidation
payments go after the marriage? procedure will they follow in disposing of
The principal shall go to the their assets?
conjugal partnership but the They will follow the rule governing
interests to Solomon. the liquidation of a conjugal
Both principal and interests shall partnership of gains where the
go to Solomon since they are his party who acted in bad faith
exclusive properties. forfeits his share in the net profits.
Both principal and interests shall Since the marriage has been
go to the conjugal partnership declared void, the rule for
since these become due after the liquidation of absolute community
marriage. of property shall be followed.
The principal shall go to The liquidation of a co-
Solomon but the interests to ownership applies since the
the conjugal partnership. annulment brought their
property relation under the
X and Y, although not suffering from any chapter on
impediment, cohabited as husband and property regimes without
wife without the benefit of marriage. marriage.
Following the birth of their child, the The law on liquidation of
couple got married. A year after, however, partnerships applies.
the court annulled the marriage and X and Y agreed verbally before their
issued a decree of annulment. What is the marriage (a) on the paternity of the
present status of the child? illegitimate child of Y and (b) on the
Legitimated. economic regime that will govern X and
Illegitimate. Y’s property relations. Is the verbal
Natural child. agreement valid?
Legitimate. No, because a marriage
settlement to be valid should
be in writing.

“Never Let The Odds Keep You From Pursuing What You Know In Your Heart You Were Meant To Do.”-Leroy Satchel Paige
Page 155 of 180
Civil Law Q&As (2007-2013) hectorchristopher@yahoo.com dbaratbateladot@gmail.com

Yes, since ante-nuptial agreements Illegitimate, because by the color


need not be in writing. of its skin, the child could not
No, because a marriage settlement possibly be that of Fidel.
cannot include an agreement on Legitimate, because the child
the paternity of an illegitimate was born within a valid
child. marriage.
Yes, since even if it is not a valid Legitimate, because Fidel agreed to
marriage settlement, it is a valid treat the child as his own after
verbal contract. Gloria told him who the father was.

Spouses X and Y have a minor daughter, The husband’s acts of forcibly ejecting his
Z, who needs support for her education. wife without just cause from the conjugal
Both X and Y, who are financially dwelling and refusing to take her back
distressed, could not give the needed constitutes
support to Z. As it happens, Z’s other desertion.
relatives are financially capable of giving recrimination.
that support. From whom may Z first constructive abandonment.
rightfully demand support? From her de facto separation.
grandfather.
brother. In his will, the testator designated X as a
uncle. legatee to receive P2 million for the
first cousin. purpose of buying an ambulance that the
residents of his Barangay can use. What
Fidel, a Filipino with fair complexion, kind of institution is this?
married Gloria. Before the marriage, Gloria a fideicomissary institution.
confessed to Fidel that she was two-month a modal institution.
pregnant with the child of a black African a conditional institution.
who had left the country for good. When a collective institution.
the child was born, Fidel could not accept
it being too black in complexion. What is X insured himself for P5 million,
the status of the child? designating Y, his wife, as his sole
Illegitimate, because Gloria beneficiary. The designation was
confessed that the child is not irrevocable. A few years later, X had their
Fidel’s. marriage annulled in court on the ground
that Y had an existing prior marriage. X

“Never Let The Odds Keep You From Pursuing What You Know In Your Heart You Were Meant To Do.”-Leroy Satchel Paige
Page 156 of 180
Civil Law Q&As (2007-2013) hectorchristopher@yahoo.com dbaratbateladot@gmail.com

subsequently died, Is Y entitled to the Marlon gets 1/2 and Cecilia


insurance benefits? gets 1/2.
Yes, since the insurance was not Marlon gets 3/4 and Cecilia 1/4.
dependent on the marriage.
Yes, since her designation as Contracts take effect only between the
beneficiary was irrevocable. parties or their assigns and heirs, except
No, X’s designation of Y is where the rights and obligations arising
revoked by operation of law from the contract are not transmissible by
upon the annulment of their their nature, by stipulation, or by provision
marriage based on Y’s fault. of law. In the latter case, the assigns or
Yes, since without judicial the heirs are not bound by the contracts.
revocation, X’s designation of Y This is known as the principle of
remains valid and binding. Relativity of contracts.
May a spouse freely donate communal or Freedom to stipulate.
conjugal property without the consent of Mutuality of contracts.
the other? Obligatory force of contracts.
Absolutely not, since the spouses
co-own such property. A buyer ordered 5,000 apples from the
Yes, for properties that the family seller at P20 per apple. The seller
may spare, regardless of value. delivered 6,000 apples. What are the
Yes, provided the donation is rights and obligations of the buyer?
moderate and intended for He can accept all 6,000 apples
charity or family rejoicing. and pay the seller at P20 per
Yes, in a donation mortis causa that apple.
the donor may still revoke in his He can accept all 6,000 apples and
lifetime. pay a lesser price for the 1,000
excess apples.
The decedent died intestate leaving an He can keep the 6,000 apples
estate of P10 million. He left the following without paying for the 1,000 excess
heirs: a) Marlon, a legitimate child and b) since the seller delivered them
Cecilia, the legal spouse. Divide the anyway.
estate. He can cancel the whole
Marlon gets 1/4 and Cecilia gets transaction since the seller violated
3/4. the terms of their agreement.
Marlon gets 2/3 and Cecilia 1/3.

“Never Let The Odds Keep You From Pursuing What You Know In Your Heart You Were Meant To Do.”-Leroy Satchel Paige
Page 157 of 180
Civil Law Q&As (2007-2013) hectorchristopher@yahoo.com dbaratbateladot@gmail.com

Lino entered into a contract to sell with suffered from poisoning caused by a
Ramon, undertaking to convey to the noxious substance found in the sardines.
latter one of the five lots he owns, without Mylene filed a case for damages against
specifying which lot it was, for the price of Acme. Which of the following defenses will
P1 million. Later, the parties could not hold?
agree which of five lots he owned Lino The expiry date of the "Sards"
undertook to sell to Ramon. What is the was clearly printed on its can,
standing of the contract? still the store sold and Mylene
Unenforceable. bought it.
Voidable. Mylene must have detected the
Rescissible. noxious substance in the sardines
Void. by smell, yet she still ate it.
Acme had no transaction with
Knowing that the car had a hidden crack in Mylene; she bought the "Sards"
the engine, X sold it to Y without informing from a store, not directly from
the latter about it. In any event, the deed Acme.
of sale expressly stipulated that X was not Acme enjoys the presumption of
liable for hidden defects. Does Y have the safeness of its canning procedure
right to demand from X a reimbursement and Mylene has not overcome such
of what he spent to repair the engine plus presumption.
damages?
Yes. X is liable whether or not he Fernando executed a will, prohibiting his
was aware of the hidden defect. wife Marina from remarrying after his
Yes, since the defect was not death, at the pain of the legacy of P100
hidden; X knew of it but he Million in her favor becoming a nullity. But
acted in bad faith in not a year after Fernando’s death, Marina was
disclosing the fact to Y. so overwhelmed with love that she
No, because Y is in estoppel, married another man. Is she entitled to
having changed engine without the legacy, the amount of which is well
prior demand. within the capacity of the disposable free
No, because Y waived the warranty portion of
against hidden defects. Fernando’s estate?
Acme Cannery produced sardines in cans Yes, since the prohibition against
known as "Sards." Mylene bought a can of remarrying is absolute, it is
Sards from a store, ate it, and deemed not written.
Yes, because the prohibition is
inhuman and oppressive and

“Never Let The Odds Keep You From Pursuing What You Know In Your Heart You Were Meant To Do.”-Leroy Satchel Paige
Page 158 of 180
Civil Law Q&As (2007-2013) hectorchristopher@yahoo.com dbaratbateladot@gmail.com

violates Marina’s rights as a free It is an ordinary donation since


woman. it was not given to the bride or
No, because the nullity of the groom.
prohibition also nullifies the legacy. It is donation propter nuptias since
No, since such prohibition is it was given with the marriage in
authorized by law and is not mind.
repressive; she could remarry It is an indirect donation propter
but must give up the money. nuptias since the bride would
eventually inherit the property
X, the owner, constituted a 10-year from her parents.
usufruct on his land as well as on the It is a remunatory donation.
building standing on it in Y’s favor. After
flood totally destroyed the building 5 years X and Y, both Filipinos, were married and
later, X told Y that an act of God resided in Spain although they intend to
terminated the usufruct and that he return to the Philippines at some future
should vacate the land. Is X, the owner of time. They have not executed any
the land, correct? marriage settlements. What law governs
No, since the building was their property relations?
destroyed through no fault of Y. They may choose between Spanish
No, since Y still has the right to law and Philippine law.
use the land and the materials Philippine law since they are
left on it. both Filipinos.
Yes, since Y cannot use the land No regime of property relations will
without the building. apply to them.
Yes, since the destruction of the Spanish law since they live in
building without the X’s fault Spain.
terminated the usufruct.
Birth determines personality. Death
In gratitude, the groom’s parents made a extinguishes it. Under what circumstances
donation of a property in writing to the may the personality of a deceased person
bride’s parents shortly before their continue to exist?
children’s wedding. The donation was In case of re-appearance of a
accepted. What is the nature of the missing person presumed dead.
donation?

“Never Let The Odds Keep You From Pursuing What You Know In Your Heart You Were Meant To Do.”-Leroy Satchel Paige
Page 159 of 180
Civil Law Q&As (2007-2013) hectorchristopher@yahoo.com dbaratbateladot@gmail.com

In protecting the works of a the brother or sister in need stops


deceased under intellectual schooling without valid reason.
property laws. the need for support of a
In case of declaration of brother or sister, already of
presumptive death of a missing age, is due to the latter's fault.
spouse.
In the settlement of the estate Virgilio owned a bare and simple
of a deceased person. swimming pool in his garden. MB, a 7-year
old child, surreptitiously entered the
Six tenants sued X, the landowner, for garden and merrily romped around the
willfully denying them water for their ledges of the pool. He accidentally tripped,
farms, which water happened to flow from fell into the pool, and drowned. MB’s
land under X’s control, his intention being parents sued
to force them to leave his properties. Is X Virgilio for damages arising from their
liable for his act and why? child’s death, premised on the principle of
No, because the tenants must be "attractive nuisance". Is Virgilio liable for
content with waiting for rainfall for the death of MB?
their farms. No, the child was 7 years old and
No, since X owns both the land and knew the dangers that the pool
the water. offered.
Yes, because the tenants’ farms Yes, being an attractive nuisance,
have the natural right of access to Virgilio had the duty to prevent
water wherever it is located. children from coming near it.
Yes, since X willfully caused No, since the pool was bare
injury to his tenants contrary and had no enticing or alluring
to morals, good customs or gadgets, floats, or devices in it
public policy. that would attract a 7-year old
child.
Illegitimate brothers and sisters, whether Yes, since Virgilio did not cover the
of full or half-blood, are bound to support swimming pool while not in use to
each other, EXCEPT when prevent children from falling into it.
the brother or sister who needs
support lives in another place.
such brothers and sisters are not
recognized by their father.

“Never Let The Odds Keep You From Pursuing What You Know In Your Heart You Were Meant To Do.”-Leroy Satchel Paige
Page 160 of 180
Civil Law Q&As (2007-2013) hectorchristopher@yahoo.com dbaratbateladot@gmail.com

The term of a 5-year lease contract Yes, the donation is not deemed
between X the lessor and Y the lessee, made until the suspensive
where rents were paid from month to condition has been fulfilled.
month, came to an end. Still, Y continued
using the property with X’s consent. In Illegitimate children, those not recognized
such a case, it is understood that they by their biological fathers, shall use the
impliedly renewed the lease surname of their
from month to month under biological father subject to no
the same conditions as to the condition.
rest. mother or biological father, at
under the same terms and the mother’s discretion.
conditions as before. mother.
under the same terms except the biological father unless he judicially
rent which they or the court must opposes it.
fix.
for only a year, with the rent raised Asiong borrowed P1 million from a bank,
by 10% pursuant to the rental secured by a mortgage on his land.
control law. Without his consent, his friend Boyong
paid the whole loan. Since Asiong
Rex, a philanthropist, donated a valuable lot benefited from the payment, can Boyong
to the municipality on the condition that it compel the bank to subrogate him in its
will build a public school on such lot within 2 right as mortgagee of Asiong's land?
years from its acceptance of the donation. No, but the bank can foreclose and
The municipality properly accepted the pay Boyong back.
donation but did not yet build the public No, since Boyong paid for
school after 2 years. Can Rex revoke the Asiong’s loan without his
donation? approval.
Yes, since the donation is Yes, since a change of creditor took
subject to a resolutory place by novation with the bank’s
condition which was not consent.
fulfilled. Yes, since it is but right that
No, but Rex is entitled to recover Boyong be able to get back his
the value of the land from the money and, if not, to foreclose the
municipality. mortgage in the manner of the
No, the transfer of ownership has bank.
been completed.

“Never Let The Odds Keep You From Pursuing What You Know In Your Heart You Were Meant To Do.”-Leroy Satchel Paige
Page 161 of 180
Civil Law Q&As (2007-2013) hectorchristopher@yahoo.com dbaratbateladot@gmail.com

Congress passed a law imposing taxes on Yes, since Fernando was a solidary
income earned out of a particular activity creditor, payment to him
that was not previously taxed. The law, extinguished the obligation.
however, taxed incomes already earned
within the fiscal year when the law took What happens to the property regimes
effect. Is the law valid? that were subsisting under the New Civil
No, because laws are intended to Code when the Family Code took effect?
be prospective, not retroactive. The original property regimes
No, the law is arbitrary in that it are immutable and remain
taxes income that has already been effective.
spent. Those enjoying specific regimes
Yes, since tax laws are the lifeblood under the New Civil Code may
of the nation. adopt the regime of absolute
Yes, tax laws are an exception; community of property under the
they can be given retroactive Family Code.
effect. Those that married under the New
Civil Code but did not choose any
Rudolf borrowed P1 million from Rodrigo of its regimes shall now be
and Fernando who acted as solidary governed by the regime of absolute
creditors. When the loan matured, Rodrigo community of property.
wrote a letter to Rudolf, demanding They are superseded by the Family
payment of the loan directly to him. Code which has retroactive effect.
Before Rudolf could comply, Fernando The testator executed a will following the
went to see him personally to collect and formalities required by the law on
he paid him. Did Rudolf make a valid succession without designating any heir.
payment? The only testamentary disposition in the
No, since Rudolf should have split will is the recognition of the testator's
the payment between Rodrigo and illegitimate child with a popular actress. Is
Fernando. the will valid?
No, since Rodrigo, the other Yes, since in recognizing his
solidary creditor, already made illegitimate child, the testator has
a prior demand for payment made him his heir.
from Rudolf. No, because the non-designation of
Yes, since the payment covers the heirs defeats the purpose of a will.
whole obligation.

“Never Let The Odds Keep You From Pursuing What You Know In Your Heart You Were Meant To Do.”-Leroy Satchel Paige
Page 162 of 180
Civil Law Q&As (2007-2013) hectorchristopher@yahoo.com dbaratbateladot@gmail.com

No, the will comes to life only when Voidable, because the Judge
the proper heirs are instituted. acted beyond his territorial
Yes, the recognition of an jurisdiction and is administratively
illegitimate heir is an ample liable for the same.
reason for a will. Void, because the Judge did not
solemnize the marriage within the
A left B, his wife, in the Philippines to work premises of his court.
in Egypt but died in that country after a
year’s continuous stay. Two months after X and Y, Filipinos, got married in Los
A’s death, B gave birth to a child, claiming Angeles, USA, using a marriage license
it is A’s child. Who can assail the issued by the Philippine consul in Los
legitimacy of the child? Angeles, acting as Civil Registrar. X and Y
A’s other heirs apart from B. did not know that they were first cousins
The State which has interest in the because their mothers, who were sisters,
welfare of overseas contract were separated when they were quite
workers. young. Since X did not want to continue
Any one who is outraged by B’s with the relation when he heard of it, he
claim. left Y, came to the Philippines and married
No one since A died. Z. Can X be held liable for bigamy?
No since X’s marriage to Y is void
QR and TS who had a marriage license ab initio or did not exist.
requested a newly appointed Judge in No since X acted in good faith,
Manila to marry them on the beach of conscious that public policy did not
Boracay. Since the Judge maintained approve of marriage between first
Boracay as his residence, he agreed. The cousins.
sponsors were all public officials. What is Yes since he married Z without
the status of the marriage. first securing a judicial
Valid, since the improper venue declaration of nullity of his
is merely an irregularity; all the marriage to Y.
elements of a valid marriage Yes since his first marriage to Y in
are present. Los Angeles is valid.
Void, because the couple did not
get local permit for a beach Allan bought Billy’s property through
wedding. Carlos, an agent empowered with a
special power of attorney (SPA) to sell the
same.

“Never Let The Odds Keep You From Pursuing What You Know In Your Heart You Were Meant To Do.”-Leroy Satchel Paige
Page 163 of 180
Civil Law Q&As (2007-2013) hectorchristopher@yahoo.com dbaratbateladot@gmail.com

When Allan was ready to pay as No, the buyer is entitled to a


scheduled, Billy called, directing Allan to customary 30-day extension of his
pay directly to him. On learning of this, obligation to take delivery of the
Carlos, Billy's agent, told Allan to pay goods.
through him as his SPA provided and to No, since there was no express
protect his commission. Faced with two agreement regarding automatic
claimants, Allan consigned the payment in rescission.
court. Billy protested, contending that the No, the seller should first
consignation is ineffective since no tender determine that Y was not justified
of payment was made to him. Is he in failing to appear.
correct? The wife filed a case of legal separation
No, since consignation without against her husband on the ground of
tender of payment is allowed in sexual infidelity
the face of the conflicting without previously exerting earnest efforts
claims on the plaintiff. to come to a compromise with him. The
Yes, as owner of the property sold, judge dismissed the case for having been
Billy can demand payment directly filed without complying with a condition
to himself. precedent. Is the dismissal proper?
Yes, since Allan made no No, efforts at a compromise will
announcement of the tender. only deepen the wife’s anguish.
Yes, a tender of payment is No, since legal separation like
required for a valid consignation. validity of marriage is not
subject to compromise
X sold Y 100 sacks of rice that Y was to agreement for purposes of
pick up from X’s rice mill on a particular filing.
date. Y did not, however, appear on the Yes, to avoid a family feud that is
agreed date to take delivery of the rice. hurtful to everyone.
After one week, X automatically rescinded Yes, since the dispute could have
the sale without notarial notice to Y. Is the been settled with the parties
rescission valid? agreeing to legal separation.
Yes, automatic rescission is
allowed since, having the An Australian living in the Philippines
character of movables and acquired shares of stock worth P10 million
consumables, rice can easily in food manufacturing companies. He died
deteriorate. in Manila, leaving a legal wife and a child
in Australia and a live-in partner with
whom

“Never Let The Odds Keep You From Pursuing What You Know In Your Heart You Were Meant To Do.”-Leroy Satchel Paige
Page 164 of 180
Civil Law Q&As (2007-2013) hectorchristopher@yahoo.com dbaratbateladot@gmail.com

he had two children in Manila. He also left Yes, after full payment, the action
a will, done according to Philippine laws, became imprescriptible.
leaving all his properties to his live-in
partner and their children. What law will A court declared Ricardo, an old bachelor,
govern the validity of the disposition in the an absentee and appointed Cicero
will? administrator of his property. After a year,
Australia law since his legal wife it was discovered that Ricardo had died
and legitimate child are Australians abroad. What is the effect of the fact of his
and domiciled in Australia. death on the administration of his
Australian law since the property?
intrinsic validity of the With Ricardo no longer an absentee
provisions of a will is governed but a deceased person, Cicero will
by the decedent’s national law. cease to be administrator of his
Philippine law since the decedent properties.
died in Manila and he executed his The administration shall be
will according to such law. given by the court having
Philippine law since the decedent’s jurisdiction over the intestate
properties are in the proceedings to a new
Philippines. administrator whom it will
appoint.
X bought a land from Y, paying him cash. Cicero automatically becomes
Since they were friends, they did not administrator of Ricardo’s estate
execute any document of sale. After 7 years, until judicially relieved.
the heirs of X asked Y to execute a deed of Cicero’s alienations of Ricardo's
absolute sale to formalize the verbal sale to property will be set aside.
their father. Unwilling to do so, X’s heirs filed
an action for specific performance against Y. Baldo, a rejected suitor, intimidated Judy
Will their action prosper? into marrying him. While she wanted to
No, after more than 6 years, question the validity of their marriage two
the action to enforce the verbal years after the intimidation ceased, Judy
agreement has already decided in the meantime to freely cohabit
elapsed. with Baldo. After more than 5 years
No, since the sale cannot under the following their wedding, Judy wants to file
Statute of Frauds be enforced. a case for annulment of marriage against
Yes, since X bought the land and
paid Y for it.

“Never Let The Odds Keep You From Pursuing What You Know In Your Heart You Were Meant To Do.”-Leroy Satchel Paige
Page 165 of 180
Civil Law Q&As (2007-2013) hectorchristopher@yahoo.com dbaratbateladot@gmail.com

Baldo on ground of lack of consent. Will all heirs in the higher level are disqualified
her action prosper? or unable to inherit?
Yes, the action for annulment is Nephews and nieces.
imprescriptible. Brothers and sisters.
No, since the marriage was State.
merely voidable and Judy Other collateral relatives up to the
ratified it by freely cohabiting 5th degree of consanguinity.
with Baldo after the force and
intimidation had ceased. Roy and Carlos both undertook a contract
No, since the action prescribed 5 to deliver to Sam in Manila a boat docked
years from the date of the in Subic. Before they could deliver it,
celebration of the marriage. however, the boat sank in a storm. The
Yes, because the marriage was contract provides that fortuitous event
celebrated without Judy's consent shall not exempt Roy and Carlos from their
freely given. obligation. Owing to the loss of the motor
boat, such obligation is deemed converted
Is the wife who leaves her husband into one of indemnity for damages. Is the
without just cause entitled to support? liability of Roy and Carlos joint or solidary?
No, because the wife must always Neither solidary nor joint since they
be submissive and respectful to the cannot waive the defense of
husband. fortuitous event to which they are
Yes. The marriage not having been entitled.
dissolved, the husband continues Solidary or joint upon the discretion
to have an obligation to support his of Sam.
wife. Solidary since Roy and Carlos failed
No, because in leaving the to perform their obligation to
conjugal home without just deliver the motor boat.
cause, she forfeits her right to Joint since the conversion of
support. their liability to one of
Yes, since the right to receive indemnity for damages made it
support is not subject to any joint.
condition.
Joanne married James, a person with no
In the order of intestate succession where known relatives. Through James' hard
the decedent is legitimate, who is the last work, he and his wife Joane prospered.
intestate heirs or heir who will inherit if When James died, his estate alone

“Never Let The Odds Keep You From Pursuing What You Know In Your Heart You Were Meant To Do.”-Leroy Satchel Paige
Page 166 of 180
Civil Law Q&As (2007-2013) hectorchristopher@yahoo.com dbaratbateladot@gmail.com

amounted to P100 million. If, in his will, Ric and Josie, Filipinos, have been
James designates Joanne as his only heir, sweethearts for 5 years. While working in
what will be the free portion of his estate. a European country where the execution
Joanne gets all; estate has no free of joint wills are allowed, the two of them
portion left. executed a joint holographic will where
Joanne gets 1/2; the other half they named each other as sole heir of the
is free portion. other in case either of them dies.
Joanne gets 1/3; the remaining 2/3 Unfortunately, Ric died a year later. Can
is free portion. Josie have the joint will successfully
Joanne gets 1/4; the remaining 3/4 probated in the Philippines?
is free portion. Yes, in the highest interest of
comity of nations and to honor the
A warranty inherent in a contract of sale, wishes of the deceased.
whether or not mentioned in it, is known No, since Philippine law
as the prohibits the execution of joint
warranty on quality. wills and such law is binding on
warranty against hidden defects. Ric and Josie even abroad.
warranty against eviction. Yes, since they executed their joint
warranty in merchantability. will out of mutual love and care,
values that the generally accepted
The doctrine of stare decisis prescribes principles of international law
adherence to precedents in order to accepts.
promote the stability of the law. But the Yes, since it is valid in the country
doctrine can be abandoned where it was executed, applying
When adherence to it would result the principle of "lex loci
in the Government’s loss of its celebrationis."
case.
When the application of the ML inherited from his father P5 million in
doctrine would cause great legitime but he waived it in a public
prejudice to a foreign national. instrument in favor of his sister QY who
When necessary to promote the accepted the waiver in writing. But as it
passage of a new law. happened, ML borrowed P6 million from PF
When the precedent has before the waiver. PF objected to the
ceased to be beneficial and waiver and filed an action for its rescission
useful. on the ground that he had the right to
ML’s P5

“Never Let The Odds Keep You From Pursuing What You Know In Your Heart You Were Meant To Do.”-Leroy Satchel Paige
Page 167 of 180
Civil Law Q&As (2007-2013) hectorchristopher@yahoo.com dbaratbateladot@gmail.com

million legitime as partial settlement of (B) Yes, insofar as


Arnold
what ML owed him since ML has proved to
acknowledged Mary as his
be insolvent. Does PF, as creditor, have
illegitimate child.
the right to rescind the waiver?
None, since the marriage did not
No, because the waiver in favor of
take place.
his sister QY amounts to a donation
Yes, if they acquired properties
and she already accepted it.
while living together as husband
Yes, because the waiver is
and wife.
prejudicial to the interest of a
Joseph, a 17-year old Filipino, married
third person whose interest is
Jenny, a 21-year old American in Illinois,
recognized by law.
USA, where the marriage was valid. Their
No, PF must wait for ML to become
parents gave full consent to the marriage
solvent and, thereafter, sue him for
of their children. After three years, Joseph
the unpaid loan.
filed a petition in the USA to promptly
Yes, because a legitime cannot be
divorce Jenny and this was granted. When
waived in favor of a specific heir; it
Joseph turned 25 years, he returned to the
must be divided among all the
Philippines and married Leonora. What is
other heirs.
the status of this second marriage?
Void, because he did not cause
While engaged to be married, Arnold and
the judicial issuance of
Josephine agreed in a public instrument to
declaration of the nullity of his
adopt out the economic regime of
first marriage to Jenny before
absolute community of property. Arnold
marrying Leonora.
acknowledged in the same instrument that
Valid, because Joseph's marriage to
Josephine’s daughter
Jenny is void, he being only 17
Mary, is his illegitimate child. But
years of age when he married her.
Josephine died before the marriage could
Valid, because his marriage to
take place. Does the marriage settlement
Leonora has all the elements of a
have any significance?
valid marriage.
None, since the instrument
Void, because Joseph is still
containing the marriage settlement
considered married to Jenny since
is essentially void for containing an
the Philippines does not recognize
unrelated matter.
divorce.

“Never Let The Odds Keep You From Pursuing What You Know In Your Heart You Were Meant To Do.”-Leroy Satchel Paige
Page 168 of 180
Civil Law Q&As (2007-2013) hectorchristopher@yahoo.com dbaratbateladot@gmail.com

T died intestate, leaving an estate of X, who was abroad, phoned his brother, Y,
P9,000,000. He left as heirs three authorizing him to sell X’s parcel of land in
legitimate children, namely, A, B, and C. A Pasay. X sent the title to Y by courier
has two children, D and E. Before he died, service. Acting for his brother, Y executed
A irrevocably repudiated his inheritance a notarized deed of absolute sale of the
from T in a public instrument filed with the land to Z after receiving payment. What is
court. How much, if any, will D and E, as the status of the sale?
A’s children, get from T’s estate? Valid, since a notarized deed of
Each of D and E will get absolute sale covered the
P1,500,000 by right of transaction and full payment was
representation since their father made.
repudiated his inheritance. Void, since X should have
Each of D and E will get P2,225,000 authorized agent Y in writing
because they will inherit from the to sell the land.
estate equally with B and C. Valid, since Y was truly his brother
D and E will get none because X’s agent and entrusted with the
of the repudiation; "B" and "C" title needed to effect the sale.
will get A’s share by right of Valid, since the buyer could file an
accretion. action to compel X to execute a
Each of D and E will get P2,000,000 deed of sale.
because the law gives them some
advantage due to the demise of In a true pacto de retro sale, the title and
"A". ownership of the property sold are
immediately vested in the vendee a retro
No decree of legal separation can be subject only to the resolutory condition of
issued repurchase by the vendor a retro within
unless the children’s welfare is the stipulated period. This is known as
attended to first. equitable mortgage.
without prior efforts at conventional redemption.
reconciliation shown to be legal redemption.
futile. equity of redemption.
unless the court first directs A natural obligation under the New Civil
mediation of the parties. Code of the Philippines is one which
without prior investigation
conducted by a public prosecutor.

“Never Let The Odds Keep You From Pursuing What You Know In Your Heart You Were Meant To Do.”-Leroy Satchel Paige
Page 169 of 180
Civil Law Q&As (2007-2013) hectorchristopher@yahoo.com dbaratbateladot@gmail.com

the obligor has a moral obligation When the donor gives donations without
to do, otherwise entitling the reserving sufficient funds for his support
obligee to damages. or for the support of his dependents, his
refers to an obligation in writing to donations are
do or not to do. Rescissible, since it results in
the obligee may enforce through economic lesion of more than 25%
the court if violated by the obligor. of the value of his properties.
cannot be judicially enforced Voidable, since his consent to the
but authorizes the obligee to donation is vitiated by mindless
retain the obligor’s payment or kindness.
performance. Void, since it amounts to wanton
expenditure beyond his means.
The husband assumed sole administration Reducible to the extent that
of the family’s mango plantation since his the donations impaired the
wife worked abroad. support due to himself and his
Subsequently, without his wife’s dependents.
knowledge, the husband entered into an
antichretic transaction with a company, Anne owed Bessy P1 million due on
giving it possession and management of October 1, 2011 but failed to pay her on
the plantation with power to harvest and due date. Bessy sent a demand letter to
sell the fruits and to apply the proceeds to Anne giving her 5 days from receipt within
the payment of a loan he got. What is the which to pay. Two days after receipt of the
standing of the contract? letter, Anne personally offered to pay
It is void in the absence of the Bessy in manager's check but the latter
wife’s consent. refused to accept the same. The 5 days
It is void absent an authorization lapsed. May
from the court. Anne’s obligation be considered
The transaction is void and can extinguished?
neither be ratified by the wife nor Yes, since Bessy’s refusal of the
authorized by the court. manager’s check, which is
It is considered a continuing presumed funded, amounts to a
offer by the parties, perfected satisfaction of the obligation.
only upon the wife’s No, since tender of payment
acceptance or the court’s even in cash, if refused, will
authorization. not discharge the obligation
without proper consignation in
court.

“Never Let The Odds Keep You From Pursuing What You Know In Your Heart You Were Meant To Do.”-Leroy Satchel Paige
Page 170 of 180
Civil Law Q&As (2007-2013) hectorchristopher@yahoo.com dbaratbateladot@gmail.com

Yes, since Anne tendered payment The owner of a thing cannot use it in a
of the full amount due. way that will injure the right of a third
No, since a manager’s check is not person. Thus, every building or land is
considered legal tender in the subject to the easement which prohibits
Philippines. its proprietor or possessor from
committing nuisance like noise, jarring,
The residents of a subdivision have been offensive odor, and smoke. This principle
using an open strip of land as passage to is known as
the highway for over 30 years. The owner Jus vindicandi.
of that land decided, however, to close it Sic utere tuo ut alienum non
in preparation for building his house on it. laedas.
The residents protested, claiming that Jus dispondendi.
they became owners of the land through Jus abutendi.
acquisitive prescription, having been in
possession of the same in the concept of Janice and Jennifer are sisters. Janice sued
owners, publicly, peacefully, and Jennifer and Laura, Jennifer’s business
continuously for more than 30 years. Is partner for recovery of property with
this claim correct? damages. The complaint did not allege
No, the residents have not that Janice exerted earnest efforts to come
been in continuous possession to a compromise with the defendants and
of the land since they merely that such efforts failed. The judge
passed through it in going to dismissed the complaint outright for
the highway. failure to comply with a condition
No, the owner did not abandon his precedent. Is the dismissal in order?
right to the property; he merely No, since Laura is a stranger to
tolerated his neighbors’ use of it for the sisters, Janice has no moral
passage. obligation to settle with her.
Yes, residents of the subdivision Yes, since court should promote
have become owners by acquisitive amicable settlement among
prescription. relatives.
Yes, community ownership by Yes, since members of the same
prescription prevails over private family, as parties to the suit, are
claims. required to exert earnest efforts to
settle their disputes before coming
to court.

“Never Let The Odds Keep You From Pursuing What You Know In Your Heart You Were Meant To Do.”-Leroy Satchel Paige
Page 171 of 180
Civil Law Q&As (2007-2013) hectorchristopher@yahoo.com dbaratbateladot@gmail.com

No, the family council, which would When does the regime of conjugal
ordinarily mediate the dispute, has partnership of gains begin to exist?
been eliminated under the Family At the moment the parties take
Code. and declare each other as
husband and wife before
X borrowed money from a bank, secured officiating officer.
by a mortgage on the land of Y, his close At the time the spouses acquire
friend. When the loan matured, Y offered properties through joint efforts.
to pay the bank but it refused since On the date the future spouses
Y was not the borrower. Is the bank’s executed their marriage
action correct? settlements because this is the
Yes, since X, the true borrower, did starting point of their marital
not give his consent to Y’s offer to relationship.
pay. On the date agreed upon by the
No, since anybody can discharge future spouses in their marriage
X’s obligation to his benefit. settlements since their agreement
No, since Y, the owner of the is the law between them.
collateral, has an interest in
the payment of the obligation. Josie, 18, married Dante, 25, without her
Yes, since it was X who has an parents’ knowledge and consent, and lived
obligation to the bank. with him. After a year, Josie returned to
her parents’ home, complained of the
The right of a mortgagor in a judicial unbearable battering she was getting from
foreclosure to redeem the mortgaged Dante, and expressed a desire to have her
property after his default in the marriage with him annulled. Who may
performance of the conditions of the bring the action?
mortgage but before the sale of the Dante.
mortgaged property or confirmation of the Her parents.
sale by the court, is known as Josie herself.
accion publiciana. The State.
equity of redemption.
pacto de retro. X, a married man, cohabited with Y, an
right of redemption. unmarried woman. Their relation bore
them BB, a baby boy. Subsequently, after
X became a widower, he married Y. Was
BB legitimated by that marriage?

“Never Let The Odds Keep You From Pursuing What You Know In Your Heart You Were Meant To Do.”-Leroy Satchel Paige
Page 172 of 180
Civil Law Q&As (2007-2013) hectorchristopher@yahoo.com dbaratbateladot@gmail.com

Yes, since his parents are now Yes, as long as they leave sufficient
lawfully married. property for themselves and for
Yes, since he is an innocent party their dependents.
and the marriage rectified the
wrong done him. X owed Y P1.5 million. In his will, X gave Y
No, since once illegitimate, a legacy of P1 million but the will provided
child shall always remain that this legacy is to be set off against the
illegitimate. P1.5 million X owed Y. After the set off, X
No, since his parents were not still owed Y P500,000. Can Y still collect
qualified to marry each other this amount?
when he was conceived. Yes, because the designation of
Y as legatee created a new and
The presence of a vice of consent vitiates separate juridical relationship
the consent of a party in a contract and between them, that of testator-
this renders the contract legatee.
Rescissible. It depends upon the discretion of
Unenforceable. the probate court if a claim is filed
Voidable. in the testate proceedings.
Void. No, because the intention of the
testator in giving the legacy is to
Can common-law spouses donate abrogate his entire obligation to Y.
properties of substantial value to one No, because X had no instruction in
another? his will to deliver more than the
No, they are only allowed to legacy of P1 million to Y.
give moderate gifts to each
other during family rejoicing. Josie owned a lot worth P5 million prior to
No, they cannot give anything of her marriage to Rey. Subsequently, their
value to each other to prevent conjugal partnership spent P3 million for
placing their legitimate relatives at the construction of a house on the lot. The
a disadvantage. construction resulted in an increase in the
Yes, unlike the case of legally value of the house and lot to P9 million.
married spouses, such donations Who owns the house and the lot?
are not prohibited.

“Never Let The Odds Keep You From Pursuing What You Know In Your Heart You Were Meant To Do.”-Leroy Satchel Paige
Page 173 of 180
Civil Law Q&As (2007-2013) hectorchristopher@yahoo.com dbaratbateladot@gmail.com

(A) Josie and the conjugal When fortuitous circumstances


partnership of gains will own both prevented the plaintiff from filing
on a 50-50 basis. the case sooner.
Josie will own both since the When the plaintiff is in
value of the house and the possession of the property.
increase in the property’s
value is less than her lot’s Conrad and Linda, both 20 years old,
value; but she applied for a marriage license, making it
is to reimburse conjugal appear that they were over 25. They
partnership expenses. married without their parents’ knowledge
Josie still owns the lot, it being her before an unsuspecting judge. After the
exclusive property, but the house couple has been in cohabitation for 6
belongs to the conjugal years,
partnership. Linda’s parents filed an action to annul the
The house and lot shall both belong marriage on ground of lack of parental
to the conjugal partnership, with consent. Will the case prosper?
Josie entitled to reimbursement for No, since only the couple can
the value of the lot. question the validity of their
marriage after they became 21
An action for reconveyance of a registered of age; their cohabitation also
piece of land may be brought against the convalidated the marriage.
owner appearing on the title based on a No, since Linda’s parents made no
claim that the latter merely holds such allegations that earnest efforts
title in trust for the plaintiff. The action have been made to come to a
prescribes, however, within 10 years from compromise with Conrad and Linda
the registration of the deed or the date of and which efforts failed.
the issuance of the certificate of title of Yes, since the marriage is voidable,
the property as long as the trust had not the couple being below 21 years of
been repudiated. What is the exception to age when they married.
this 10-year prescriptive period? Yes, since Linda’s parents never
When the plaintiff had no notice of gave their consent to the marriage.
the deed or the issuance of the
certificate of title. Pepito executed a will that he and 3
When the title holder concealed the attesting witnesses signed following the
matter from the plaintiff. formalities of law, except that the Notary
Public failed to come. Two days later, the

“Never Let The Odds Keep You From Pursuing What You Know In Your Heart You Were Meant To Do.”-Leroy Satchel Paige
Page 174 of 180
Civil Law Q&As (2007-2013) hectorchristopher@yahoo.com dbaratbateladot@gmail.com

Notary Public notarized the will in his law Separate since their property
office where all signatories to the will relations with their legal
acknowledged that the testator signed the spouses are still subsisting.
will in the presence of the witnesses and Co-ownership since they agreed to
that the latter themselves signed the will work for their mutual benefit.
in the presence of the testator and of one Communal since they earned the
another. Was the will validly notarized? same as common-law spouses.
No, since it was not notarized on
the occasion when the signatories What is the prescriptive period for filing an
affixed their signatures on the will. action for revocation of a donation based
Yes, since the Notary Public on acts of ingratitude of the donee?
has to be present only when 5 years from the perfection of the
the signatories acknowledged donation.
the acts required of them in 1 year from the perfection of
relation to the will. the donation.
Yes, but the defect in the mere 4 years from the perfection of the
notarization of the will is not fatal donation.
to its execution. Such action does not prescribe.
No, since the notary public did not
require the signatories to sign their Before Karen married Karl, she inherited
respective attestations again. P5 million from her deceased mother
which amount she brought into the
Venecio and Ester lived as common-law marriage. She later used part of the
spouses since both have been married to money to buy a new Mercedes Benz in her
other persons from whom they had been name, which Karen and her husband used
separated in fact for several years. as a family car. Is the car a conjugal or
Hardworking and bright, each earned Karen’s exclusive property?
incomes from their respective professions It is conjugal property since the
and enterprises. What is the nature of spouses use it as a family car.
their incomes? It is Karen’s exclusive property
Conjugal since they earned the since it is in her name.
same while living as husband and It is conjugal property having been
wife. bought during the marriage.

“Never Let The Odds Keep You From Pursuing What You Know In Your Heart You Were Meant To Do.”-Leroy Satchel Paige
Page 175 of 180
Civil Law Q&As (2007-2013) hectorchristopher@yahoo.com dbaratbateladot@gmail.com

(D) It is Karen’s should the lessees sue for damages? (1%)


exclusive
(1). A, the owner
property since she bought it
with her own money. (2). B, the engineer

Because of X’s gross negligence, Y (3). both A & B


suffered injuries that resulted in the
abortion of the foetus she carried. Y sued SUGGESTED ANSWER:

X for, among other damages, P1 million for


3. Both A & B.
the death of a family member. Is Y entitled
to indemnity for the death of the foetus The lessee may proceed against A for
she carried? breach of contract, and against B for
Yes, since the foetus is already tort or statutory liability. Under
regarded as a child from Article 1654
conception, though unborn. of the New Civil Code, the lessor is
No, since X’s would not have obliged to make all the necessary
known that the accident would repairs in order to keep the leased
result in Y’s abortion. property suitable for the use to which
No, since birth determines it has been devoted. Consequently,
personality, the accident did under Article 1659 NCC, the
not result in the death of a proprietor of a building or structure
person. is responsible for the damages
Yes, since the mother believed in resulting from its total or partial
her heart that she lost a child. collapse, if it is due to lack of
necessary repairs.

Under Article 1723, NCC, the


2010 Civil Law Exam MCQ engineer or architect who drew up
(September 12, 2010) the plans and specifications for a
building is liable for damage if 15
No.II. Multiple choice. years from the completion of the
structure the same should collapse by
(A). A had a 4-storey building which was a reason of a defect by those plans
constructed by Engineer B. After five and specifications, or due to the
years, the building developed cracks and defects in the ground. This liability
its stairway eventually gave way and maybe enforced against the architect
collapsed, resulting to injuries to some or engineer even by a third party who
lessees. Who has no privity of contract with the
architect or engineer under Article
2192, NCC.
“Never Let The Odds Keep You From Pursuing What You Know In Your Heart You Were Meant To Do.”-Leroy Satchel Paige
Page 176 of 180
Civil Law Q&As (2007-2013) hectorchristopher@yahoo.com dbaratbateladot@gmail.com

ALTERNATIVE ANSWER: O, owner of Lot A, learning that Japanese


soldiers may have buried gold and other
No.1. A , the owner .
treasures at the adjoining vacant Lot B

The lessee can sue only the lessor for belonging to spouses X & Y, excavated in

breach of contract under Article 1659 Lot B where she succeeded in unearthing

in relation to Article 1654, NCC. The gold and precious stones. How will the

lessee cannot sue the architect or the treasures found by O be divided? (1%)

engineer because there was no


(1). 100% to O as finder
privity of contracts between them.
When sued, however, the lessor may
(2). 50% to O and 50% to the spouses X
file a third party claim against the
and Y
architect or the engineer.

(3). 50% to O and 50% to the state


ANOTHER ALTERNATIVE ANSWER:

No. 2. B, the Engineer . (4). None of the above.

Under Article 1723 the engineer or SUGGESTED ANSWER:


architect who drew up the plans and
specifications for a building is liable No. 4. None of the above.

for damages if within 15 years from


The general rule is that the treasure
the completion of the structure, the
shall belong to the spouses X and Y,
same should collapse by reason of a
the owner of Lot B. Under Article 438
defect in those plans and
(NCC), the exception is that when the
specifications, or due to the defects
discovery of a hidden treasure is
in the ground. Under Article 2192
made on the property of another and
(NCC), however, if the damages
by chance, one-half thereof shall
should be the result of any of the
belong to the owner of the land and
defects in the construction
the other one-half is allowed to the
mentioned in Art 1723, NCC, the third
finder. In the problem, the finding of
person suffering damages may
the treasure was not by chance
proceed only against the engineer or
because O knew that the treasure
architect or contractor within the
was in Lot B. While a trespasser is
period fixed therein. The damages
also not entitled to any share, and
suffered by the lessee in the problem
there is no indication in the problem
are clearly those resulting from
whether or not O was a trespasser, O
defects in the construction plans or
is not entitled to a
specifications.

“Never Let The Odds Keep You From Pursuing What You Know In Your Heart You Were Meant To Do.”-Leroy Satchel Paige
Page 177 of 180
Civil Law Q&As (2007-2013) hectorchristopher@yahoo.com dbaratbateladot@gmail.com

share because the finding was not or mother alone, even in a public
“by chance.” document, is not sufficient because the
father and mother did not have a
special power of attorney for the
purpose. Under Article 745 (NCC), the
A executed a Deed of Donation in favor of
donee must accept the donation
B, a bachelor, covering a parcel of land
personally, or through an authorized
valued at P1 million. B was, however, out
person with a special power of attorney
of the country at the time. For the
for the purpose; otherwise, the
donation to be valid, (1%)
donation shall be void.

(1). B may e-mail A accepting the No.3 is also false. B cannot accept
donation.
the donation anytime at his
convenience. Under Article 749 NCC,
(2). The donation may be accepted by B’s
the donee may accept the donation
father with whom he lives.
only during the lifetime of the donor.
(3). B can accept the donation anytime
A executed a 5-page notarial will before a
convenient to him.
notary public and three witnesses. All of

(4). B’s mother who has a general power them signed each and every page of the will.

of attorney may accept the donation for


One of the witnesses was B, the father of
him.
one of the legatees to the will. What is the
(5). None of the above is sufficient to effect of B being a witness to the will?
make B’s acceptance valid (1%)

SUGGESTED ANSWER: (1). The will is invalidated

No. 5 None of the above is sufficient (2). The will is valid and effective
to make B's acceptance valid .
(3). The legacy given to B’s child is not
Since the donation covered an valid
immovable property, the donation
and the acceptance must be in public SUGGESTED ANSWER:

document and e-mail is not a public


No. 3. The legacy given to B's child is
document. Hence, No.1 is false.
not valid.

No. 2 and No.4 are both false. The


The validity of the will is not affected
acceptance by the donee’s father
by the legacy in favor of the son of an
alone

“Never Let The Odds Keep You From Pursuing What You Know In Your Heart You Were Meant To Do.”-Leroy Satchel Paige
Page 178 of 180
Civil Law Q&As (2007-2013) hectorchristopher@yahoo.com dbaratbateladot@gmail.com

attesting witness to the will. ALTERNATIVE ANSWER:


However, the said legacy is void
under Article 823 NCC. (D). all the above

ALTERNATIVE ANSWER: (2). A deposit made in compliance with a


legal obligation is:
No. 2 .The will is valid and effective.

(A). an extrajudicial deposit;


Under Article 823 ( NCC ),the legacy
given in favor of the son of an
(B). a voluntary deposit;
instrumental witness to a will has no
effect on the validity of the will.
(C). a necessary deposit;
Hence, the will is valid and effective.
(D). a deposit with a
warehouseman;

2007 Civil Law Exam (E). letters a and b

MCQ (September 09, SUGGESTED ANSWER:

2007)
(C). a necessary deposit

No.IX. Multiple choice: Choose the right


(3). A contract of antichresis is always:
answer. (2% each)

(A). a written contract;


(1). The parties to a bailment are the:

(B). a contract, with a stipulation that the


(A). bailor;
debt will be paid through receipt of the
fruits of an immovable;
(B). bailee;

(C). Involves the payment of interests, if


comodatario;
owing;

(D). all the above;


(D). All of the above;

(E). letters a and b


(E). Letters a and b

SUGGESTED ANSWER:
SUGGESTED ANSWER:

(E). letters a and b

“Never Let The Odds Keep You From Pursuing What You Know In Your Heart You Were Meant To Do.”-Leroy Satchel Paige
Page 179 of 180
Civil Law Q&As (2007-2013) hectorchristopher@yahoo.com dbaratbateladot@gmail.com

(D). All of the above; (D). 1/3 of the total debts must be
represented by the approving creditors;
(4). An, assignee in a proceeding under
the
(E). Letters a and b
Insolvency Law does not have the duty of:
SUGGESTED ANSWER:
(A). suing to recover the properties of the
state of the insolvent debtor; (C). 3/5 of the number of creditors
should agree to the settlement;
(B). selling property of the insolvent
debtor;
[Note: Items 4&5 on Insolvency Law
are not included within the coverage
(C). ensuring that a debtor
of Civil Law but Commercial Law. It is
corporation operate the business
therefore suggested that the
efficiently and effectively while the
examinees be given full credit for the
proceedings are pending;
two items regardless of their

(D). collecting and discharging debts owed answers.]

to the insolvent debtor.

SUGGESTED ANSWER:
References:

(C). ensuring that a debtor



corporation operate the business Answers to Bar Examination
efficiently and effectively while the Questions by the UP LAW
proceedings are pending; COMPLEX (2007, 2009, 2010)

UP LAW REVIEW
(5). In order to obtain approval of the
proposed settlement of the debtor in an
PHILIPPINE ASSOCIATION OF LAW
insolvency proceeding. SCHOOLS (2008)

lawphil.net
(A). the court must initiate the proposal

(B). 2/3 of the number of creditors should


agree to the settlement;

(C). 3/5 of the number of creditors


should agree to the settlement;

“Never Let The Odds Keep You From Pursuing What You Know In Your Heart You Were Meant To Do.”-Leroy Satchel Paige
Page 180 of 180

You might also like